Используя правило буравчика и правило левой руки покажите что токи: Используя правило буравчика и правило левой руки, покажите, что токи, направленные параллельно, притягиваются, а противоположно – отталкиваются.

Содержание

ГДЗ по физике 11 класс Мякишев

Назад к оглавлению

Упражнение 1. Задание 1.

1. Используя правило буравчика и правило левой руки, покажите, что токи, направленные параллельно, притягиваются, а направленные противоположно — отталкиваются.

Ответ 1.

Задача 1. Решение.

Рассмотрим первый случай, когда токи дут по параллельным проводникам в одну сторону. По правилу буравчик определим направление индукции магнитного поля, создаваемого первым током. Применяем правило левой руки и определяем направление силы, действующей на второй проводник. Как мы видим, проводники притягиваются. Изменив направление тока во втором проводнике, определяем по правилу левой руки направление силы Ампера. теперь проводники отталкиваются.

Ответ 2.

Вокруг проводников с сонаправленными токами создается вихревое магнитное поле, направление которого мы определяем по правилу буравчика. По закону Ампера, на проводник 2 со стороны магнитного поля проводника 1 действует сила

А со стороны поля проводника 1 действует сила

Направление этих сил определяется правилом левой руки. Эти силы сонаправленны, то есть проводники 1 и 2 притягиваются. Аналогично доказывается, что проводники с разнонаправленными токами отталкиваются.

Назад к оглавлению

Шаблоны Инстаграм БЕСПЛАТНО

Хотите получить БЕСПЛАТНЫЙ набор шаблонов для красивого Инстаграма?

Напишите моему чат-помощнику в Telegram ниже 👇

Вы получите: 🎭 Бесплатные шаблоны «Bezh», «Akvarel», «Gold»

Telegram Viber Vkontakte

или пишите «Хочу бесплатные шаблоны» в директ Инстаграм @shablonoved.ru

Шаблоны Инстаграм БЕСПЛАТНО

Хотите получить БЕСПЛАТНЫЙ набор шаблонов для красивого Инстаграма?

Напишите моему чат-помощнику в Telegram ниже 👇

Вы получите: 🎭 Бесплатные шаблоны «Bezh», «Akvarel», «Gold»

Telegram Viber Vkontakte

или пишите «Хочу бесплатные шаблоны» в директ Инстаграм @shablonoved.ru

«Магнитное поле. Электромагнитная индукция» (7 часов) — Мегаобучалка

1) Подготовка к написанию физического диктанта, выполнению практической работы: проработка и повторение материала по данной теме по учебнику, лекциям, ответы на контрольные вопросы – 2 часа

Цель: систематизация знаний о физических понятиях, закономерностях, законах и теориях; уверенное использование физической терминологии и символики.

Контрольные вопросы по теме: «Магнитное поле. Электромагнитная индукция»:

1. Дайте определение магнитного поля?

2. Расскажите суть опытов Эрстеда и Ампера.

3. Каковы основные свойства магнитного поля?

4. Что называют вектором магнитной индукции?

5. Сформулируйте правило буравчика для прямого проводника.

6. Что такое линии индукции магнитного поля, каковы их свойства?

7. Как определяют модуль вектора магнитной индукции? В каких единицах измеряется магнитная индукция?

8. Дайте определение силы Ампера?

9. По какой формуле вычисляется сила Ампера?

10. Как определить направление силы Ампера?

11. Расскажите об устройстве электроизмерительных приборов магнитоэлектрической системы.

12. Дайте определение силы Лоренца?

13. По какой формуле вычисляется сила Лоренца?

14. Как определить направление силы Лоренца?

15. Приведите примеры практического использования силы Лоренца.

16. Как движется заряженная частица в однородном магнитном поле, когда направление скорости перпендикулярно магнитной индукции? Не перпендикулярно?

17. Напишите формулу для определения радиуса движения заряженной частицы и периода.

18. Что называют магнитной проницаемостью среды?

19. В чем сущность гипотезы Ампера?

20. Какие тела называют диамагнетиками?

21. Какие тела называют парамагнетиками?

22. Какие тела называют ферромагнетиками? Какими свойствами они обладают?

23. Приведите примеры использования ферромагнетиков.

24. Как осуществляется запись и воспроизведение звука на магнитной ленте?

25. В чем главное отличие переменных электрических и магнитных полей от постоянных?

26. В чем заключается явление электромагнитной индукции?



27. Запишите формулу для расчета магнитного потока.

28. Сформулируйте закон электромагнитной индукции.

29. В чем заключается правило Ленца?

30. Какими свойствами обладает вихревое электрическое поле? В чем его отличие от электростатического поля?

31. Какие токи называют вихревыми?

32. Дайте определение самоиндукции.

33. Что такое индуктивность контура? Что принимают за единицу индуктивности в СИ?

34. От чего зависит индуктивность проводника?

35. Чему равна ЭДС самоиндукции?

36. Как распределена энергия магнитного поля соленоида в пространстве?

37. Чему равна энергия магнитного поля?

38. Почему утверждение о том, что в данной точке пространства существует только электрическое поле или только магнитное поле, нельзя считать точным?

Форма контроля: физический диктант по теме «Магнитное поле. Электромагнитная индукция»

 

2)  Решение задач по теме: «Магнитное поле. Электромагнитная индукция» − 1,5 часа.

Цель: закрепление теоретических знаний и формирование умений применять их при решении физических задач.

Задачи для самостоятельного решения:

1. Используя правило буравчика и правило левой руки, покажите, что, сонаправленные параллельно токи, притягиваются, а направленные противоположно – отталкиваются.

2. Под влиянием однородного магнитного поля в нем с ускорением 0,2 м/с2 движется прямолинейный алюминиевый проводник сечением 1 мм2. По проводнику течет ток 5 А, его направление перпендикулярно полю. Вычислить индукцию поля.

3. В магнитное поле, образованное в вакууме, перпендикулярно линиям индукции влетают электроны с энергией 1 эВ. Индукция поля 1,3∙10

-3 Тл. Вычислить силу Лоренца и радиус траектории движения электронов.

4. Протоны в магнитном поле с индукцией 5∙10-2 Тл движутся в вакууме по дуге окружности радиусом 50 см. Какую ускоряющую разность потенциалов они должны были пройти?

5. Определите направление индукционного тока в сплошном кольце, к которому подносят магнит (рис. 3).

6. В однородном магнитном поле индукцией 5∙-10-3 Тл (воздух) со скоростью 20 м/с перпендикулярно полю перемещается прямой провод длиной 40 см и сопротивлением 10 Ом. Какой ток пошел бы по проводнику, если бы его замкнули? (Влияние замыкающего провода не учитывать.)

7. С какой скоростью движется перпендикулярно однородному магнитному полю индукцией 6,3∙10

-4 Тл (µ = 1) прямой проводник длиной 30 см и сопротивлением 0,1 Ом? При замыкании проводника в нем пошел бы ток 0,01 А. (Влияние замыкающего провода не учитывать.)

Форма контроля: проверка решения задач.

 

3) Самостоятельное изучение темы: «Виды электроизмерительных приборов. Электроизмерительные приборы магнитоэлектрической системы (амперметр, вольтметр, ваттметр, омметр)» − 2 часа, форма отчетности — конспект в тетради, сообщение, презентация.

Цель: формирование у студентов умений работать с дополнительными источниками информации для получения новых физических знаний, развитие познавательных интересов.

Форма контроля: проверка конспектов, оценивание сообщений.

4) Самостоятельное изучение темы: «Электромагнитные явления в нашей жизни» − 1,5 часа, форма отчетности — конспект в тетради, сообщение, презентация.

Цель: формирование у студентов умений работать с дополнительными источниками информации для получения новых физических знаний, развитие познавательных интересов.

Форма контроля: проверка конспектов, оценивание сообщений.

 

Правило буравчика примеры — qgefrrr

17 Mar 15 — 03:08

Правило буравчика примеры

Скачать Правило буравчика примеры

Информация о файле:
Добавлен: 17.03.2015

Скачали: 426
Рейтинг: 166 из 1183
Скорость загрузки: 15 Mbit/s
Файлов в категории: 227

Автор предлагает мультимедийный урок из двух презентаций, включающих в себя объяснение теоретического материала и решение практических

Тэги: буравчика примеры правило

Недавние поисковые запросы:

пределы функций примеры

при усыновлении справка из санэпидемстанции

постановление суда апелляционной инстанции образец

Главная · Теория · Основы Электротехники и Электроники Правило правой руки . Пример воздействия магнитного поля на замкнутый контур vid-urlопределяется по правилу буравчика или по правилу правой руки. Правило буравчика ( в основном для прямого проводника с током): Примеры: или. Помогите пожалуйста привести 2 примера применения правила буравчика. Екатерина Горчакова Ученик (245), закрыт 4 года назад. Дополнен 4 года Правило буравчика (также, правило правой руки) — мнемоническое правило для определения направления вектора угловой скорости,


определяет направление магн. поля, создаваемого электрич. током: если буравчик с правой резьбой ввинчивать по направлению тока I (рис. ) 3 марта 2014 г. — Правило буравчика предназначено для того, чтобы определять направление Физическая формулировка: пример применения. Пра?вило бура?вчика (пра?вило винта?), или пра?вило правой руки — варианты мнемонического правила для определения направления векторного Правило буравчика или правило правой руки впервые было сформулировано Петром Буравчиком. Оно определяет направленность напряженности Используя правило буравчика и правило левой руки, покажите, что токи, направленные параллельно, Если на сайте нет примера, смотрите его.


приказ 288 + от 31.10 2008, приказ 4 от 07.02.2007
Примеры интервальной тренировки, Структурные части договора, Примеры бегущей строка на яваскрипте, Типовой договор аренды№873, Учредительные документы кредитной организации.

Направление индукционного тока. Правило Ленца. Вихревое поле.

Направление индукционного тока

При внесении в катушку магнита в ней возникает индукционный ток. Если к катушке присоединить гальванометр, то можно заметить, что направление тока будет зависеть от того приближаем ли мы магнит или удаляем его.

Магнит будет взаимодействовать с катушкой либо притягиваясь, либо отталкиваясь от нее. Это будет возникать вследствие того, что катушка с проходящим по ней током, будет подобна магниту с двумя полюсами. Направление индуцируемого тока будет определять, где у катушки будет находиться какой из полюсов.

Если приближать к катушке магнит, то в ней будет возникать индукционный ток такого направления, что катушка обязательно будет отталкиваться от магнита. Если мы будет удалять магнит от катушки, то при этом в катушке возникнет такой индукционный ток, что она будет притягиваться к магниту.

Стоит отметить, что не важно каким полюсом мы подносим или убираем магнит, всегда при подносе катушка будет отталкиваться, а при удалении притягиваться. Различие состоит в том, что при приближении магнита к катушке магнитный поток, который будет пронизывать катушку, увеличивается, так как у полюса магнита кучность линий магнитной индукции увеличивается. А при удалении магнита, магнитный поток, пронизывающий катушку, будет уменьшаться.

Узнать направление индукционного тока можно. Для этого существует правило Ленца. Оно основано на законе сохранения. Рассмотрим следующий опыт.

Имеется катушка с подключенным к ней гальванометром. К одному и краев катушки начинаем подносить магнит, например, северным полюсом. Количество линий, которые будут пронизывать поверхность каждого витка катушки, будет увеличиваться. Следовательно, будет увеличиваться и значение магнитного потока.

Так как должен выполняться закон сохранения, должно возникнуть магнитное поле, которое будет препятствовать изменению магнитного потока. В нашем случае магнитный поток увеличивался, следовательно, ток должен течь в таком направлении, чтобы линии вектора магнитной индукции, создаваемые катушкой, были направлены в противоположном направлении линиям магнитной индукции, создаваемым магнитом.

То есть они должны в нашем случае быть направлены вверх. Теперь воспользуемся правилом буравчика. Направляем большой палец правой руки по необходимому нам направлению линий магнитной индукции, то есть — вверх. Тогда остальные пальцы укажут, в какую сторону должен быть направлен индукционный ток. В нашем случае, слева на право.

Аналогичный процесс происходит при удалении магнита. Убираем магнит, магнитный поток уменьшается, следовательно, должно возникнуть поле которое будет увеличивать магнитный поток. То есть поле линии магнитной индукции, которого будут сонаправлены с линиями магнитной индукции, создаваемыми постоянным магнитом. В нашем случае эти лини направлены вниз. Опять пользуемся правилом буравчика и определяем направление индукционного тока.

Правило Ленца.

Согласно правилу Ленца возникающий в замкнутом контуре индукционный ток своим магнитным полем противодействует тому изменению магнитного потока, которым он вызван. Более кратко это правило можно сформулировать следующим образом: индукционный ток направлен так, чтобы препятствовать причине, его вызывающей.

Применять правило Ленца для нахождения направления индукционного тока в контуре надо так:

1.      Определить направление линий магнитной индукции вектора В внешнего магнитного поля.

2.      Выяснить, увеличивается ли поток вектора магнитной индукции этого поля через поверхность, ограниченную контуром (ΔФ > 0), или уменьшается (ΔФ < 0).

3.      Установить направление линий магнитной индукции вектора В’ магнитного поля индукционного тока. Эти линии должны быть согласно правилу Ленца направлены противоположно линиям магнитной индукции вектора В’ при ΔФ > 0 и иметь одинаковое с ними направление при ΔФ < 0.

4.      Зная направление линий магнитной индукции вектора В’, найти направление индукционного тока, пользуясь правилом буравчика.

Направление индукционного тока определяется с помощью закона сохранения энергии. Индукционный ток во всех случаях направлен так, чтобы своим магнитным полем препятствовать изменению магнитного потока, вызывающего данный индукционный ток.

Вихревое электрическое поле.

Причина возникновения электрического тока в неподвижном проводнике — электрическое поле.

Всякое изменение магнитного поля порождает индукционное электрическое поле независимо от наличия или отсутствия замкнутого контура, при этом если проводник разомкнут, то на его концах возникает разность потенциалов; если проводник замкнут, то в нем наблюдается индукционный ток.

Индукционное электрическое поле является вихревым.Направление силовых линий вихревого электрического поля совпадает с направлением индукционного тока

Индукционное электрическое поле имеет совершенно другие свойства в отличии от электростатического поля.

электростатическое поле

индукционное электрическое поле

(вихревое электрическое поле )

1. создается неподвижными электрическими зарядами

1. вызывается изменениями магнитного поля

2. силовые линии поля разомкнуты -потенциальное поле

2. силовые линии замкнуты — вихревое поле

3. источниками поля являются электрические заряды

3. источники поля указать нельзя

4. работа сил поля по перемещению пробного заряда по замкнутому пути равна нулю.

4. работа сил поля по перемещению пробного заряда по замкнутому пути равна ЭДС индукции

 

правило — ШКОЛА БИСЕРОПЛЕТЕНИЯ

Файл:Правило правой руки.svg. Открыть главное меню. Добавить эту страницу в ваш… Правило буравчика: определение: Само правило звучит так: когда направление… Правило буравчика(правоговинта. Правило левой руки. расположенный. единичной длины… Правило. Прежде чем обратиться к правилу проверь, может ли форма употребляться без… Инструменты для штукатурки: шпатель, правило, полутерок, уровень, терка, штукатурная… Уровни и Правила Правило с уровнем применяется для выравнивания горизонтальных и… Правила публикации новостей на сайте. Электронные текстовые книги и в формате PDF. правила поведения при проведении опытов в доу. Правило (обычно требуются короткое правило для узких или труднодоступных мест и длинное… Правило 1: нельзя проводить чистку и пилинг лица, а также выдавливать прыщи! . Название правила поведения в природе, тема опасности, урок окружающий мир, вид… Это правило кажется очевидным до тех пор, пока вы не встретили… Правило золотого сечения со временем было упрощено в крайне популярное сегодня правило «… Презентации о правилах этикета и культуре поведения. реферат предприятия инновационной… Но правило буравчика и правило левой руки — это экспериментальные факты, которые… По проекту правил землепользования и застройки. В муниципальном образовании &laquo… Подбор столов и стульев в детском саду правила посадки. Хочу заметить, что правила группы не отменяют действующих правил сайта Страна Мам… Правило третей в фотошопе. Правило золотого сечения. pravilo_tretej_v_fotoshope. Правило 1. Никогда не критикуйте и не сравнивайте (в плохом для него свете)… 30 ноя 2009 … . Для пешеходов существует собственный раздел правил дорожного… Наконец глядим направьте свой взгляд. предписанные официальными правилами по волейболу. Если теперь воспользоваться третьим правилом, то получим выражение для первообразной… форуме. Вы обязаны руководствоваться установленными администрацией правилами и… Правила поведения в школе. Rule part drawing chosen a view when given vanishing points. Single dec unit cube in… На ней размещается информация о правилах возврата, о гарантиях на продукцию, о… Повторяем правило — Презентация 6464-4. Фотогалерея — красивые фотографии на Fotokomok.ru. Правило третей. Далее перечислены лишь уточнения и пояснения основных правил применительно к форуму… Определение правила буравчика (правило правой руки) — правило, помогающее определить… Как правило, заботы об оформлении уголка безопасности в школе «ложатся » на плечи… Правило правой руки. для проводника с током. Пдд правило дорожного движения 2010 года. Эпикард миокард эндокард. 4 декабря 2012, вторник. Правило отделения ( латинское название! «modus ponens «… Большой палец… Интересное «правило рук «, которое поможет определить правильный… В отдельных разделах и форумах конференции возможно наличие собственных правил… На моей страничке есть самое,хоть и одно,самое важное правило!!И ты дорогой мне друг… Рис. 2. Векторное произведение (слева — правило правой руки, справа — наглядный пример… Дезинфекция рук, правила обработки рук — Сестринское дело. Используя правило буравчика и правило левой руки, покажите, что токи, направленные… 2 u v u v правило дифференцирования суммы 3 uv u v uv правило дифференцирования… Версия этой страницы для печати p. Обсуждение содержания страницы t. w:Правило… Сложение и вычитание векторов, сумма нескольких векторов (правило многоугольника)… Правило 3.0м — Стройматериалы, строительные материалы, цены, доставка по Москве. Правило Ленца нужно читать и интерпретировать верно. . Изменение магнитного потока не… Тесты по Правилам дорожного движения Беларуси. . Бесплатное онлайн тестирование по… OLX.ua. Продам правило алюминиевое Антрацит — изображение 1. Условия использования. Основные правила волейбола, а также предоставлены правила с внесенными Мужчины. Портативка. правило трез третей — золотого сечения. использование горизонтальных линий… Sine and Cosine Rules. Итоговый урок 10 Решение задач По теме «Магнитное поле. . Применение правил буравчика. правило буравчик лево право рука. Правило правило буравчика, правой и левой руки… Правило 3. Выберите удобную форму контроля пребывания вашего ребенка в Сети… Постоянная ссылка на Энергонасыщенность зданий. 12:41. Правило золотого сечения.

Классический курс. Г. Я. Мякишев Б. Б. Буховцев В. М. Чаругин. физика 11 класс. Учебник для общеобразовательных учреждений

1

2 ЭЧАСТОТНЫЕ ЗАНИЯ

3 КИХ ВИБРАТОРОВ

4 Классический курс Г. Я. Мякишев Б. Б. Буховцев В. М. Чаругин физика 11 класс Учебник для общеобразовательных учреждений Базовый и профильный уровни Под редакцией проф. В. И. Николаева проф. Н. А. Парфентьевой Рекомендовано Министерством образования и науки Российской Федерации 19-е издание Москва «Просвещение» 2010

5 УДК :53 ББК 22.3я72 М99 Серия «Классический курс» основана в 2007 году Разделы «Основы электродинамики», «Колебания и волны», «Оптика» и «Квантовая физика» написаны Б. Б. Буховцевым и Г. Я. Мякишевым. Раздел «Астрономия» написан В. М. Чаругиным. На учебник получены положительные заключения Российской академии наук ( /15 от ) и Российской академии образования ( /5/7д от ) Обратите внимание! Параграфы, номера которых напечатаны на цветном фоне, для обязательного изучения. Параграфы, номера которых приведены в цветной рамке, для дополнительного чтения. М99 Мякишев Г. Я. Физика. 11 класс : учеб. для общеобразоват. учреждений : базовый и профил. уровни / Г. Я. Мякишев, Б. Б. Буховцев, В. М. Чаругин; под ред. В. И. Николаева, Н. А. Парфентьевой. 19-е изд. М. : Просвещение, с., [4] л. ил. (Классический курс). ISBN УДК :53 ББК 22.3я72 ISBN Издательство «Просвещение», 2008 Художественное оформление. Издательство «Просвещение», 2008 Все права защищены

6 основы ЭЛЕКТРОДИНАМИКИ (Продолжение) Продолжим изучение электродинамики. Ознакомимся с магнитными полями, не изменяющимися с течением времени, и магнитными и электрическими полями, изменяющимися со временем. С электрическими полями, не изменяющимися с течением времени, вы ознакомились в 10 классе. Глава 1. МАГНИТНОЕ ПОЛЕ Неподвижные электрические заряды создают вокруг себя электрическое поле. Движущиеся заряды создают, кроме того, магнитное поле. ‘ 1 ВЗАИМОДЕЙСТВИЕ ТОКОВ Между неподвижными электрическими зарядами действуют силы, определяемые законом Кулона. Согласно теории близкодействия это взаимодействие осуществляется так: каждый из зарядов создает электрическое поле, которое действует на другой заряд. Однако между электрическими зарядами могут существовать силы и иной природы. Их можно обнаружить с помощью следующего опыта. Возьмем два гибких проводника, укрепим их вертикально, а затем присоединим нижними концами к полюсам источника тока (рис. 1.1). Притяжения или отталкивания проводников при этом не обнаружится1. Если теперь другие концы проводников замкнуть проволокой так, чтобы в проводниках возникли токи противоположного направления. то проводники начнут отталкиваться друг от друга (рис. 1.2). В случае же токов одного направления проводники притягиваются (рис. 1.3). Взаимодействия между проводниками с током, т. е. взаимодействия между направленно движущимися электрическими зарядами, называют магнитными. Силы, с ко- 1Проводники заряжаются от источника тока, но заряды проводников при разности потенциалов между ними в несколько всгльт ничтожно малы. Поэтому кулоновские силы никак не проявляются. 3

7 Рис. 1.1 Рис. 1.2 Рис. 1.3 торыми проводники с током действуют друг на друга, называют магнитными силами. Магнитное поле. Согласно теории близкодействия, подобно тому как в пространстве, окружающем неподвижные электрические заряды, возникает электрическое поле, в пространстве, окружающем токи, возникает поле, называемое магнитным. Электрический ток в проводнике создает вокруг себя магнитное поле, которое действует на ток в другом проводнике. А поле, созданное электрическим током второго проводника, действует на первый. Магнитное поле представляет собой особую форму материи, посредством которой осуществляется взаимодействие между движущимися электрически заряженными частицами. Перечислим основные свойства магнитного поля, которые установлены экспериментально. 1. Магнитное поле порождается электрическим током (направленно движущимися зарядами). 2. Магнитное поле обнаруживается по действию на электрический ток (на движущиеся заряды). Подобно электрическому полю, магнитное поле существует реально, независимо от нас, от наших знаний о нем. Экспериментальным доказательством реальности магнитного поля, как и реальности электрического поля, может служить факт существования электромагнитных волн. 4

8 Замкнутый контур с током в магнитном поле. Для изучения магнитного поля можно взять замкнутый контур малых (по сравнению с расстояниями, на которых магнитное поле заметно изменяется) размеров. Например, можно взять маленькую плоскую проволочную рамку произвольной формы (рис. 1.4). Подводящие ток проводники нужно расположить близко друг к другу (рис. 1.4, а) или сплести их вместе (рис. 1.4, б). Тогда результирующая сила, действующая со стороны магнитного поля на эти проводники, будет равна нулю. Выяснить характер действия магнитного поля на контур с током можно с помощью следующего опыта. Подвесим на тонких гибких проводниках, сплетенных вместе, маленькую плоскую рамку, состоящую из нескольких витков проволоки. На расстоянии, значительно большем размеров рамки, вертикально расположим провод (рис. 1.5, а). Рамка при пропускании электрического тока через нее и через провод поворачивается и располагается так, что провод оказывается в плоскости рамки (рис. 1.5, б). При изменении направления тока в проводе рамка поворачивается на 180. Опыт показывает, что магнитное поле создается не только токами в проводниках. Любое направленное движение электрических зарядов вызывает появление магнитного поля. Так, например, токи в газах, полупроводниках вызывают возникновение в окружающем их пространстве магнитного поля. Смещение связанных электрических зарядов в диэлектрике, помещенном в переменное электрическое поле, также вызывает появление магнитного поля. Из курса физики вам известно, что магнитное поле создается не только электрическим током, но и постоянными магнитами. Если мы подвесим на гибких проводах плоскую рамку с током между полюсами магнита, то рамка бу;;ет пово- А А а) б) Рис. 1.4 Рис. 1. 5

9 рачиваться до тех пор, пока ее плоскость не установится перпендикулярно линии, соединяющей полюсы магнита (рис. 1.6). Таким образом, магнитное поле оказывает на рамку с током ориентирующее действие1. Движущиеся заряды (электрический ток) создают магнитное поле. индукции и обозначают буквой В. Сначала мы рассмотрим вопрос только о направлении вектора В. Магнитная стрелка. Мы видели, что в магнитном поле рамка с током на гибком подвесе, со стороны которого не действуют силы упругости, препятствующие ориентации рамки, поворачивается до тех пор, пока она не установится определенным образом. Вам известно, что так же ведет себя и магнитная стрелка маленький продолговатый магнит с двумя полюсами на концах южным S и северным N. 1Однородное магнитное поле оказывает на рамку, как показывает опыт, лишь ориентирующее действие. В неоднородном магнитном поле рамка, кроме того, будет двигаться поступательно, притягиваясь к проводнику с током или отталкиваясь от него. 6

10 Направление вектора магнитной индукции. Ориентирующее действие магнитного поля на магнитную стрелку или рамку с током можно использовать для определения направления вектора магнитной индукции. За направление вектора магнитной индукции принимается направление, которое показывает северный полюс N магнитной стрелки, свободно устанавливающейся в магнитном поле (рис. 1.7, а). Это направление совпадает с направлением положительной нормали к замкнутому контуру с током (рис. 1.7, б). Положительная нормаль направлена в ту сторону, куда перемещается буравчик (с правой нарезкой), если вращать его по направлению тока в рамке (рис. 1.7, в). Используя рамку с током или магнитную стрелку, можно определить направление вектора магнитной индукции в любой точке поля. На рисунках 1.8, 1.9 показаны опыты с магнитной стрелкой, повторяющие опыты с рамкой (см. рис. 1.5, 1.6). В магнитном поле прямолинейного проводника с током магнитная стрелка в каждой точке устанавливается по касательной к окружности (см. рис. 1.9). Плоскость такой окружности перпендикулярна проводу, а центр ее лежит на оси провода. Направление вектора магнитной индукции устанавливают с помощью правила буравчика: если направление поступательного движения буравчика совпадает с направлением тока в проводнике, то направление вращения ручки буравчика указывает направление вектора магнитной индукции. Опыт по определению направления вектора индукции магнитного Рис. 1.7 Рис. 1.8 в) f at

11 В Рис Рис. 1.9 Рис поля Земли проводит каждый, кто ориентируется на местности по компасу. Линии магнитной индукции. Наглядную картину магнитного поля можно получить, если построить так называемые линии магнитной индукции. Линиями магнитной индукции называют линии, касательные к которым в любой V их точке совпадают с вектором В в данной точке поля (рис. 1.10). Линии вектора магнитной индукции аналогичны линиям вектора напряженности электростатического поля. Для магнитного поля прямолинейного проводника с током из приведенных ранее опытов следует, что линии магнитной индукции концентрические окружности, лежащие в плоскости, перпендикулярной этому проводнику с током (см. рис. 1.9). Центр окружностей находится на оси проводника. Стрелки на линиях указывают, в какую сторону направлен вектор магнитной индукции, касательный к данной линии. Магнитное поле Земли На рисунке 1.11 показана картина магнитного поля катушки с током (соленоида). Если длина соленоида много больше его диаметра, то магнитное поле внутри соленоида можно считать однородным. Линии магнитной индукции такого поля параллельны и находятся на равных расстояниях друг от друга. На рисунке 1.12 показано магнитное поле Земли. Линии магнитной индукции поля Земли подобны линиям Рис магнит 8

12 и Рис Рис ной индукции поля соленоида. Магнитный северный полюс N близок к Южному географическому полюсу, а магнитный южный полюс S к Северному географическому полюсу. Ось такого большого магнита составляет с осью вращения Земли угол 11,5. Периодически магнитные полюсы меняют свою полярность. Последняя такая замена произошла около 30 ООО лет назад. Картину линий магнитной индукции можно сделать видимой, воспользовавшись мелкими железными опилками. С этим методом вы уже знакомы. В магнитном поле каждый кусочек железа, насыпанный на лист картона, намагничивается и ведет себя как маленькая магнитная стрелка. Большое количество таких стрелок позволяет в большем числе точек определить направление магнитного поля и, следовательно, точнее выяснить расположение линий магнитной индукции. Примеры картин магнитного поля приведены на рисунках Вихревое поле. Важная особенность линий магнитной индукции состоит в том, что они не имеют ни начала, ни конца. Они всегда замкнуты. Вспомним, что с электростатическим полем дело обстоит иначе. Его силовые линии во всех случаях имеют источники: они начинаются на положительных зарядах и оканчиваются на отрицательных. Поля с замкнутыми векторными линиями называют вихревыми. Магнитное поле вихревое поле. Замкнутость линий магнитной индукции представляет собой фундаментальное свойство магнитного поля. Оно заш ш ш ш W г I I VA i.., -., УГ,Ч ‘.I I.1 \ Д Рис

13 ключается в том, что магнитное поле не имеет источников. Магнитных зарядов, подобных электрическим, в природе не существует. Магнитное поле вихревое поле, в каждой точке поля вектор магнитной индукции имеет определенное направление. Это направление указывает магнитная стрелка или его можно определить по правилу буравчика. Магнитное поле не имеет источников; магнитных зарядов в природе не существует. 1. Как ориентирую тся в однородном магнитном поле замкнутый контур с током и магнитная стрелка! * 2. Что называют линиями магнитной индукции! 3. Какие поля называют вихревыми! 4. Чем вихревое поле отличается от потенциального! МОДУЛЬ ВЕКТОРА МАГНИТНОЙ ИНДУКЦИИ. 3 СИЛА АМПЕРА Магнитное поле действует на все участки проводника с током. Зная силу, действующую на каждый малый участок проводника, можно вычислить силу, действующую на весь замкнутый проводник в целом. Закон, определяющий силу, действующую на отдельный небольшой участок проводника (элемент тока), был установлен в 1820 г. А. А м п е р о м1. Так как создать обособленный элемент тока нельзя, то Ампер проводил опыты с замкнутыми проводниками. Меняя форму проводников и их расположение, он сумел установить выражение для силы, действующей на отдельный элемент тока. Модуль вектора магнитной индукции. Выясним экспериментально, от чего зависит сила, действующая на проводник с током в магнитном поле. Это позволит нам дать определение модуля вектора магнитной индукции, а затем найти силу Ампера. Действие магнитного поля на проводник с током будем изучать на установке, изображенной на рисунке Свободно подвешенный горизонтально проводник находится в поле постоянного подковообразного магнита. Поле маг- 1 Точнее говоря, Ампер установил закон для силы взаимодействия между двумя небольшими участками (элементами) проводников с током. Он был сторонником теории дальнодействия и не пользовался понятием поля. Однако по традиции и в память о заслугах этого ученого выражение для магнитной силы, действующей на проводник с током со стороны магнитного поля, также называют законом Ампера. 10

14 А м п е р А н д р е М ар и ( ) великий ф ранцузский ф изик и м атем ати к, один из основоположников электродинамики. Ввел в ф и зику понятие «электрический ток» и разработал первую теорию м агнетизм а, основанную на гипотезе м олекулярны х токов, откры л механическое взаимодействие электрических токов и установил количественные соотнош ения для силы этого взаимодействия. Назван М аксвеллом «Ньютон электричества». Работал такж е в области механики, теории вероятностей и м атем атического анализа. нита сосредоточено в основном между его полюсами, поэтому магнитная сила действует практически только на часть проводника длиной АI, расположенную непосредственно между полюсами. Сила F измеряется с помощью специальных весов, связанных с проводником двумя стерженьками. Она направлена горизонтально, перпендикулярно проводнику и линиям магнитной индукции. Увеличивая силу тока в 2 раза, можно заметить, что и действующая на проводник сила также увеличивается в 2 раза. Добавив еще один такой же магнит, мы в 2 раза увеличим размеры области, где существует магнитное поле, и тем самым в 2 раза увеличим длину части проводника, на которую действует магнитное поле. Сила при этом также увеличится в 2 раза. И наконец, сила Ампера зависит от угла, образованного вектором В с проводником. Рис

15 В этом можно убедиться, меняя наклон подставки, на которой находятся магниты, так, чтобы изменялся угол между проводником и линиями магнитной индукции. Сила достигает максимального значения Fm, когда вектор магнитной индукции перпендикулярен проводнику. Итак, максимальная сила, действующая на отрезок проводника длиной АI, по которому идет ток, прямо пропорциональна произведению силы тока I на длину участка Al: Fт~ I АI. Этот опытный факт можно использовать для определения модуля вектора магнитной индукции. В самом деле, F поскольку Fm~ IAI, то отношение не будет зависеть ни от силы тока в проводнике, ни от длины участка проводника. Именно поэтому это отношение можно принять за характеристику магнитного поля в том месте, где расположен участок проводника длиной АI. Модуль вектора магнитной индукции определяется отношением максимальной силы, действующей со стороны магнитного поля на отрезок проводника с током, к произведению силы тока на длину этого отрезка: B =J71- IAI (1-D Магнитное поле полностью характеризуется вектором магнитной индукции В. В каждой точке магнитного поля можно определить направление вектора магнитной индукции и его модуль, если измерить силу, действующую на отрезок проводника с током. со стороны магнитного поля с индукцией В, составляющей с элементом тока угол а, имеет вид F I \В Д/ sin а. (1.2) Это выражение называют законом Ампера. Сила Ам пера равна произведению модуля силы тока, вектора магнитной индукции, длины от резка п роводника и синуса угла между направлениями векторов магнитной индукции и тока. Направление силы Ампера. В рассмотренном выше опыте вектор F перпендикулярен элементу тока и вектору В. Его направление определяется правилом левой руки: если левую руку расположить так, чтобы перпендикулярная У проводнику составляющая вектора магнитной индукции В входила в ладонь, а четыре вытянутых пальца были направлены по направлению тока, то отогнутый на 90 большой палец укажет направление силы, действующей на отрезок проводника (рис. 1.19). Это правило справедливо во всех случаях. Единица магнитной индукции. Мы ввели новую величину вектор магнитной индукции. За единицу модуля вектора магнитной индукции можно принять магнитную индукцию однородного поля, в котором на отрезок проводника длиной 1 м при силе тока в нем 1 А действует со стороны поля максимальная сила Fm 1 Н. Согласно формуле (1.1) единица магнитной индукции равна А м Единица магнитной индукции получила название тесла (Тл) в честь югославского ученого-электротехника Н. Тесла ( ). 13

17 Измеряя силу, действующую со стороны магнитного поля на участок проводника с током, можно определить модуль вектора магнитной индукции. Сформулирован закон Ампера для силы, действующей на участок проводника с током в магнитном поле. 1. Как определяется модуль вектора магнитной индукции! 2. Чему равен модуль вектора силы Ампера! * 3. Сф ормулируйте правило для определения направления силы Ампера. 4. В каких единицах измеряется магнитная индукция! 4 ЭЛЕКТРОИЗМЕРИТЕЛЬНЫЕ ПРИБОРЫ Рис Ориентирующее действие магнитного поля на контур с током (см. 2) используют в электроизмерительных приборах магнитоэлектрической системы амперметрах и вольтметрах. Измерительный прибор магнитоэлектрической системы устроен следующим образом (рис. 1.20). На легкую алюминиевую рамку 2 прямоугольной формы с прикрепленной к ней стрелкой 4 намотана катушка. Рамка укреплена на двух полуосях ОО’. В положении равновесия ее удерживают две тонкие спиральные пружины 3. Силы упругости со стороны пружин, возвращающие катушку в положение равновесия, пропорциональны углу отклонения стрелки от положения равновесия. Катушку помещают между полюсами постоянного магнита М с наконечиками специальной формы. Внутри катушки расположен цилиндр 1 из железа. Такая конструкция обеспечивает радиальное направление линий магнитной индукции в той области, где находятся витки катушки (рис. 1.21). В результате при любом положении катушки силы, действующие на нее со стороны магнитного поля, максимальны и при неизменной силе тока постоянны. Векторы F и F изображают силы, действующие на катушку со стороны магнитного поля и поворачивающие ее. Катушка с током поворачивается до тех пор, пока силы упругости со

18 стороны пружин не уравновесят силы, действующие на рамку со стороны магнитного поля. Увеличивая силу тока в 2 раза, мы обнаружим, что стрелка поворачивается на угол, вдвое больший, и т. д. Это происходит потому, что силы, действующие на катушку со стороны магнитного поля, прямо пропорциональны силе тока: Рис Fm~ I. Благодаря этому можно определить силу тока по углу поворота катушки, если проградуировать прибор. Для этого надо установить, каким углам поворота стрелки соответствуют известные значения силы тока. Такой же прибор может измерять и напряжение. Для этого нужно градуировать прибор так, чтобы угол поворота стрелки соответствовал определенным значениям напряжения. Кроме того, сопротивление вольтметра должно быть много больше сопротивления амперметра. В основе устройства электроизмерительных приборов магнитоэлектрической системы лежит действие магнитного поля на рамку с током. ~ 1. Почему магнитные силы, действующ ие на проводники катушки 9 прибора, не зависят от угла поворота катушки? 2. Что удерживает рамку от вращения в магнитном поле! 3. Чем ам перм етр отличается от вольтметра! ПРИМЕНЕНИЕ ЗАКОНА АМПЕРА. 5 ГРОМКОГОВОРИТЕЛЬ Зная направление и модуль силы, действующей на любой участок проводника с током, можно вычислить суммарную силу, действующую на весь замкнутый проводник. Для этого надо найти сумму сил, действующих на каждый участок проводника с током. Закон Ампера используют для расчета сил, действующих на проводники с током, во многих технических устройствах. В частности в электродвигателях, с которыми вы ознакомились в предыдущих классах. Разберем устройство громкоговорителя. Громкоговоритель служит для возбуждения звуковых волн под действием переменного электрического тока, ме- 15

19 По катушке проходит переменный электрический ток с частотой, равной звуковой частоте сигнала с микрофона или с выхода радиоприемника, проигрывателя, магнитофона. Под действием силы Ампера катушка колеблется вдоль оси громкоговорителя ООх (см. рис. 1.22, а) в такт с колебаниями тока. Эти колебания передаются диафрагме, и поверхность диафрагмы излучает звуковые волны. Первоклассные громкоговорители воспроизводят без значительных искажений звуковые колебания в диапазоне ООО Гц. Но такие устройства очень сложны. Поэтому обычно применяют системы из нескольких громкоговорителей, каждый из которых воспроизводит звук в определенном небольшом интервале частот. Общим недостатком всех громкоговорителей является их малый КПД. Они излучают лишь 1 3% подводимой энергии. Звук в радиоприемнике, проигрывателе и магнитофоне возникает в результате движения катушки с током в поле постоянного магнита. Наряду с электромеханическими громкоговорителями в настоящее время широкое применение получили громкоговорители, основанные на пьезоэлектрическом эффекте. Этот эффект проявляется в виде деформации некоторых типов кристаллов в электростатическом поле. Две пьезопластинки склеивают. Пластинки подбирают так, что одна из них увеличивается по длине под действием поля, а другая уменьшается (см. рис. 1.22, б). В результате получают элемент, который сильно изгибается под действием поля и при переменном электрическом поле создает акустическую волну. Пьезогромкоговорители очень удобны в изгозк- Ё ~ Рис ю а) Звуковая волна М ~ ->. -si Г Т б) няющегося со звуковой частотой. В электродинамическом громкоговорителе (динамике) используется действие магнитного поля постоянного магнита на переменный ток в подвижной катушке. Схема устройства громкоговорителя показана на рисунке 1.22, а. Звуковая катушка ЗК располагается в зазоре кольцевого магнита М. С катушкой жестко связан бумажный конус диафрагма D. Диафрагма укреплена на упругих подвесах, позволяющих ей совершать вынужденные колебания вместе с подвижной катушкой.

20 товлении и могут быть совсем маленькими. Вследствие этого они нашли широкое применение в радиотелефонах, мобильных телефонах, ноутбуках и микрокомпьютерах. Взаимодействие токов и пьезоэлектрический эффект положены в основу принципа работы современных громкоговорителей.? Укажите направление вектора магнитной индукции, электрического тока и силы Ампера на схем е гром коговорителя (см. рис. 1.22). ДЕЙСТВИЕ МАГНИТНОГО ПОЛЯ 6 НА ДВИЖУЩИЙСЯ ЗАРЯД. СИЛА ЛОРЕНЦА Электрический ток это упорядоченно движущиеся заряженные частицы. Поэтому действие магнитного поля на проводник с током есть результат действия поля на движущиеся заряженные частицы внутри проводника. Наймем силу, действующую на одну частицу. Силу, действующую на движущуюся заряженную частицу со стороны магнитного поля, называют силой Лоренца в честь великого голландского физика X. Лоренца ( ) основателя электронной теории строения вещества. Силу Лоренца можно найти с помощью закона Ампера. Модуль силы Лоренца равен отношению модуля силы F, действующей на участок проводника длиной AZ, к числу N заряженных частиц, упорядоченно движущихся в этом участке проводника: Рассмотрим отрезок тонкого прямого проводника с током (рис. 1.23). Пусть длина отрезка АI и площадь поперечного сечения проводника S настолько малы, что вектор индукции магнитного поля В можно считать одинаковым в пределах этого отрезка проводника. Сила тока I в проводнике связана с зарядом частиц q, концентрацией заряженных частиц (числом зарядов в единице объема) и скоростью их упорядоченного движения v следующей формулой: I — qnvs. (1-4) Рис

21 Модуль силы, действующей со стороны магнитного поля на выбранный элемент тока, равен: F = \I\BAI sin а. Подставляя в эту формулу выражение (1.4) для силы тока, получаем:.f = g nvsalb sin а = v q NB sin а, где N = nsal число заряженных частиц в рассматриваемом объеме. Следовательно, на каждый движущийся заряд со стороны магнитного поля действует сила Лоренца, равная: = = g vb sin а, (1.5) где а угол между вектором скорости и вектором магнитной индукции. Сила Лоренца перпендикулярна векторам В и и. Ее направление определяется с помощью того же правила левой руки, что и направление силы Ампера: если левую руку расположить так, чтобы составляющая магнитной индукции В, перпендикулярная скорости заряда, входила в ладонь, а четыре вытянутых пальца были направлены по движению положительного заряда (против движения отрицательного), то отогнутый на 90 большой палец укажет направление действующей на заряд силы Лоренца Fn (рис. 1.24). Электрическое поле действует на заряд q с силой F3J[ = qe. Следовательно, если есть и электрическое поле, и магнитное поле, то суммарная сила F, действующая на заряд, равна: 18 F = Рэл + Fn- Так как сила Лоренца перпендикулярна скорости частицы, то она не совершает работы. магнитном поле В, направленном перпендикулярно к начальной скорости частицы и (рис. 1.25). Сила Лоренца зависит от модулей векторов скорости частицы и индукции магнитного поля. Так как магнитное поле не меняет модуль скорости движущейся частицы, то остается неизменным и модуль силы Лоренца. Эта сила перпендикулярна скорости и, следовательно, определяет центростремительное ускорение частицы. Неизменность по модулю центростремительного ускорения частицы, движущейся с постоянной по модулю скоростью, означает, что частица равномерно движется по окружности радиусом г. Определим этот радиус. Согласно второму закону Ньютона (см. рис. 1.25) Отсюда = \q\vb. mv Ы в ‘ ( 1.6) Время, за которое частица делает полный оборот (период обращения), равно: т = v =ГТ5- \q\b d — 7> Использование действия магнитного поля на движущийся заряд. Действие магнитного поля на движущийся заряд широко используют в современной технике. Достаточно упомянуть телевизионные трубки (кинескопы), в которых летящие к экрану электроны отклоняются с помощью магнитного поля, создаваемого особыми катушками. Сила Лоренца используется в ускорителе заряженных частиц (циклотрон) для получения частиц с большими энергиями. Циклотрон состоит из двух полых полуцилиндров (дуантов) 3, находящихся в однородном магнитном поле (рис. 1.26). Между дуантами создается переменное 19

23 Батарея, создающая электрическое поле. Согласно ускоряющее напряжение формуле (1.6) при увеличении скорости частицы 1 радиус окружности (траектории 2), по которой движется частица, увеличивается. Период обращения частицы не зависит от скорости (см. формулу (1.7)), и, следовательно, через полпериода, вследствие изменения направления электрического поля, частица снова оказывается в ускоряющем ее поле и т. д. На последнем витке частица вылетает из циклотрона. На действии магнитного поля основано также и устройство приборов, позволяющих разделять заряженные частицы по их удельным зарядам, т. е. по отношению заряда частицы к ее массе, и по полученным результатам точно определять массы частиц. Такие приборы получили название масс-спектрографов. На рисунке 1.27 изображена принципиальная схема простейшего масс-спектрографа. Вакуумная камера прибора помещена в магнитное поле (вектор индукции В перпендикулярен рисунку). Ускоренные электрическим полем заряженные частицы (электроны или ионы), описав дугу, попадают на фотопластинку, где оставляют след, позволяющий с высокой точностью измерить радиус траектории г. По этому радиусу определяется удельный заряд иона. Зная заряд иона, легко вычислить его массу. На движущуюся заряженную частицу со стороны магнитного поля действует сила Лоренца. Эта сила перпендикулярна скорости и не совершает работы. У ф 1. Чему равен модуль силы Лоренца! 2. Как движется заряженная частица в однородном магнитном поле, если начальная скорость частицы перпендикулярна линиям магнитной индукции! 3. Как определить направление силы Лоренца! 7 МАГНИТНЫЕ СВОЙСТВА ВЕЩЕСТВА Магнитное поле создается не только электрическими токами, но и постоянными магнитами. Намагничивание вещества. Постоянные магниты могут быть изготовлены лишь из сравнительно немногих веществ, 20

24 но все вещества, помещенные в магнитное поле, намагничиваются, т. е. сами становятся источниками магнитного поля. В результате этого вектор магнитной индукции при наличии вещества отличается от вектора магнитной индукции в вакууме. Гипотеза Ампера. Причина, вследствие которой тела обладают магнитными свойствами, была установлена французским ученым Ампером. Сначала, под непосредственным впечатлением от наблюдения за поворачивающейся вблизи проводника с током магнитной стрелкой в опытах Эрстеда Ампер предположил, что магнетизм Земли вызван токами, проходящими внутри земного шара. Главный шаг был сделан: магнитные свойства тела можно объяснить циркулирующими внутри него токами. Далее Ампер пришел к общему заключению: магнитные свойства любого тела определяются замкнутыми электрическими токами внутри него. Этот решающий шаг от возможности объяснения магнитных свойств тела токами к категорическому утверждению, что магнитные взаимодействия это взаимодействия токов, свидетельство большой научной смелости Ампера. Согласно гипотезе Ампера внутри молекул и атомов циркулируют элементарные электрические токи. (Теперь мы хорошо знаем, что эти токи образуются вследствие движения электронов в атомах.) Если плоскости, в которых циркулируют эти токи, расположены беспорядочно по отношению друг к другу из-за теплового движения молекул (рис. 1.28, а), то их действия взаимно компенсируются, и никаких магнитных свойств тело не обнаруживает. В намагниченном состоянии элементарные токи в теле ориентированы так, что их действия складываются (рис. 1.28, б). Гипотеза Ампера объясняет, почему магнитная стрелка и рамка (контур) с током в магнитном поле ведут себя одинаково (см. 2). Стрелку можно рассматривать как совокупность маленьких контуров с током, ориентированных одинаково. Наиболее сильные магнитные поля создают вещества, называемые ферромагнетиками. Магнитные поля создаются ферромагнетиками не только вследствие обращения электронов вокруг ядер, но и вследствие их собственного Рис а) б) 21

25 вращения. Собственный вращательный момент (момент импульса) электрона называется спином. Электроны всегда как бы вращаются вокруг своей оси и, обладая зарядом, создают магнитное поле наряду с полем, появляющимся за счет их орбитального движения вокруг ядер. В ферромагнетиках существуют области с параллельными ориентациями спинов, называемые доменами; размеры доменов порядка 0,5 мкм. Параллельная ориентация спинов обеспечивает минимум потенциальной энергии. Если ферромагнетик не намагничен, то ориентация доменов хаотична, и суммарное магнитное поле, создаваемое доменами, равно нулю. При включении внешнего магнитного поля домены ориентируются вдоль линий магнитной индукции этого поля, и индукция магнитного поля в ферромагнетиках увеличивается, становясь в тысячи и даже миллионы раз больше индукции внешнего поля. Температура Кюри. При температурах, больших некоторой определенной для данного ферромагнетика, его ферромагнитные свойства исчезают. Эту температуру называют температурой Кюри по имени открывшего данное явление французского ученого. Если достаточно сильно нагреть намагниченный гвоздь, то он потеряет способность притягивать к себе железные предметы. Температура Кюри для железа 753 С, для никеля 365 С, а для кобальта 1000 С. Существуют ферромагнитные сплавы, у которых температура Кюри меньше 100 С. Первые детальные исследования магнитных свойств ферромагнетиков были выполнены выдающимся русским физиком А. Г. Столетовым ( ). Ферромагнетики и их применение. Хотя ферромагнитных тел в природе не так уж много, именно их магнитные свойства получили наибольшее практическое применение. Железный или стальной сердечник в катушке во много раз усиливает создаваемое ею магнитное поле, не увеличивая силу тока в катушке. Это экономит электроэнергию. Сердечники трансформаторов, генераторов, электродвигателей и т. д. изготовляют из ферромагнетиков. При выключении внешнего магнитного поля ферромагнетик остается намагниченным, т. е. создает магнитное поле в окружающем пространстве. Это объясняется тем, что домены не возвращаются в прежнее положение и их ориентация частично сохраняется. Благодаря этому существуют постоянные магниты. Постоянные магниты находят широкое применение в электроизмерительных приборах, громкоговорителях и телефонах, звукозаписывающих аппаратах, магнитных компасах и т. д. 22

26 Большое применение получили ферриты ферромагнитные материалы, не проводящие электрического тока. Они представляют собой химические соединения оксидов железа с оксидами других веществ. Один из известных ферромагнитных материалов магнитный железняк является ферритом. Магнитная запись информации. Из ферромагнетиков изготовляют магнитные ленты и тонкие магнитные пленки. Магнитные ленты широко используют для звукозаписи в магнитофонах и для видеозаписи в видеомагнитофонах. Магнитная лента представляет собой гибкую основу из полихлорвинила или других веществ. На нее наносится рабочий слой в виде магнитного лака, состоящего из очень мелких игольчатых частиц железа или другого ферромагнетика и связующих веществ. Запись звука производят на ленту с помощью электромагнита, магнитное поле которого изменяется в такт со звуковыми колебаниями. При движении ленты вблизи магнитной головки различные участки пленки намагничиваются. Схема магнитной индукционной головки показана на рисунка 1.29, а, где 1 сердечник электромагнита; 2 магнитная лента; 3 рабочий зазор; 4 обмотка электромагнита. При воспроизведении звука наблюдается обратный процесс: намагниченная лента возбуждает в магнитной головке электрические сигналы, которые после усиления поступают на динамик магнитофона. Тонкие магнитные пленки состоят из слоя ферромагнитного материала толщиной от 0,03 до 10 мкм. Их применяют в запоминающих устройствах элек- Щель для Электромагнитная головка Рис

27 тронно-вычислительных машин (ЭВМ). Магнитные пленки предназначены для записи, хранения и воспроизведения информации. Их наносят на тонкий алюминиевый диск или барабан. Информацию записывают и воспроизводят примерно так же, как и в обычном магнитофоне. Запись информации в ЭВМ можно производить и на магнитные ленты. Развитие технологии магнитной записи привело к появлению магнитных микроголовок, которые используются в ЭВМ, позволяющих создавать немыслимую ранее плотность магнитной записи. На ферромагнитном жестком диске диаметром меньше 8 см хранится до нескольких терабайт (1012 байт) информации. Считывание и запись информации на таком диске осуществляется с помощью микроголовки, расположенной на поворотном рычаге (рис. 1.29, б). Сам диск вращается с огромной скоростью, и головка плавает над ним в потоке воздуха, что предотвращает возможность механического повреждения диска. Все вещества, помещенные в магнитное поле, создают собственное поле. Наиболее сильные поля создают ферромагнетики. Из них делают постоянные магниты, так как поле ферромагнетика не исчезает после выключения намагничивающего поля. Ферромагнетики широко применяются на практике. 1. Какие вещества называют ф ерромагнетиками! 2. Для каких целей применяют ф ерромагнитны е материалы! 3. Как осущ ествляется запись информации в ЭВМ! ПРИМЕРЫ РЕШЕНИЯ ЗАДАЧ Решение задач с применением закона Ампера (и использованием выражения для силы Лоренца) аналогично решению задач механики. Кроме механических сил, надо учитывать силу Ампера (или силу Лоренца) и правильно определять ее направление. 1. Между полюсами магнита подвешен горизонтально на двух невесомых нитях прямой проводник длиной I = 0,2 м и массой т= 10 г. Вектор индукции однородного магнитного поля перпендикулярен проводнику и направлен вертикально; Б = 49 мтл. На какой угол а от вертикали отклонятся нити, поддерживающие проводник, если по нему пропустить ток? Сила тока I = 2 А. Решение. На проводник действуют следующие силы: силы упругости двух нитей Т, сила тяжести mg и сила Ам- 24

28 ee a у / 4. чу) I т Е кв 1″ В V О mg О X *z ‘ X Рис Рис пера F (рис. 1.30). Модуль силы Ампера F = IBI. При равновесии проводника суммы проекций сил на вертикальное и горизонтальное направления (с учетом их знаков) равны нулю: Отсюда -m g + Tcos а = 0, -F + Tsin а = 0. tga = = ~ 0,2. mg mg Следовательно, угол а = 11,3. 2. В пространстве, где созданы одновременно однородные и постоянные электрическое и магнитное поля, по прямолинейной траектории движется протон. Известно, что напряженность ^ электрического поля равна Е. Определите индукцию В магнитного поля. Решение. Прямолинейное движение протона возможно в двух случаях. 1) Вектор Е направлен вдоль траектории движения > протона. Тогда вектор В также должен быть направлен вдоль этой траектории, и его модуль может быть любым, так как магнитное поле не будет действовать на частицу. 2) Векторы Е, В и и взаимно перпендикулярны, и сила, действующая на протон со стороны электрического поля, равна по модулю и противоположна по направлению силе Лоренца, действующей на протон со стороны магнитного поля (рис. 1.31). Так как то ee + Fn 0, ее — еив 0 и В = Е v 25

29 УПРАЖНЕНИЕ 1 /, 1. Используя правило буравчика и правило левой руки, покажите, что токи, направленные параллельно, притягиваются, а направленные противоположно отталкиваются. 2. По двум скрещивающимся под прямым углом прямолинейным проводникам пропускают токи. Силы токов 1Хи 12 (рис. 1.32). Как будет изменяться расположение проводников относительно друг друга? 3. Проводник длиной I — 0,15 м перпендикулярен вектору магнитной индукции однородного магнитного поля, модуль которого В = 0,4 Тл. Сила тока в проводнике I = 8 А. Определите работу силы Ампера, которая была совершена при перемещении проводника на 0,025 м по направлению действия этой силы. 4. Определите радиус окружности и период обращения электрона в однородном магнитном поле с индукцией В =0,01 Тл. Скорость электрона перпендикулярна вектору магнитной индукции и равна 106 м/с. КРАТКИЕ ИТОГИ ГЛАВЫ 1 1. Взаимодействие электрических токов осуществляется посредством магнитного поля. Основной характеристикой магнитного поля является вектор магнитной индукции В. Модуль вектора магнитной индукции определяется отношением максимальной силы, действующей на отрезок проводника с током со стороны магнитного поля, к произведению силы тока на длину этого отрезка. 2. Линии магнитной индукции всегда замкнуты. Магнитное поле является вихревым. 3. Согласно закону Ампера на отрезок проводника с током длиной А1 со стороны магнитного поля действует сила, модуль которой равен F I \B\l sin а, где а угол между направлением тока и вектором В; I сила тока в проводнике. Направление силы определяется по правилу левой руки. 4. На движущуюся заряженную частицу в магнитном поле действует сила Лоренца, модуль которой равен 26

30 Fn = q vb sin а, где a угол между скоростью части- цы и вектором В. Сила Лоренца перпендикулярна скорости частицы и вектору В. 5. Все тела в магнитном поле намагничиваются, т. е. сами создают магнитное поле. У большинства веществ магнитные свойства выражены довольно слабо. Лишь в ферромагнетиках, к которым относится железо, индукция магнитного поля существенно увеличивается. Хотя ферромагнетиков сравнительно немного, но они имеют очень большое практическое значение, так как позволяют во много раз увеличивать индукцию магнитного поля без затрат энергии. Глава 2. ЭЛЕКТРОМАГНИТНАЯ ИНДУКЦИЯ До сих пор мы рассматривали электрические и магнитные поля, не изменяющиеся с течением времени. Было выяснено, что электростатическое поле создается неподвижными заряженными частицами, а магнитное поле движущимися, т. е. электрическим током. Теперь познакомимся с электрическими и магнитными полями, которые меняются со временем. Самый важный факт, который удалось обнаружить, это теснейшая взаимосвязь между электрическим и магнитным полями. Оказалось, что изменяющееся во времени магнитное поле порождает электрическое поле, а изменяющееся электрическое поле магнитное. Без этой связи между полями разнообразие проявлений электромагнитных сил не было бы столь обширным, каким оно наблюдается на самом деле. Не существовало бы ни радиоволн, ни света. ОТКРЫТИЕ ЭЛЕКТРОМАГНИТНОЙ 8 ИНДУКЦИИ В 1821 г. М. Фарадей записал в своем дневнике: «Превратить магнетизм в электричество». Через 10 лет эта задача была им решена. Не случайно первый, решающий шаг в открытии новых свойств электромагнитных взаимодействий сделан основоположником представлений об электромагнитном поле М. Фарадеем, который был уверен в единой природе электрических и магнитных явлений. Благодаря этому он и сделал открытие, вошедшее в основу устройства генерато- 27

31 Рис. 2.1 а) б) ров всех электростанций мира, превращающих механическую энергию в энергию электрического тока. (Источники, работающие на других принципах: гальванические элементы, аккумуляторы и пр., дают ничтожную долю вырабатываемой электрической энергии.) Электрический ток, рассуждал М. Фарадей, способен намагнитить кусок железа. Не может ли магнит, в свою очередь, вызвать появление электрического тока? Долгое время эту связь обнаружить не удавалось. Трудно было додуматься до главного, а именно: движущийся магнит, или меняющееся во времени магнитное поле, может возбудить электрический ток в катушке. Какого рода случайности могли помешать открытию, показывает следующий факт. Почти одновременно с Фарадеем получить электрический ток в катушке с помощью магнита пытался швейцарский физик Колладон. В ходе работы он пользовался гальванометром, легкая магнитная стрелка которого помещалась внутри катушки прибора. Чтобы магнит не оказывал непосредственного влияния на стрелку, концы катушки, куда Колладон вводил магнит, надеясь получить в ней ток, были выведены в соседнюю комнату и там присоединены к гальванометру. Вставив магнит в катушку, Колладон шел в соседнюю комнату и с огорчением убеждался, что гальванометр не показывает тока. Стоило бы ему все время наблюдать за гальванометром, а кого-нибудь попросить заняться магнитом, замечательное открытие было бы сделано. Но этого не случилось. Покоящийся относительно катушки магнит не вызывает в ней тока. Явление электромагнитной индукции заключается в возникновении электрического тока в проводящем контуре, который либо покоится в переменном во времени магнитном поле, либо движется в постоянном магнитном поле таким образом, что число линий магнитной индукции, пронизывающих контур, меняется. Это явление было открыто 28

32 29 августа 1831 г. Редкий случай, когда дата нового замечательного открытия известна так точно! Сначала Фарадеем была открыта электромагнитная индукция в неподвижных относительно друг друга проводниках при замыкании и размыкании цепи. Затем, ясно понимая, что сближение или удаление проводников с током должно приводить к тому же результату, что и замыкание и размыкание цепи, Фарадей с помощью опытов доказал: ток возникает при перемещении катушек относительно друг друга. Знакомый с трудами Ампера, Фарадей понимал, что магнит это совокупность маленьких токов, циркулирующих в молекулах. 17 октября, как зарегистрировано в его лабораторном журнале, был обнаружен индукционный ток в катушке во время введения (или удаления) магнита. В течение одного месяца Фарадей опытным путем установил все главные особенности явления электромагнитной индукции. В настоящее время опыты Фарадея может повторить каждый. Для этого надо иметь две катушки, магнит, батарею элементов и достаточно чувствительный гальванометр. В установке, изображенной на рисунке 2.1, а, индукционный ток возникает в одной из катушек в момент замыкания или размыкания электрической цепи другой катушки, неподвижной относительно первой. В других опытах индукционный ток возникает при изменении силы тока в одной из катушек с помощью реостата (рис. 2.1, б), при движении катушек относительно друг друга (рис. 2.2, а), при движении постоянного магнита относительно катушки (рис. 2.2, б). Уже сам Фарадей заметил то общее, от чего зависит появление индукционного тока в опытах, которые поставлены по-разному. В замкнутом проводящем контуре возникает ток при изменении числа линий магнитной индукции, пронизывающих поверхность, ограниченную этим контуром. 29

33 И чем быстрее меняется число линий магнитной индукции, тем больше возникающий индукционный ток. При этом причина изменения числа линий магнитной индукции не существенна. Это может быть и изменение числа линий магнитной индукции, пронизывающих поверхность, ограниченную неподвижным проводящим контуром, вследствие изменения силы тока в соседней катушке (см. рис. 2.1, б), и изменение числа линий индукции вследствие движения контура в неоднородном магнитном поле, густота линий которого меняется в пространстве (рис. 2.3). В проводящем замкнутом контуре возникает электрический ток, если контур находится в переменном магнитном поле или движется в постоянном во времени магнитном поле так, что число линий магнитной индукции, пронизывающих контур, меняется.? 1. В чем главное отличие переменных электрических и магнитных полей от постоянных! 2. В чем заключается явление электромагнитной индукции! 3. Как должен двигаться замкнутый проводящий контур в однородном магнитном поле, не зависящем от времени: поступательно или вращательно, чтобы в нем возник индукционный ток! 9 МАГНИТНЫЙ ПОТОК Для того чтобы дать точную количественную формулировку закона электромагнитной индукции Фарадея, нужно ввести новую величину поток вектора магнитной индукции. Вектор магнитной индукции В характеризует магнитное поле в каждой точке пространства. Можно ввести еще одну величину, зависящую от значений вектора Б не в одной точке, а во всех точках поверхности, ограниченной плоским замкнутым контуром. Для этого рассмотрим плоский замкнутый проводник (контур), ограничивающий поверхность площадью S и помещенный в однородное магнитное поле (рис. 2.4). Нормаль п (вектор, модуль которого равен единице) к плоскости проводника составляет угол а с направлением вектора магнитной индукции В. Магнитным потоком Ф (потоком вектора магнитной индукции) через поверхность площадью S называют величину, равную произведению модуля 30

34 вектора магнитной индукции В на площадь S и косинус угла а между векторами В и п: <t>= B S cos а. (2.1) Произведение В cos а — В п представляет собой проекцию вектора магнитной индукции на нормаль п к плоскости контура. Поэтому Рис. 2.4 Ф = B ns. (2.2) Магнитный поток тем больше, чем больше Впи S. Величина Ф названа «магнитным потоком» по аналогии с потоком воды, который тем больше, чем больше скорость течения воды и площадь сечения трубы. Магнитный поток графически можно истолковать как величину, пропорциональную числу линий магнитной индукции, пронизывающих поверхность площадью S. Единицей магнитного потока является вебер. Магнитный поток в 1 вебер (1 Вб) создается однородным магнитным полем с индукцией 1 Тл через поверхность площадью 1 м2, расположенную перпендикулярно вектору магнитной индукции. Магнитный поток зависит от ориентации поверхности, которую пронизывает магнитное поле. НАПРАВЛЕНИЕ ИНДУКЦИОННОГО ТОКА. 10 ПРАВИЛО ЛЕНЦА Присоединив катушку, в которой возникает индукционный ток, к гальванометру, можно обнаружить, что направление этого тока зависит от того, приближается ли магнит к катушке (например, северным полюсом) или удаляется от нее (см. рис. 2.2, б). Возникающий индукционный ток того или иного направления как-то взаимодействует с магнитом (притягивает или отталкивает его). Катушка с проходящим по ней током подобна магниту с двумя полюсами северным и южным. Направление индукционного тока определяет, какой конец катушки выполняет роль северного полюса (линии магнитной индукции выходят из него). На основе закона сохранения энергии можно предсказать, в каких случаях катушка будет притягивать магнит, а в каких отталкивать его. Взаимодействие индукционного тока с магнитом. Если магнит приближать к катушке, то в ней появляется индук- 31

35 ционныи ток такого направления, что магнит обязательно отталкивается. Для сближения магнита и катушки нужно совершить положительную работу. Катушка становится подобной магниту, обращенному одноименным полюсом к приближающемуся к ней магниту. Одноименные же полюса отталкиваются. При удалении магнита, наоборот, в катушке возникает ток такого направления, чтобы появилась притягивающая магнит сила. Рис. 2.5 В чем состоит различие двух опытов: приближение магнита к катушке и его удаление? В первом случае число линий магнитной индукции, пронизывающих витки катушки, или, что то же самое, магнитный поток, увеличивается (рис. 2.5, а), а во втором случае уменьшается (рис. 2.5, б). Причем в первом случае линии индукции В’ магнитного поля, созданного возникшим в катушке индукционным током, выходят из верхнего конца катушки, так как катушка отталкивает магнит, а во втором случае, наоборот, входят в этот конец. Эти линии магнитной индукции на рисунке 2.5 изображены черным цветом. В случае а катушка с током аналогична магниту, северный полюс которого находится сверху, а в случае б снизу. Аналогичные выводы можно сделать с помощью опыта, показанного на рисунке 2.6. На концах стержня, который может свободно вращаться вокруг вертикальной оси, закреплены два проводящих алюминиевых кольца. Одно из них с разрезом. Если поднести магнит к кольцу без разреза, то в нем возникнет индукционный ток и направлен он будет так, что это кольцо оттолкнется от магнита и стержень повернется. Если удалять магнит от кольца, то оно, наоборот, притянется к магниту. С разрезанным кольцом магнит не взаимодействует, так как разрез препятствует возникновению в кольце индукционного тока. Отталкивает или притягивает катушка магнит, это зависит от направления индукционного тока в ней. Поэтому закон сохранения энергии позволяет сформулировать правило, определяющее направление индукционного тока. 32

Информио

×

Неверный логин или пароль

×

Все поля являются обязательными для заполнения

×

Сервис «Комментарии» — это возможность для всех наших читателей дополнить опубликованный на сайте материал фактами или выразить свое мнение по затрагиваемой материалом теме.

Редакция Информио.ру оставляет за собой право удалить комментарий пользователя без предупреждения и объяснения причин. Однако этого, скорее всего, не произойдет, если Вы будете придерживаться следующих правил:

  1. Не стоит размещать бессодержательные сообщения, не несущие смысловой нагрузки.
  2. Не разрешается публикация комментариев, написанных полностью или частично в режиме Caps Lock (Заглавными буквами). Запрещается использование нецензурных выражений и ругательств, способных оскорбить честь и достоинство, а также национальные и религиозные чувства людей (на любом языке, в любой кодировке, в любой части сообщения — заголовке, тексте, подписи и пр.)
  3. Запрещается пропаганда употребления наркотиков и спиртных напитков. Например, обсуждать преимущества употребления того или иного вида наркотиков; утверждать, что они якобы безвредны для здоровья.
  4. Запрещается обсуждать способы изготовления, а также места и способы распространения наркотиков, оружия и взрывчатых веществ.
  5. Запрещается размещение сообщений, направленных на разжигание социальной, национальной, половой и религиозной ненависти и нетерпимости в любых формах.
  6. Запрещается размещение сообщений, прямо либо косвенно призывающих к нарушению законодательства РФ. Например: не платить налоги, не служить в армии, саботировать работу городских служб и т.д.
  7. Запрещается использование в качестве аватара фотографии эротического характера, изображения с зарегистрированным товарным знаком и фотоснимки с узнаваемым изображением известных людей. Редакция оставляет за собой право удалять аватары без предупреждения и объяснения причин.
  8. Запрещается публикация комментариев, содержащих личные оскорбления собеседника по форуму, комментатора, чье мнение приводится в статье, а также журналиста.

Претензии к качеству материалов, заголовкам, работе журналистов и СМИ в целом присылайте на адрес

×

Информация доступна только для зарегистрированных пользователей.

×

Уважаемые коллеги. Убедительная просьба быть внимательнее при оформлении заявки. На основании заполненной формы оформляется электронное свидетельство. В случае неверно указанных данных организация ответственности не несёт.

Формулировка правила Гимлета для круглого тока. Правило левой руки

Магнитное поле и его графическое изображение
Направление линии
Магнитное поле тока связано с
направлением тока в проводнике.
Правило Гимлета
, если направление
поступательного движения
буравчика совпадает с направлением тока
в проводнике
, то направление
вращения ручки буравчика
совпадает с направлением
линий магнитного поля тока.
С помощью правила буравчика
в направлении тока
идентифицируют направления линий
магнитного поля, создаваемого этим током
, а в направлении линий
магнитного поля —
направления тока, генерирующего
этого поля.

Неоднородное и однородное магнитное поле

Токовый проводник расположен

1. Направление электрического тока от нас
(в плоскости листа)
Магнитные линии
поля будут
направлены на
по часовой стрелке

Правило Гимлета

Токоведущий провод находится
перпендикулярно плоскости листа:
2.Направление электрического тока к нам
(от плоскости листа)
Магнитные линии
поля будут
направлены против
по часовой стрелке

Токопроводник перпендикулярен плоскости листа: 1. Направление электрического тока от нас (к плоскости листа) Согласно правам

Правило правой руки
Для определения
направлений магнитных линий более удобны
поля соленоида
используйте другое правило
, которое иногда называют правилом правой руки
.
, если вы возьмете соленоид
ладонью правой руки
, показывая четырьмя пальцами по
направлению тока по очереди,
, а затем отложите большой палец
, покажет направление
линий магнитного поля
внутри соленоида.

Проводник с током перпендикулярен плоскости листа: 2. Направление электрического тока на нас (из плоскости листа) По правому

Соленоид, как и магнит, имеет полюса:
на конце соленоида, из которого выходят магнитные линии
, называется северным полюсом, а входящий в
— южным.
Зная направление тока в соленоиде,
правилом правой руки можно определить
направление магнитных линий внутри него, а
, следовательно, его магнитные полюса и наоборот.
Правило правой руки может применяться к
для определения направления линий магнитного поля
в центре одного витка
с током.

Правило правой руки

для проводника

Если правая рука
устроит так, что
на большой палец
послал ток
, то остальные
четыре пальца
показывают направление
магнитных линий
индукция

1.Создается магнитное поле …


2. Что показывает изображение магнитных линий?
3. Дайте характеристику однородного магнитного поля.
Запустите рисунок.
4. Охарактеризуйте неоднородные магнитные поля
. Запустите рисунок.
5. Представьте себе однородное магнитное поле в
в зависимости от направления магнитных линий.
Объясните.
6. Объясните, как работает правило буравчика.
7. Укажите два случая зависимости направления
магнитных линий от направления электрического тока.
8. Какое правило следует использовать для
определения направления магнитных линий
соленоида. Из чего он состоит?
9. Как определить полюса соленоида?

Правило правой руки для проводника с током

Обнаружение магнитного поля
по его влиянию на электричество
.
Правило левой руки.

1. Создается магнитное поле … 2. Что показывает изображение магнитных линий? 3. Дайте характеристику однородного магнитного поля.Run dash

На каждый проводник с током,
помещенный в магнитное поле и
не совпадающий с его
магнитными линиями, это поле
действует с некоторой силой.

Обнаружение магнитного поля по его действию на электрический ток. Правило левой руки.

Выводы:
Магнитное поле создается электрическим током
и обнаруживается его действием
на электрический ток.
Направление тока в проводнике,
направление линий магнитного поля и
направление силы, действующей на
проводника.

На каждый проводник с током, помещенный в магнитное поле и не совпадающий с его магнитными линиями, это поле действует с некоторой силой.

Правило левой руки
Направление силы
, действующей на проводник с током
в магнитном поле,
можно определить с помощью правила
левой руки.
Если поставить левую руку
так, чтобы линии магнитного поля
входили в ладонь
перпендикулярно ей, а четыре пальца
были направлены по току
.Отложенный большой палец 900
покажет
направление тока
на проводнике питания.

Выводы:

Для направления тока во внешней цепи
направление взято от «+»
к «-», т.е. против направления
движения электрона в цепи

Правило левой руки

Определение силы ампер
Если положить левую руку
так, чтобы магнитный вектор
индукции шел в ладонь, а
вытянутых пальцев были
направлены по току, тогда отведенный
большой палец
указывает направление действия
Ампер силы на проводник с
электрического тока.

Направление от «+» к «-», т. Е. Против направления движения электрона в цепи

Может применяться правило левой руки
для определения направления силы, s
, магнитное поле которого действует на
отдельных движущихся
заряженных частицы.

Определение амперной силы

Сила, действующая на заряд
Если левая рука
расположить так, чтобы линии
магнитного поля входили в
ладони перпендикулярно ей,
и четыре пальца были
направленных
положительно заряженных
частиц (или против движения
отрицательно заряженных) тогда
, отведенный назад на 900 больших
пальца, покажет направление
силы частиц
Лоренца.

Правило левой руки может быть использовано для определения направления силы, с которой магнитное поле действует на отдельные движущиеся заряды

С помощью правила левой руки
можно определить направление
текущее направление магнитных
линий, знак заряд движущихся
частиц.

Сила, действующая на заряд

Случай, когда сила действия
магнитного поля на проводник при ударе электрическим током
или движущихся
заряженных частиц F = 0

Используя правило левой руки, можно определить направление тока, направление магнитных линий, знак заряда движущейся частицы.

Решите задачу:

Случай, когда сила магнитного поля на проводнике с током или движущейся заряженной частицей F = 0

Решите задачу:

Отрицательно заряженная частица
движется со скоростью v в магнитном поле
. Сделайте тот же рисунок в блокноте
и укажите стрелкой
направление силы, с которой поле
действует на частицу.
Магнитное поле действует с силой F на
частицу, движущуюся со скоростью v.
Определите знак заряда частицы.


Магнитное поле электрического тока


Магнитное поле формируется вокруг проводника с током, так что свободно вращающаяся магнитная стрелка, помещенная рядом с проводником, будет стремиться занять положение, перпендикулярное плоскости, проходящей вдоль него.
Это легко проверить, выполнив следующий эксперимент.

Магнитное поле
провод постоянного тока

Прямолинейный проводник вставляется в отверстие горизонтально уложенного листа картона, и через него пропускается ток.На картон насыпают железные опилки и следят за тем, чтобы они располагались концентрическими кругами с общим центром в точке пересечения листа картона с кондуктором.

Магнитная игла, подвешенная на нити возле этого проводника, займет положение, указанное на рисунке. При изменении направления тока в проводнике магнитная стрелка повернется на угол 180 °, оставаясь в положении, перпендикулярном плоскости вдоль проводника.

В зависимости от направления тока в проводнике направление формируемых им магнитных линий магнитного поля определяется правилом буравчика, которое формулируется следующим образом:

Если поступательное движение буравчика совпадает с направлением тока в проводнике, то вращательное движение его ручки указывает направление магнитных линий поля, сформированных вокруг этого проводника.

Если через проволоку, согнутую в форме кольца, пропускать ток, то под его действием также возникает магнитное поле.
Проволока, изогнутая по спирали и состоящая из нескольких витков, расположенных так, чтобы их оси совпадали, называется соленоидом.

Магнитное поле соленоида

Когда ток проходит через катушку соленоида или один виток провода, возбуждается магнитное поле. Направление этого поля также определяется правилом буравчика. Если поместить ось буравчика перпендикулярно плоскости кольцевого проводника или вдоль оси соленоида и повернуть его ручку в направлении тока, то поступательное движение буравчика укажет направление магнитных линий кольца. или соленоидное поле.

Магнитное поле, возбуждаемое током обмотки соленоида, аналогично магнитному полю постоянного магнита, то есть конец соленоида, из которого выходят магнитные линии, является его северным полюсом, а противоположный конец — южным полюсом.

Направление магнитного поля зависит от направления тока, и когда направление тока в прямолинейном проводнике или в катушке изменяется, направление магнитных линий поля, возбуждаемого этим током, также изменяется.

В однородном магнитном поле во всех точках поле имеет одинаковое направление и одинаковую напряженность.
В противном случае поле называется неоднородным.
Графически однородное магнитное поле изображается параллельными линиями с одинаковой плотностью, например, в воздушном зазоре между двумя противоположными полюсами противоположного магнита.

Винтовая линейка

Прямой провод с током. Ток (I), протекающий по проводу, создает вокруг него магнитное поле (B).

Правое правило

Правило буравчика : «Если направление поступательного движения буравчика (винта) с правой резьбой совпадает с направлением тока в проводнике, то направление вращения ручки буравчика совпадает с направлением магнитного вектор индукции «.

Определение направления магнитного поля вокруг проводника

Правило для правой руки «Если вы поместите большой палец правой руки в направлении тока, то направление захвата проводника четырьмя пальцами покажет направление линий магнитной индукции.”

Для соленоида это сформулировано следующим образом: «Если вы возьмете соленоид ладонью правой руки так, чтобы четыре пальца по очереди были направлены вдоль тока, то большой палец, расположенный в стороны, покажет направление силовых линий магнитного поля. внутри соленоида ».

Линейка левой руки

Ампер обычно используется для определения направления силы. Правило левой руки : «Если вы поместите левую руку так, чтобы линии индукции вошли в ладонь, а вытянутые пальцы были направлены вдоль тока, тогда втянутые большой палец укажет направление силы, действующей на проводник.”

Фонд Викимедиа. 2010.

Посмотреть, что такое «Правило винта» в других словарях:

    Прямой провод с током. Ток (I), протекающий по проводу, создает вокруг него магнитное поле (B). Правило Гимлета (винтовая линейка) или мнемонические варианты правила правой руки … Википедия

    линейка правая

    Прямой провод с током. Ток (I), протекающий по проводу, создает вокруг него магнитное поле (B).Правило буравчика (также правило правой руки) — это мнемоническое правило для определения направления вектора угловой скорости, характеризующего скорость … Wikipedia

    Прямой провод с током. Ток (I), протекающий по проводу, создает вокруг него магнитное поле (B). Правило буравчика (также правило правой руки) — это мнемоническое правило для определения направления вектора угловой скорости, характеризующего скорость … Wikipedia

    gimlet rule — sraigto taisyklė statusas T sritis fizika atitikmenys: angl.правило для пробкового шурупа; винт правило вок. Korkenzieherregel, f; Korkzieherregel, f; Максвеллше Шраубенрегель, ф; Uhrzeigerregel, fr. буравчик правило, n; Правое правило винта, n pranc. … … Fizikos terminų žodynas

    ступица ротора Энциклопедия «Авиация»

    Ступица ротора — Рис. 1. Поворотная ступица ротора. втулка ротора — несущий винт в сборе; он предназначен для крепления лопастей, передачи крутящего момента от вала главного редуктора к лопаткам, а также для восприятия и передачи им… … Энциклопедия «Авиация»

    Ступица ротора — Рис. 1. Поворотная ступица ротора. втулка ротора — несущий винт в сборе; он предназначен для крепления лопастей, передачи крутящего момента от вала главного редуктора к лопастям, а также для восприятия и передачи на … … Энциклопедия «Авиация»

    Ступица ротора — Рис. 1. Поворотная ступица ротора. втулка ротора — несущий винт в сборе; он предназначен для крепления лопастей, передачи крутящего момента от вала главного редуктора к лопаткам, а также для восприятия и передачи им… … Энциклопедия «Авиация»

Facebook

Твиттер

В контакте с

Google+

Диафрагма

Влияние магнитного поля на ток. Правило левой руки. Правило левой руки и электромагнитной индукции

Поместите между полюсами магнита проводник, по которому течет постоянный электрический ток. Сразу отметим, что проводник будет выталкиваться полем магнита из межполюсного пространства.

Это можно объяснить следующим образом. Вокруг проводника с током (рис. 1) образуется собственное магнитное поле, силовые линии которого на одной стороне проводника направлены так же, как силовые линии магнита, а на другой стороне проводника. — в обратном направлении. В результате на одной стороне проводника (на Рисунке 1 выше) магнитное поле оказывается конденсированным, а на другой его стороне (на Рисунке 1 внизу) — разреженным. Поэтому проводник испытывает давление на него.А если проводник не закрепить, то он будет двигаться.

Рисунок 1. Действие магнитного поля на ток.

Для быстрого определения направления движения проводника с током в магнитном поле существует так называемое правило левой руки (рис. 2.).

Рисунок 2. Правило левой руки.

Правило левой руки: если вы поместите левую руку между полюсами магнита так, чтобы линии магнитного поля входили в ладонь, а четыре пальца руки совпадали с направлением тока в проводнике, большой палец покажет направление движения кондуктора.

Итак, на проводнике, по которому протекает электрический ток, действует сила, которая стремится переместить его перпендикулярно магнитным силовым линиям. Опытным путем можно определить величину этой силы. Оказывается, сила, с которой магнитное поле действует на проводник с током, прямо пропорциональна силе тока в проводнике и длине той части проводника, которая находится в магнитном поле (Рисунок 3 слева ).

Это правило действует, если проводник расположен под прямым углом к ​​магнитным силовым линиям.

Рисунок 3. Сила взаимодействия магнитного поля и тока.

Если проводник расположен не под прямым углом к ​​силовым линиям магнитного поля, а, например, как показано на рисунке 3 справа, то сила, действующая на проводник, будет пропорциональна силе тока в проводнике и длине проекция части проводника, находящейся в магнитном поле, на плоскость, перпендикулярную силовым линиям магнитного поля. Отсюда следует, что если проводник параллелен силовым линиям магнитного поля, то сила, действующая на него, равна нулю.Если проводник перпендикулярен направлению силовых линий магнитного поля, то сила, действующая на него, достигает максимального значения.

Сила, действующая на проводник с током, также зависит от магнитной индукции. Чем толще расположены силовые линии магнитного поля, тем больше сила, действующая на проводник с током.

Обобщая все вышесказанное, можно выразить действие магнитного поля на проводник с током следующим правилом:

Сила, действующая на проводник с током, прямо пропорциональна магнитной индукции, силе тока в проводнике и длине выступа части проводника, находящейся в магнитном поле, на плоскость, перпендикулярную магнитному полю. поток.

Следует отметить, что влияние магнитного поля на ток не зависит ни от вещества проводника, ни от его поперечного сечения. Влияние магнитного поля на ток можно наблюдать даже в отсутствие проводника, пропускающего, например, поток быстро движущихся электронов между полюсами магнита.

Воздействие магнитного поля на ток широко используется в науке и технике. Использование этого действия основано на устройстве электродвигателей, преобразующих электрическую энергию в механическое устройство магнитоэлектрических устройств для измерения напряжения и тока, электродинамических громкоговорителей, преобразующих электрические колебания в звук, специальных радиоламп — магнетронов, электронно-лучевых трубок и т. Д. .Они используют магнитное поле для измерения массы и заряда электрона и даже для изучения структуры вещества.

Поместите между полюсами постоянного магнита проводник, по которому протекает электрический ток. Сразу замечаем, что проводник будет выталкиваться полем магнита из межполюсного пространства.

Для быстрого определения направления движения проводника с током, помещенным в постоянное магнитное поле, существует так называемое правило левой руки.

Правило левой руки: если вы поместите левую руку между полюсами магнита так, чтобы силовые линии магнитного поля входили в ладонь и четыре пальца вашей руки совпадали с направлением тока в проводнике, то большой согнутый палец укажет направление движения кондуктора .

Другими словами, легче сказать, что на проводник с током действует некоторая сила, которая стремится вытолкнуть его перпендикулярно магнитным силовым линиям.Опытным путем можно узнать его стоимость. Оказывается, сила, с которой магнитное поле действует на проводник с током, прямо пропорциональна силе тока и длине той части проводника, которая находится в магнитном поле .

Это правило подходит для частного случая, если проводник расположен перпендикулярно силовым линиям магнитного поля. Если проводник с током расположен не под прямым углом к ​​магнитным силовым линиям, то сила, действующая на него, будет пропорциональна току в проводнике и длине выступа части проводника, находящейся в магнитном поле, на плоскость, перпендикулярная магнитным силовым линиям.Отсюда сразу напрашивается вывод: если проводник параллелен силовым линиям магнитного поля, то сила, действующая на него, равна нулю . И наоборот, если проводник перпендикулярен силовым линиям магнитного поля, то сила, действующая на него, достигнет максимального значения.

Сила, действующая на проводник с током, зависит от магнитной индукции. Чем плотнее линии электропередач, тем больше сила, действующая на проводник с током. Следовательно, мы можем выразить влияние магнитного поля на проводник с током следующим определением:

Сила, действующая на проводник с током, прямо пропорциональна магнитной индукции, силе тока и длине выступа, расположенного в магнитном поле в плоскости, перпендикулярной магнитному потоку.

Необходимо добавить, что сила тока не зависит ни от материала проводника, ни от сечения. Действие магнитного поля проявляется даже при отсутствии проводника, пропускающего поток быстро движущихся электронов между полюсами постоянного магнита. Воздействие магнитного поля на ток нашло широкое применение в науке и технике. Эти принципы основаны на устройстве электродвигателей, магнитоэлектрических устройств для измерения напряжения и тока, магнетронов, электронно-лучевых трубок и т. Д.Магнитное поле используется для измерения массы и заряда электрона и даже при изучении структуры вещества.

Когда проводник с током, помещенный в магнитное поле, приводится в движение. это явление магнитной индукции . Есть и обратное явление: когда замкнутый проводник движется в магнитном поле, в нем возникает электрический ток. Это уже называется электромагнитной индукцией . Возьмите проводник с закрытыми концами в гальванометре и быстро пересеките это магнитное поле этим проводником.При этом отметим, что стрелка гальванометра отклоняется в момент пересечения силовых линий магнитного поля. Следовательно, по проводнику будет проходить электрический ток.

Магнитное поле пересекаем проводником в обратном направлении. Стрелка гальванометра снова отклонится, но в другом направлении. Это говорит о том, что по проводнику снова прошел электрический ток, но в обратном направлении.

Таким образом, когда проводник пересекает магнитное поле, в самом проводнике возникает ЭДС, направление которой определяется направлением движения проводника.Эта ЭДС называется индуцированной ЭДС или индукцией ЭДС, то есть наведение ЭДС в проводнике есть не что иное, как явление электромагнитной индукции .

Когда проводник движется, свободные носители заряда движутся вместе с ним. Изучая магнитную индукцию, мы узнали, что на электрический заряд, движущийся в магнитном поле, действует сила в направлении, перпендикулярном направлению магнитного потока. Следовательно, когда свободные носители заряда движутся вместе с проводником, пересекающим силовые линии, на них будут действовать силы, заставляющие носители заряда перемещаться по проводнику, что приводит к появлению электрического тока в проводнике.

Если перемещать проводник в магнитном поле с разной скоростью, заметим, что стрелка гальванометра или микроамперметра будет отклоняться тем больше, чем выше скорость, с которой проводник пересекает магнитное поле. При очень малой скорости движения проводника в нем не генерируется ток, а точнее ток будет настолько мал, что гальванометр не сможет его зафиксировать.

Размещая проводник в пространстве между полюсами, мы тем самым увеличиваем количество линий магнитного поля, охватываемых замкнутым контуром проводника, а при движении проводника назад количество этих линий уменьшается, или, иначе, в первом В этом случае магнитный поток, охватывающий нашу замкнутую петлю, увеличивается, а во втором случае уменьшается.С этой точки зрения возникновение индуцированного тока в замкнутом контуре можно объяснить как изменение величины магнитного потока внутри контура; большее или меньшее отклонение стрелки гальванометра при разных скоростях движения проводника говорит о том, что индукция ЭДС зависит от скорости изменения магнитного потока. внутри контура.

При быстром увеличении (или наоборот) магнитного потока внутри контура большая ЭДС индукции , а при медленном изменении — малая.

Электромагнитная индукция лежит в основе многих электрических устройств, электродинамических микрофонов, звукоснимателей, различных электроизмерительных приборов, электрогенераторов и т. Д.

Линии магнитной индукции представляют собой концентрические окружности с центром на проводнике.

Прямолинейный ток магнитного поля наблюдает через горизонтально расположенный лист картона за вертикальным прямым проводом, который является частью электрической цепи. Опилки-стрелки при замыкании тока в цепи и после легкого постукивания по листу образуют цепочки в виде кругов с общим центром на оси тока.Следовательно, магнитное поле электрического тока графически изображается в виде линий магнитной индукции , аналогичных линиям электростатического поля. Линии магнитной индукции — это окружности с центрами на оси тока, расположенные в плоскостях, перпендикулярных направлению тока. . Их направление определяется правилом правого винта: при поступательном движении винта в направлении тока его вращение указывает направление магнитного поля этого тока .
Разница между линиями магнитной индукции и линиями электростатического поля: первые замкнуты и окружают электрический ток; вторые — разомкнутые, начинаются на поверхности положительно заряженных тел и заканчиваются на поверхности отрицательно заряженных.

Мы применяем закон Био — Савара — Лапласа для расчета магнитных полей простейших токов.

Вывод формулы для постоянного тока магнитного поля:

В качестве константы интегрирования возьмем угол α (угол между векторами dl и r) и выразим через него все остальные значения.

Магнитная индукция, создаваемая одним проводящим элементом, равна

Поскольку угол α для всех элементов постоянного тока изменяется от 0 до π, то

Вычисляем интеграл, и получаем формулу индукции магнитного поля постоянного тока

В формуле мы использовали:

Магнитная индукция постоянного тока

Магнитная проницаемость среды

Магнитная постоянная

Сила тока

Расстояние от провода до точки, где мы рассчитываем магнитную индукцию

Угол между вектором dl и r


Магнитное поле токовой петли , или токовый контур, показано на рисунке (кружок с точкой означает, что на этом сечении ток направлен перпендикулярно плоскости шаблона к нам, а кружок крестиком означает, что ток направлен от нас).Направление линий магнитной индукции вдоль оси петли будет указано магнитной стрелкой, помещенной в ее центр. Две противоположные стороны текущей обтекаемой поверхности можно сравнить с двумя полюсами магнитной стрелки: сторона, с которой выходят линии магнитной индукции, находится с северным полюсом магнитной стрелки, а в которую они входят, — с южным.
Направление магнитного поля токовой петли также можно определить по правилу правого винта: если положить кончик винта в центр катушки и повернуть винт в направлении тока, то его движение вперед будет указывать направление линий магнитной индукции.
Таким образом, между направлениями тока в замкнутом проводнике и его магнитным полем существует взаимная связь, их «сцепление».

Направление вектора магнитной индукции связано с направлением тока в цепи правилом правого винта —

Согласно закону Био-Савара-Лапласа индукция магнитного поля, создаваемого элементом тока dl на расстоянии r от него, составляет

где α — угол между текущим элементом и радиус-вектором, проведенным от этого элемента к точке наблюдения; r — расстояние от текущего элемента до точки наблюдения.

В нашем случае α = π / 2, грех α = 1; где но — расстояние, отсчитываемое от центра катушки до рассматриваемой точки на оси катушки. В этой точке векторы образуют конус с углом решения в вершине 2 = π — 2β где β — угол между сегментами но и r .

Магнитное поле соленоида. Внутри длинного соленоида с током магнитное поле однородно, а линии магнитной индукции параллельны друг другу.Направление B и направление тока в катушках соленоида связаны правилом правого винта

сила тока I . Если соленоид находится в вакууме (или в воздухе), то магнитная индукция поля в нем численно равна

B 0 = μ 0 IN / l = μ 0 In,

, где n = N / l ; В — число ампер-витков на единицу длины соленоида; μ 0 — магнитная постоянная, характеризующая магнитное поле в вакууме .

Поле внутри длинного соленоида однородно и направлено от южного полюса (S) к северному полюсу (N). Модуль магнитной индукции поля в соленоиде пропорционален количеству оборотов усилителя на единицу его длины.

Магнитная постоянная

μ 0 = 4π · 10-7 кг · м / (с 2 · A 2)

Пункт 6 Примеры проявления магнитного взаимодействия:

Итак, движущиеся заряды создают вокруг себя магнитное поле, которое проявляется в действии силы на движущиеся заряды, попадающие в это поле.

Магнитное взаимодействие движущихся электрических зарядов согласно представлениям теории ближнего действия объясняется следующим образом. Каждый движущийся электрический заряд создает магнитное поле в окружающем пространстве. Магнитное поле непрерывно в пространстве и действует на другие движущиеся электрические заряды.

Сила, с которой магнитное поле действует на проводник с током, называется мощностью ампер.
Экспериментальное исследование магнитного взаимодействия показывает, что модуль силы Ампера пропорционален длине проводника l с током и зависит от ориентации проводника в магнитном поле.

Для характеристики способности магнитного поля оказывать силовое воздействие на проводник с током введена векторная величина — магнитная индукция .
Силовой эффект магнитного поля может быть обнаружен по действию силы Ампера на прямой проводник с током и по вращающему эффекту в замкнутом контуре.
При исследовании магнитного поля с помощью прямого проводника с током магнитная индукция определяется следующим образом: модуль магнитной индукции равен отношению максимального значения модуля силы Ампера, действующего на проводник с током, к сила тока I в проводнике и его длина l :

Поз.7 Сила Лоренца сила, действующая со стороны магнитного поля на положительный заряд, движущийся со скоростью ( здесь — скорость упорядоченного движения носителей положительного заряда ) . Модуль силы Лоренца:

где α — угол между и.

На заряд, движущийся по линии, не действует сила ().

Направление силы отрицательного заряда противоположно, поэтому к правило применяется к электронам справа. .

Поскольку сила Лоренца направлена ​​перпендикулярно движущемуся заряду, т.е. перпендикулярно , , работа этой силы всегда равна нулю . Следовательно, действуя на заряженную частицу, сила Лоренца не может изменить кинетическую энергию частицы.

Часто сила Лоренца представляет собой сумму электрических и магнитных сил. :

, г. (2.5.4)

здесь электрическая сила ускоряет частицу, изменяет ее энергию.


Каждый день мы наблюдаем действие магнитной силы на движущийся заряд на экране телевизора.

Движение электронного луча вдоль плоскости экрана стимулируется магнитным полем отклоняющей катушки. Если поднести к плоскости экрана постоянный магнит, то легко заметить его влияние на электронный луч из-за искажений изображения.

Пункт 8 Принцип действия измерительных приборов со шкалой основан на применении электрических устройств, преобразующих движение зонда в изменения тока или напряжения, регистрируемые электроизмерительным прибором, на шкале которого отображаются показания измерений. измеряются.[ 1 ]

Согласно учебнику изучают устройство и принцип действия измерительных приборов магнитоэлектрических, электромагнитных и электродинамических систем, выясняют назначение отдельных узлов и деталей. Используя формулы для крутящих моментов каждой из этих систем, определите область их применения, определите, какие из этих устройств имеют однородную шкалу, а некоторые — неравномерную шкалу, на какой ток они реагируют. [ 3 ]

На взаимодействии проводников с током основан Принцип действия электродинамических и ферродинамических измерительных приборов .[ 4 ]

В работе над сборно-разборными образцами изучается устройство и принципов работы средств измерений, магнитоэлектрических, электромагнитных и электродинамических систем. [ 5 ]

Для прямых измерений, где физический принцип, как правило, определяется однозначно. Принцип измерительного прибора , набор методов (описание метода) имеет метрологический характер. Такие общие метрологические методы, называемые методами прямых измерений, позволяют в некоторых случаях исключить (или компенсировать) наиболее существенные систематические ошибки измерения.[ 6 ]

В основе этого метода лежит использование эффекта физического явления, на котором он основан. Принцип действия измерительного прибора . Например, измерение постоянного тока методом прямой оценки может выполняться с использованием эффекта механического взаимодействия магнитного поля и измеряемого тока. [ 7 ]

Эффективное значение измеряется электромагнитными, электродинамическими и тепловыми системами. Принцип действия измерительных приборов различных систем изучают в процессе электрических измерений. [ 8 ]

Необходимо проводить измерения на образцах с разной толщиной слоя. Следует описать принцип действия применяемых измерительных приборов , и выбор соответствующих средств измерений может происходить в соответствии с каждой конкретной измерительной задачей. [ 13 ]

Магнитоэлектрический измерительный механизм (рис.321, а) выполнен в виде постоянного магнита 1, снабженного полюсными наконечниками 2, между которыми закреплен стальной сердечник 3. В кольцевом воздушном зазоре, образованном полюсными наконечниками и сердечником, подвижная катушка 5 намотана на алюминиевый каркас 6 (рис. 321, б). Катушка сделана из очень тонкой проволоки и закреплена на оси, соединенной со стрелкой винтовыми пружинами 4 или скобами. Через те же пружины или удлинители на катушку подается ток.

Когда ток I проходит через катушку, на каждый из ее проводников действует электромагнитная сила.Суммарное действие всех электромагнитных сил создает крутящий момент M, стремящийся повернуть катушку и соответствующую стрелку устройства на некоторый угол?. Поскольку индукция B магнитного поля создает постоянный магнит, неизменный и не зависящий от тока I, то

M = c 1 I (93)

где c 1 — постоянная величина, зависящая от конструктивных параметров данного устройства (количество витков катушки, ее размер, индукция B в воздушном зазоре).


Простейший генератор можно представить как катушку, вращающуюся в магнитном поле (рис.1.4, а, б). Концы змеевика размещены на двух пластинах коллектора. Неподвижные щетки прижимаются к пластинам коллектора, к которым подключен внешний контур.

Принцип работы генератора основан на явлении электромагнитной индукции. Пусть катушка приводится в движение внешним приводным двигателем. PD Проводники активной части катушки пересекают магнитное поле и ЭДС индуцируются в них электромагнитной индукцией. e 1, и e 2, направление которых определяется правилом правой руки. При вращении катушки по часовой стрелке в верхнем проводнике под северным полюсом ЭДС направлена ​​от нас, а в нижнем проводнике под южным полюсом — к нам. В ходе поворота ЭДС складываются, в результате получается ЭДС e = e 1 — e 2.


Принцип работы двигателя. То же устройство работает в режиме электродвигателя (рис.1.5), если на щетки подается постоянное напряжение. Под напряжением U ток течет через щетки, коллекторные пластины и катушку i. Согласно закону электромагнитной силы (закон Ампера) взаимодействие тока и магнитного поля AT создает напряженность f, которая направлена ​​перпендикулярно i . Направление силы f определяется по правилу левой руки (рис. 1.5): сила действует на верхний проводник вправо, на нижний — влево.Эта пара сил создает крутящий момент Mvr, который поворачивает поворот по часовой стрелке. Когда верхний проводник проходит в зону южного полюса, а нижний проводник — в зону северного полюса, концы проводников и присоединенные к ним коллекторные пластины входят в контакт со щетками разной полярности.

Item 9 В любой точке пространства, окружающего Землю, и на ее поверхности обнаруживается действие магнитных сил. Другими словами, в пространстве, окружающем Землю, создается магнитное поле, силовые линии которого показаны на рис.1.

Магнитный и географический полюса Земли не совпадают друг с другом. Северный магнитный полюс N находится в южном полушарии, у берегов Антарктиды, и южный магнитный полюс S находится в Северном полушарии, недалеко от северного побережья острова Виктория (Канада). Оба полюса непрерывно движутся (дрейфуют) по земной поверхности со скоростью около 5 в год из-за изменчивости процессов, генерирующих магнитное поле.Кроме того, ось магнитного поля не проходит через центр Земли, а отстает от него на 430 км. Магнитное поле Земли не симметрично. Благодаря тому, что ось магнитного поля проходит как раз под углом 11,5 градусов к оси вращения планеты, мы можем использовать компас. Согласно современным представлениям, основная часть магнитного поля Земли имеет внутриземное происхождение. Магнитное поле Земли создается ее ядром. Внешнее ядро ​​Земли жидкое и металлическое.Металл — проводящее вещество, и если бы в жидком ядре существовали постоянные токи, соответствующий электрический ток создал бы магнитное поле. Из-за вращения Земли такие токи существуют в ядре, поскольку Земля в некотором приближении представляет собой магнитный диполь, то есть своего рода магнит с двумя полюсами: южным и северным.

NS на резьбе л


Наличие магнитного поля в любой точке Земли можно установить с помощью магнитной стрелки.Если повесить магнитную иглу NS на резьбе л (рис. 2) так, чтобы точка поворота совпадала с центром тяжести стрелки, тогда стрелка установится в направлении касательной к силовой линии магнитного поля Земли.

В северном полушарии — южный конец будет наклонен к Земле, а стрелка будет с углом наклона горизонта Q (на магнитном экваторе Q равно нулю).Вертикальная плоскость, в которой расположена стрелка, называется плоскостью магнитного меридиана. Все плоскости магнитных меридианов пересекаются по прямой. NS , а следы магнитных меридианов на земной поверхности сходятся в магнитных полюсах N и S . Поскольку магнитные полюса не совпадают с географическими полюсами, стрелка будет отклонена от географического меридиана.Угол, который образует вертикальную плоскость, проходящую через стрелку (т. Е. Магнитный меридиан) с географическим меридианом, называется магнитным склонением (рис. 2). Вектор полей магнитного поля Земли можно разложить на две составляющие: горизонтальную и вертикальную (рис. 3). Значение углов наклона и склонения, а также горизонтальной составляющей позволяет определить величину и направление полной напряженности магнитного поля Земли в данной точке.Если магнитная стрелка может свободно вращаться только вокруг вертикальной оси, то она будет установлена ​​под действием горизонтальной составляющей магнитного поля Земли в плоскости магнитного меридиана. Горизонтальная составляющая, магнитное склонение a и наклон Q называется стихией земного магнетизма. Все элементы земного магнетизма

меняются с течением времени.

Единица измерения напряженности геомагнитного поля () в системе C — ампер на метр (А / м).Другая единица Эрстеда (E) или гамма, равная 10 -5 э, также использовалась в магниторазведке. . Однако практически измеряемым параметром магнитного поля является магнитная индукция (или плотность магнитного потока), где — магнитная проницаемость среды. Единицей магнитной индукции в системе C является Тесла (Тл). В магниторазведке используется меньшая единица нанотесла (нТл), равная 10 -9 Т. Поскольку для большинства сред, в которых изучается магнитное поле (воздух, вода, подавляющее большинство немагнитных осадочных пород), Магнитное поле Земли можно измерить количественно либо в единицах магнитной индукции (в нТл), либо в соответствующей напряженности поля — гамма


Направление тока и направление линий его магнитного поля (Зарицкий А.Н.)
Электромагнитные явления

В этом уроке мы узнаем о магнитном эффекте тока на примере опыта Эрстеда и опыта Ампера.Рассмотрим правило Гимле и правило правой руки для прямого проводника с током и для соленоида с током

В течение долгого времени электрическое и магнитное поля изучались отдельно. Но в 1820 году датский ученый Ганс Кристиан Эрстед во время лекции по физике обнаружил, что магнитная стрелка вращается около проводника с током (см. Рис. 1). Доказано магнитное действие тока. После проведения нескольких экспериментов Эрстед обнаружил, что вращение магнитной стрелки зависит от направления тока в проводнике.

Рис. 1. Опыт Oersted

Чтобы представить себе принцип вращения магнитной стрелки около проводника с током, рассмотрим вид с торца проводника (см. Рис. 2, ток направлен на рисунок — с рисунка), около которого магнитный стрелки установлены. После прохождения тока стрелки определенным образом выстраиваются противоположными полюсами друг к другу. Поскольку магнитные стрелки выстраиваются по касательной к магнитным линиям, магнитные линии, направляющие ток, представляют собой круги, и их направление зависит от направления тока в проводнике.


Рис. 2. Расположение магнитных стрелок возле прямого проводника с током

Для более наглядной демонстрации магнитопровода с токопроводом может быть проведен следующий эксперимент. Если вокруг проводника с током облить железные опилки, то через некоторое время опилки, попав в магнитное поле проводника, намагничиваются и помещаются в круги, окружающие проводник (см. Рис. 3).

Рис.3. Расположение железных опилок вокруг проводника с током ()

Для определения направления магнитных линий возле проводника с током существует правило буравчика (правило правого винта) — если вы вкручиваете винт по направлению тока в проводнике, то направление вращения ручки винта будет указывать направление линий магнитного поля тока (см. рис. 4).

Рис. 4. Правило Гимле ()

Также можно использовать правило правой руки — если направить большой палец правой руки в направлении тока в проводнике, то четыре согнутых пальца укажут направление силовых линий текущего магнитного поля (см.рис.5).

Рис. 5. Правило правой руки ()

Оба эти правила дают одинаковый результат и могут использоваться для определения направления тока вдоль направления силовых линий магнитного поля.

После открытия появления магнитного поля возле проводника с током Эрстед разослал результаты своих исследований большинству ведущих европейских ученых. Получив эти данные, французский математик и физик Ампер начал серию экспериментов и через некоторое время продемонстрировал публике опыт взаимодействия двух параллельных проводников с током.Ампер обнаружил, что если электрический ток течет в одном направлении по двум проводникам, параллельным друг другу, такие проводники притягиваются друг к другу (см. Рис. 6 b), если ток течет в противоположных направлениях, проводники отталкиваются друг от друга (см. Рис. 6 a. ).


Рис. 6. Ампер Опыт ()

Ампер сделал следующие выводы из своих экспериментов:

1. Магнитное поле существует вокруг магнита, проводника или электрически заряженной движущейся частицы.

2. Магнитное поле действует с некоторой силой на заряженную частицу, движущуюся в этом поле.

3. Электричество — это направленное движение заряженных частиц, поэтому магнитное поле действует на проводник с током.

На рисунке 7 показан прямоугольник провода, направление тока в котором показано стрелками. Используя правило буравчика, проведите магнитную линию по сторонам прямоугольника, указав ее направление стрелкой.

Рис. 7. Иллюстрация проблемы

Решение

По сторонам прямоугольника (токопроводящей рамки) закручиваем воображаемый буравчик по направлению тока.

Рядом с правой стороной кадра магнитные линии выйдут из рисунка слева от проводника и войдут в плоскость рисунка справа от него. На это указывает линейка стрелки в виде точки слева от проводника и креста справа от него (см. Рис. 8).

Аналогичным образом определяем направление магнитных линий около других сторон кадра.


Рис. 8. Иллюстрация проблемы

Эксперимент Ампера, в котором магнитные стрелки были установлены вокруг катушки, показал, что когда через катушку протекает ток, стрелки к концам соленоида устанавливаются с разными полюсами вдоль воображаемого линии (см. рис.9). Это явление показало, что рядом с катушкой с током существует магнитное поле, а также что у соленоида есть магнитные полюса. Если вы измените направление тока в катушке, магнитные стрелки повернутся.

Рис. 9. Опытный ампер. Формирование магнитного поля возле токовой катушки

Для определения магнитных полюсов токовой катушки используется правило правой руки для соленоида (см. Рис. 10) — если вы оборачиваете соленоид ладонью правой руки, указывая четырьмя пальцами в направлении тока в катушки, большой палец покажет направление силовых линий магнитного поля внутри соленоида, то есть на его северном полюсе.Это правило позволяет определить направление тока в витках катушки по расположению ее магнитных полюсов.


Рис. 10. Правило правой руки для соленоида с током

Определите направление тока в катушке и полюса источника тока, если магнитные полюса, показанные на рисунке 11, появляются при прохождении тока через катушку. .

Рис. 11. Иллюстрация к заданию

Решение

Согласно правилу правой руки для соленоида, мы обнимаем катушку так, чтобы большой палец указывал на ее северный полюс.Четыре согнутых пальца укажут направление тока по проводнику, поэтому правый полюс источника тока положительный (см. Рис. 12).

Рис. 12. Иллюстрация к задаче

В этом уроке мы рассмотрели явление возникновения магнитного поля вблизи проводника постоянного тока и токовой катушки (соленоида). Также были изучены правила нахождения магнитных линий этих полей.

Библиография

  1. А.В. Перышкин, Е.М.Гутник. Физика 9. — Дрофа, 2006.
  2. .
  3. Г. Степанова. Сборник задач по физике. — М .: Просвещение, 2001.
  4. .
  5. А. Фадеев. Тесты по физике (7-11 классы). — М., 2002.
  6. .
  7. В. Григорьев, Г. Мякишев. Силы в природе. — М .: Наука, 1997.
  8. .

Домашнее задание

  1. Интернет-портал Clck.ru ().
  2. Интернет-портал Class-fizika.narod.ru ().
  3. Интернет-портал Festival.1september.ru ().

E.Резерфорд. 39000 убитых. «Причины и природа радиоактивности». А.А. Беккерель. Цель урока: Хиросима и Нагасаки. 66000 погибло. Содержание: «Атомная энергия — за и против?». М. Кюри. 1945 г. Урок спора 11 класс. А потом зоркий пилот, так что от природы мастерски воняют ветрами. Выступление учителя физики общеобразовательной школы № 23 стр. Новоназначенная Ошкина Л.Б. Э. Ферми. А потом умелой рукой направить и нести Свой корабль »Аристофан V-IV вв.ДО Н.Э. ? — Распад ZAX = Z-2A-4X + 24he? — Распад ZAY = Z + 1AY + -10e. Ф. Содди. Чернобыль.

«Излучение и спектры» — Содержание. Спектры, линейный спектр. Атомная радиация. Полосатый спектр состоит из отдельных полос, разделенных темными промежутками. Спектры в природе. И виды излучения. Из-за католюминесценции загораются экраны телевизоров с электронно-лучевой трубкой. Фотолюминесценция. Справка по истории. Непрерывный спектр. Виды излучения. Пример лампы дневного света. Начать просмотр.

«Электролиз» — Выполнил: ученик 11М класса МОУ лицея № 6 Эрик Аббязов. Электролиз расплава. Основные положения электродных процессов. На аноде (+) окисление. Повышенная окислительная ионная активность. Примеры электролиза расплава: раствор NaCl Катод (-) Анод (+) h30. Анион хлора отправляется на анод. Cu2 +, Ag +, Hg2 +, Pt2 +, Au3 +. В серии напряжений металла натрий находится намного левее водорода. Если расплавить соль, то произойдет расщепление кристаллической решетки на ионы.

«Power Transmission» — Power Transmission. 35 кВ. ~. Генератор. Следовательно, возникает необходимость передавать электроэнергию на большие расстояния. 6 кВ. Потребителю. Повышающий трансформатор. Электричество невозможно сохранить в больших масштабах. Потребление электроэнергии 110 кВ. Потребители электроэнергии есть везде. Линия передачи. Понижающий трансформатор. 11 кВ.

«Использование электроэнергии» — Сердцевиной кабеля являются три токопроводящих провода (по количеству фаз). Работу выполнили учащиеся 11 Б класса лицея №18 Лилия Фархаева и Николай Усанин. Снаружи кабель залит битумом. Кабельные линии (КЛ) проходят под землей. Схема передачи электроэнергии. На оптовом рынке обычно используется последнее ценовое выражение. В настоящее время переменный ток используется практически повсеместно.

«Класс электромагнитной индукции 11» — Что такое ферромагнетики? Что такое магнитная проницаемость? Какие вещества называют диа- и парамагнитными? Вопросы. Открытие электромагнитной индукции.11 класс. Где используются ферромагнетики? Магнитный поток. Электромагнитная индукция. Ток, возникающий в замкнутой цепи, называется индукционным. Эксперименты Фарадея. Правило Ленца. Каковы свойства ферромагнетиков?

Магнитное поле Презентация магнитной индукции. Магнитная индукция. B

Чтобы использовать предварительный просмотр презентаций, создайте себе аккаунт (аккаунт) Google и войдите в него: https://accounts.google.com


Подписи к слайдам:

Урок — лекция «Магнитное поле.Постоянные магниты и текущее магнитное поле. Индукция магнитного поля »Региональное государственное автономное образовательное учреждение среднего профессионального образования Белгородский строительный техникум Белгород

Впервые связь между электрическими и магнитными явлениями была обнаружена в 1820 году Гансом Кристианом Эрстедом: если над проводником направить магнитную иглу, по земному меридиану, а по проводнику пропускается электрический ток, затем стрелка отклоняется на определенный угол.

В 1820 году Андре Ампер открыл закон взаимодействия проводников с током.

Магнитное поле — это особая форма материи, через которую осуществляется взаимодействие между движущимися заряженными частицами. Основные свойства магнитного поля: Магнитное поле создается электрическим током (движущиеся заряды). Магнитное поле обнаруживается действием электрического тока (движущиеся заряды). Магнитное поле действительно существует независимо от нас, от наших знаний о нем.

Магнит — тело, имеющее собственное магнитное поле Типы магнитов: Природные магниты (магнитная руда) образуются, когда руда, содержащая железо или оксиды железа, охлаждается и намагничивается земным магнетизмом. Временные магниты — они действуют как постоянные магниты, только когда они находятся в сильном магнитном поле, и теряют свой магнетизм, когда магнитное поле исчезает (скрепки и гвозди). Электромагниты представляют собой металлический сердечник с индукционной катушкой, по которой протекает электрический ток.

Полюса магнита нельзя разделить!

Магнитное поле и его графическое изображение В качестве направления мы договорились принять направление северного конца магнитной стрелки.Силовые линии выходят из северного полюса и входят, соответственно, в южный полюс магнита. Линии магнитной индукции — это кривые, касательные которых в каждой точке совпадают с направлением вектора в этой точке.

Правило подвеса: если направление поступательного движения подвеса совпадает с направлением тока в проводнике, то направление вращения ручки подвеса совпадает с направлением вектора магнитной индукции.

Правило Гимлета

Важной особенностью линий магнитной индукции является то, что они не имеют начала и конца.Они всегда закрыты. Поля с замкнутыми силовыми линиями называются вихревыми. Магнитное поле — это вихревое поле. Магнитные линии с прямым проводником Магнитные линии соленоида (катушки)

Силовые линии магнита Конфигурацию силовых линий магнита легко определить с помощью небольших железных опилок, которые намагничиваются в исследуемом магнитном поле и ведут себя как маленькие магнитные стрелки (вращаются по силовым линиям). Магнитное поле одноименных полюсов Магнитное поле противоположных полюсов

Формула связи между вектором магнитной индукции и напряженностью магнитного поля: — вектор магнитной индукции (Т) — напряженность магнитного поля (А / м) — магнитная проницаемость среды (для вакуума = 1) — магнитная постоянная

Закон Био — Савара — Лапласа: H — напряженность магнитного поля в заданной точке (А / м) I — сила тока (А) l — длина сечения проводника (м) r — радиус-вектор, соединяющий сечение проводника с рассматриваемой точкой поля, угол между направлением тока в сечении и радиусом -вектором

Основные источники: Генденштейн Л.Э. Дик Ю.И. Физика. Учебник за 11 кл. — М., 2005. Дмитриева В.Ф. Задания по физике: учебник. разрешение. — М., 2003. Сборник задач и вопросов по физике, под ред. Р.А. Гладкова. — М., 2003. Дополнительные источники: Громов С.В. Шаронова Н.В. Физика, 10-11: Книга для учителя. — М., 2004. Касьянов В.А. Методические рекомендации по использованию учебников В.А. Касьянова «Физика. 10 кл. «,» Физика. 11 кл. «при изучении физики на базовом и профильном уровне. — М., 2006. Касьянов В.А. Физика.10, 11 кл. Тематическое и учебное планирование. — М., 2002. Федеральный компонент государственного стандарта общего образования / Минобрнауки России. — М., 2004. Интернет-ресурсы youtube .com http: // www.kakprosto.ru http: // ru.wikipedia.org / http: // allforchildren.ru / why /whatis37.php http: // elektrobgau.narod .ru /

«Энергия магнитного поля» — Определение индуктивности. Скалярная величина. Энергия катушки. Плотность энергии магнитного поля. Переходные процессы.Самоиндукция. Импульсное магнитное поле. Время отдыха. Плотность энергии. Постоянные магнитные поля. Экстракции в цепи с индуктивностью. Колебательный контур. Расчет индуктивности.

«Движение частиц в магнитном поле» — Изменение параметров. Электронно-лучевая трубка. Контрольные вопросы. Проявление действия силы Лоренца. Ценить. Масс-спектрограф. Применение силы Лоренца. Цель эксперимента. Магнитное поле. Циклотрон. Направления силы Лоренца. Движение частиц в магнитном поле.Сила Лоренца. Определение величины силы Лоренца.

«Характеристики магнитного поля» — Направленное движение зарядов. Поле. Поверните по силовым линиям. Прямое текущее поле. Выражение для определения магнитной индукции. Магнитные взаимодействия. Линии магнитной индукции. Определение потока вектора магнитной индукции. Закон Био-Савара-Лапласа. Движение заряженных частиц в магнитосфере Земли.

«Определение магнитного поля» — Магнитное поле.Тела, долго сохраняющие намагниченность. Экспериментальное задание. Графическое представление магнитных полей. Ж. Верн. Действие электрического тока. На основании данных, полученных в ходе экспериментов, заполним таблицу. Оборудование. Ганс Кристиан Эрстед. Этап обобщения и систематизации знаний.

«Магнитное поле, магнитные линии» — Силовые линии магнитного поля полоскового магнита. Направление. Определение направления магнитной линии.Магнит имеет разные силы притяжения в разных областях. Полосовой магнит. Перемещение электрических зарядов. Магнитопроводы соленоида. Магнитное поле. Как можно найти МП. Различные искусственные магниты.

«Свойства магнитного поля» — Линии магнитной индукции. Линии магнитной индукции всегда замкнуты. Взаимодействие токов. Магнитное поле Земли. Электроизмерительные приборы. Магнитное поле проявляется как воздействие на проводники с током. Масс-спектрограф. Модуль вектора магнитной индукции.Постоянный магнит. Некоторые значения магнитной индукции.

Всего 20 презентаций

1. Вокруг Земли есть магнитное поле, как оно создается? 2. Есть постоянные магниты и …………………. Магниты. Магнитное поле существует вокруг любого проводника с током, то есть вокруг движущихся электрических зарядов. Электрический ток и магнитное поле неотделимы друг от друга. Таким образом, вокруг неподвижных электрических зарядов есть только электрическое поле, вокруг движущихся зарядов, т.е.е. электрический ток, существуют как электрические, так и магнитные поля. Магнитное поле возникает вокруг проводника, когда в нем возникает ток, поэтому ток следует рассматривать как источник магнитного поля. Катушка с железным сердечником внутри называется электромагнитом. Катушки тока широко используются в технике в качестве магнитов. Они удобны тем, что их магнитное действие можно изменять. Магнитный эффект катушки с током тем сильнее, чем больше в ней витков. С увеличением силы тока влияние магнитного поля токовой катушки усиливается, с уменьшением — ослабевает.Железо, вставленное внутрь катушки, усиливает магнитное действие катушки. Катушка с железным сердечником внутри называется электромагнитом.


Магнитный сепаратор В зерно примешиваются очень мелкие железные опилки. Эти опилки не прилипают к гладким зернам здоровых зерен, а прилипают к зёрнам сорняков. Зерна 1 высыпаются из бункера на вращающийся барабан 2. Внутри барабана находится сильный электромагнит. Притягивая частицы 4 железа, он уносит зерна сорняков из потока 3 зерна, и, таким образом, зерно очищается от сорняков и случайно попавших в него железных предметов.




Магнитное поле характеризуется векторной физической величиной, которая обозначается символом и называется индукцией магнитного поля (или магнитной индукцией). Модуль этой силы зависит от самого магнитного поля. Кроме того, сила магнитного поля на проводнике пропорциональна длине l этого проводника и силе тока I в нем.


Никола Тесла Обычно упоминание имени этого ученого в школьных учебниках связано с единицей магнитной индукции (1 Тесла), названной в его честь.Он был гениальным изобретателем и ученым, опередившим свое время. За свою жизнь Н. Тесла сделал около 1000 различных изобретений и открытий, получил почти 800 патентов на изобретения в различных областях техники. О нем ходили самые разные слухи, его называли колдуном и мистификатором. Тесла так далеко ушел от официальной науки, что сегодня большинство его работ остаются непонятными и необъяснимыми.

Одержимость Теслы наукой не знала границ. Он выделял четыре часа на сон, из которых два обычно тратил на обдумывание идей.Лорд Кельвин писал о нем как о «самом преданном человеке в области электротехники из всех своих современников». После 1900 года он получил множество патентов на изобретения в различных областях техники. Он открыл переменный ток, флуоресцентный свет, беспроводную передачу энергии, построил первые электрические часы, двигатель на солнечной энергии. Он изобрел радио до Маркони и Попова, получил трехфазный ток до Доливо-Добровольского. Вся энергетическая промышленность 20 века выросла на его патентах. В 1917 году Тесла был награжден медалью Эдисона — высшей наградой Американского института инженеров-электриков.В 30-е годы ему была присуждена Нобелевская премия. Но он отказался принять это, не желая делиться этим с Эдисоном, которому он не прощал критику переменного тока до конца своих дней.

Лекции Николы Теслы были красочным шоу, а обвинения в магии постоянно сопровождали деятельность Теслы. Тесла вытаскивал из своего портфеля небольшой трансформатор, работающий на высокочастотном переменном токе при высоком напряжении и чрезвычайно низкой силе тока. Когда он его включил, вокруг него закрутились молнии.И он спокойно ловил их руками, а люди с первых мест в зале поспешно отступали.

В 1893 году Tesla устроила шоу на Всемирной выставке в Чикаго. Стоя на подиуме в центре выставочного зала, он пропустил через себя ток в два миллиона вольт. По мнению Эдисона, от сумасшедшего серба не должно было остаться даже пыли. Однако Тесла спокойно улыбался, а в руке Эдисона горел свет, получая энергию как бы из ниоткуда. Теперь мы знаем, что ток высокой частоты проходит только по поверхности, не причиняя вреда человеку.Тогда этот фокус казался чудом.

Эксперимент с лампочкой был хорошим зрелищем. Тесла включил свой трансформатор, и в его руках начала светиться обычная лампочка. Это уже было поразительно. Когда он достал из своего портфеля лампочку без нити накала, просто пустую лампочку, а она все еще светилась, удивлению публики не было предела. Или во время своей лекции о высокочастотном электромагнитном поле перед учеными Королевской академии Тесла дистанционно включал и выключал электродвигатель, в его руках лампочки загорались сами по себе.Тогда это был 1892 год!

Передача энергии без проводов и де и Тесла увлекся идеей передачи энергии на расстояние без проводов и ему удалось добиться выдающихся успехов в этой области. В Tesla он публично демонстрирует возможность беспроводной передачи электроэнергии на большие расстояния и проводит грандиозный опыт беспроводной передачи энергии. Он утверждал, что сделал самое важное открытие — земные стационарные волны.Земля может служить проводником.

Чтобы использовать предварительный просмотр презентаций, создайте себе учетную запись (учетную запись) Google и войдите в нее: https://accounts.google.com


Подписи к слайдам:

Индукция магнитного поля

Думающий ум не чувствует себя счастливым пока он не сможет связать воедино разрозненные факты, которые он наблюдает. Hevesi

Теоретические вопросы: Что такое магнитное поле? Что создает магнитное поле? Кто первым открыл магнитное поле вокруг проводника с током?

Как графически представить магнитное поле? Как получить изображение магнитных линий с помощью железных опилок? Каковы магнитные линии прямого проводника, соленоида и постоянного магнита?

Что делает магнитное поле? Как экспериментально обнаружить наличие силы, действующей на проводник с током в магнитном поле? Как определить направление этой силы? Сформулируйте правило левой руки.

Проверьте свое домашнее задание Определите направление силы, действующей на проводник со стороны постоянного магнита. Определите направление тока в проводнике.

Без сомнения, все наши знания начинаются с опыта. Иммануил Кант

Индукция магнитного поля Вывод 1: Магнитные поля различаются по силе воздействия на железные предметы, проводники с током и движущиеся заряды.

Модуль вектора магнитной индукции F магнитного поля силы тока I длины проводника LF зависит от:

F / IL = const B = F / IL Тесла 1Tl = 1N / ( А м) Вывод 2: Магнитная индукция — это силовая характеристика магнита.поля.

Направление вектора магнитной индукции Вывод 3: Вектор B направлен по касательной к магнитным линиям. Направление вектора B указывает на северный полюс магнитной стрелки.

Типы магнитных полей: Однородное поле Неоднородное поле Вывод 4: Магнитное поле однородно, если во всех его точках магнитная индукция одинакова как по величине, так и по направлению.

Ответьте на вопросы: Как называется силовая характеристика магнитного поля? Как это обозначено? По какой формуле рассчитывается модуль вектора магнитной индукции? Можно сказать, что модуль магнитной индукции зависит от силы, с которой действует магнит.поле действует на проводник током, силой тока и длиной проводника? Как называется агрегат для измерения магнитной индукции. Используя рисунки 120,121,122 (стр. 159), установите, какие поля однородны, а какие нет.

Пройдите тест и проверьте себя. Вариант -1 Вариант-2 1-A 1-B 2-B 2-A 3-A 3-B 4-A 4-B 5-B, C, D 5-A

Домашнее задание: § 46, ответы на вопросы после §, упражнение: 37 (письменно)

Краткое содержание урока Я понял и запомнил материал урока, доволен собой.Материал показался мне очень сложным и неинтересным, поэтому мне было скучно.


По теме: методические разработки, презентации и заметки

Самостоятельная работа «Магнитное поле и его изображение. Действие магнитного поля на проводник с током. Lorentz Force »в 12 вариантах. Физика 9 класс.

Эта самостоятельная работа поможет отработать навыки определения силы Ампера, силы Лоренца на уроках физики в 9 классе и как повторение на уроках в 10 классе…

Магнитное поле. Действие магнитного поля на проводник с токами.

Вводное занятие раздела «Электромагнитное поле» по теме «Магнитное поле. Воздействие магнитного поля на проводник с током.», 9 класс. Урок разработан с использованием технологии критического мышления с использованием .. .

«Магнитное поле. Действие магнитного поля на проводник с током»

Данная презентация используется в первом уроке по теме «МАГНИТНОЕ ПОЛЕ».В него вошли опыт Эрстеда, эксперименты Ампера, определение магнитного поля, линии магнитной индукции, определение и формула были даны …

  • 2. Как определяется магнитное поле?
  • 3. Как отображается магнитное поле?
  • 4. Каковы магнитные линии прямого проводника, соленоида и постоянного магнита?

  • 5. Как определить направление линий

магнитное поле?

  • 6.Как определить направление силы, действующей на проводник с током в магнитном поле?
  • 7. Как определить направление силы, действующей на проводник с током

в магнитном поле?



Z Определить направление силы Ампера



Магнитное поле характеризуется

значением, которое

определяет напряженность

магнитного поля


показывает, насколько сильное магнитное поле действует на проводник с током

[B] = 1Tl (Тесла)

B = F / IL


  • 2.Проводник с током (первое правое правило)
  • 3. Соленоид (второе правое правило)

  • Объектив 1. В магнитном поле с индукцией В помещен проводник с током. Через некоторое время ток в проводнике увеличился в 4 раза. Изменилась ли индукция в этом случае IN магнитного поля, в которое помещался проводник?

  • Цель 2. Какая сила действует на проводящую шину длиной 10 м, по которой проходит ток 7000 А в магнитном поле с индукцией 1?8 т?

  • Объектив 3. Сила тока в горизонтально расположенном проводнике длиной 20 см и весом 4 г составляет 10 А. Найдите индукцию (модуль и направление) магнитного поля, в котором проводник должен быть помещен таким образом, чтобы сила тяжести уравновешивается силой Ампера.

  • Домашнее задание: §46, упражнение 37

Формула центростремительного ускорения для силы Лоренца. Сила Лоренца. Определение силы Лоренца

« Физика — 11 класс»

Магнитное поле действует с силой на движущиеся заряженные частицы, в том числе проводники с током.
Какая сила действует на одну частицу?

1.
Сила, действующая на движущуюся заряженную частицу со стороны магнитного поля, называется силой Лоренца в честь великого голландского физика Х. Лоренца, создавшего электронную теорию строения материи.
Сила Лоренца может быть найдена с помощью закона Ампера.

Модуль силы Лоренца равен отношению модуля силы F, действующей на участок проводника длиной Δl, к количеству N заряженных частиц, движущихся упорядоченно в этом участке проводника:

Так как сила (сила Ампера), действующая на сечение проводника со стороны магнитного поля
, равна F = | Я | BΔl sin α ,
и ток в проводнике I = qnvS
, где
q — заряд частицы
n — концентрация частиц (т.е.е., количество зарядов в единице объема)
v — скорость частицы
S — сечение проводника.

Тогда получаем:
На каждый движущийся заряд со стороны магнитного поля действует сила Лоренца , равная:

где α — угол между вектором скорости и вектором магнитной индукции.

Сила Лоренца перпендикулярна векторам и.

2.
Направление силы Лоренца

Направление силы Лоренца определяется с использованием тех же правил левой руки , что и направление силы Ампера:

Если левая рука расположена так, что составляющая магнитной индукции, перпендикулярная скорости заряда, входит в ладонь, а четыре вытянутых пальца направлены вдоль движения положительного заряда (против движения отрицательного), то большой палец, согнутый на 90 °, укажет направление силы Лоренца, действующей на заряд F l

3.
Если в пространстве, где движется заряженная частица, есть и электрическое поле, и магнитное поле, то полная сила, действующая на заряд, равна: = el + l, где сила, с которой электрическое поле действует на заряд q равно F el = q …

4.
Сила Лоренца не работает , поскольку она перпендикулярна вектору скорости частицы.
Это означает, что сила Лоренца не изменяет кинетическую энергию частицы и, следовательно, модуль ее скорости.
Под действием силы Лоренца изменяется только направление скорости частицы.

5.
Движение заряженной частицы в однородном магнитном поле

Имеется однородное магнитное поле , направленное перпендикулярно начальной скорости частицы.

Сила Лоренца зависит от модулей векторов скорости частиц и индукции магнитного поля.
Магнитное поле не изменяет модуль скорости движущейся частицы, а это означает, что модуль силы Лоренца остается неизменным.
Сила Лоренца перпендикулярна скорости и, следовательно, определяет центростремительное ускорение частицы.
Неизменность по модулю центростремительного ускорения частицы, движущейся с постоянной скоростью по модулю, означает, что

В однородном магнитном поле заряженная частица движется равномерно по окружности радиуса r .

По второму закону Ньютона

Тогда радиус круга, по которому движется частица, равен:

Время, за которое частица совершает полный оборот (период обращения), равно:

6.
Использование действия магнитного поля на движущийся заряд.

Действие магнитного поля на движущийся заряд применяется в телевизионных трубках-кинескопах, в которых летящие к экрану электроны отклоняются магнитным полем, создаваемым специальными катушками.

Сила Лоренца используется в циклотроне, ускорителе заряженных частиц, для производства частиц высоких энергий.

Устройство масс-спектрографов также основано на действии магнитного поля, что позволяет точно определять массы частиц.

Нигде школьный курс физики не имеет такого сходства с большой наукой, как электродинамика. В частности, его краеугольный камень — воздействие на заряженные частицы со стороны электромагнитного поля, нашло широкое применение в электротехнике.

Формула силы Лоренца

Формула описывает взаимосвязь между магнитным полем и основными характеристиками движущегося заряда. Но для начала нужно разобраться, что это такое.

Определение и формула силы Лоренца

В школе часто показывают опыт работы с магнитом и железными опилками на листе бумаги. Если поместить его под бумагу и слегка встряхнуть, то опилки выстроятся вдоль линий, которые обычно называют линиями магнитного натяжения. Проще говоря, это силовое поле магнита, окружающего его, как кокон. Он замкнут на себе, то есть не имеет ни начала, ни конца. Это векторная величина, направленная от южного полюса магнита на север.

Если в него влетит заряженная частица, поле подействует на нее очень любопытным образом. Она не сбавляла скорость и не ускорялась, а только отклонялась в сторону. Чем быстрее и сильнее поле, тем сильнее на него действует эта сила. Она была названа силой Лоренца в честь физика, который первым открыл это свойство магнитного поля.

Рассчитайте по специальной формуле:

здесь q — величина заряда в кулонах, v — скорость, с которой движется заряд, в м / с, а B — индукция магнитного поля в единицах измерения Т (Тесла).

Направление силы Лоренца

Ученые заметили, что существует определенная закономерность между тем, как частица летит в магнитное поле и где она ее отклоняет. Чтобы их было легче запомнить, они разработали специальное мнемоническое правило. Чтобы его запомнить, нужно совсем немного усилий, потому что он использует то, что всегда под рукой — руку. Точнее, левая рука, в честь которой она называется правилом левой руки.


Итак, ладонь должна быть раскрыта, четыре пальца смотрят вперед, большой палец выступает в сторону.Угол между ними равен 900. Теперь необходимо представить, что магнитный поток — это стрела, которая вонзается в ладонь изнутри и выходит сзади. В то же время пальцы смотрят в том же направлении, в котором летит воображаемая частица. В этом случае большой палец покажет, куда он отклонится.

Интересно!

Важно отметить, что правило левой руки применяется только к частицам со знаком плюс. Чтобы узнать, куда отклонится отрицательный заряд, нужно указать четырьмя пальцами в том направлении, откуда летит частица.Все остальные манипуляции остаются прежними.

Последствия действия силы Лоренца

Тело летит в магнитное поле под определенным углом. Интуитивно понятно, что его значение имеет некоторое значение для характера воздействия поля на него, здесь вам нужно математическое выражение, чтобы сделать его более понятным. Вы должны знать, что и сила, и скорость — векторные величины, то есть у них есть направление. То же самое и с линиями магнитного натяжения.Тогда формулу можно записать так:

sin α здесь угол между двумя векторными величинами: скоростью и потоком магнитного поля.

Как известно, синус нулевого угла тоже равен нулю. Оказывается, если траектория частицы движется по силовым линиям магнитного поля, то она никуда не отклоняется.


В однородном магнитном поле силовые линии находятся на одинаковом и постоянном расстоянии друг от друга.Теперь представьте, что частица движется перпендикулярно этим линиям в таком поле. В этом случае сила Лоуренса заставит его двигаться по окружности в плоскости, перпендикулярной силовым линиям. Чтобы найти радиус этого круга, вам нужно знать массу частицы:

Величина заряда взята не случайно за модуль. Это означает, что не имеет значения, входит ли в магнитное поле отрицательная или положительная частица: радиус кривизны будет одинаковым. Изменится только направление, в котором он летит.

Во всех остальных случаях, когда заряд находится под определенным углом α с магнитным полем, он будет двигаться по траектории, напоминающей спираль с постоянным радиусом R и шагом h. Его можно найти по формуле:

Еще одним следствием свойств этого явления является то, что она не выполняет никакой работы. То есть он не отдает и не забирает энергию у частицы, а только меняет направление ее движения.


Наиболее яркой иллюстрацией этого эффекта взаимодействия магнитного поля и заряженных частиц является северное сияние.Магнитное поле, окружающее нашу планету, отклоняет заряженные частицы, приходящие от Солнца. Но поскольку он наиболее слаб на магнитных полюсах Земли, электрически заряженные частицы проникают туда, заставляя атмосферу светиться.

Центростремительное ускорение, придаваемое частицам, используется в электрических машинах — электродвигателях. Хотя здесь уместнее говорить о силе Ампера — частном проявлении силы Лоуренса, действующей на проводник.

Принцип работы ускорителей частиц также основан на этом свойстве электромагнитного поля.Сверхпроводящие электромагниты отклоняют частицы от линейного движения, заставляя их двигаться по кругу.


Самое любопытное то, что сила Лоренца не подчиняется третьему закону Ньютона, который гласит, что каждое действие имеет свое собственное противодействие. Это связано с тем, что Исаак Ньютон считал, что любое взаимодействие на любом расстоянии происходит мгновенно, но это не так. На самом деле это происходит через поля. К счастью, затруднений удалось избежать, поскольку физикам удалось переработать третий закон в закон сохранения количества движения, что справедливо и для эффекта Лоуренса.

Формула силы Лоренца при наличии магнитного и электрического полей

Магнитное поле присутствует не только в постоянных магнитах, но и в любом проводнике электричества. Только в этом случае помимо магнитной составляющей в ней присутствует еще и электрическая составляющая. Однако даже в этом электромагнитном поле эффект Лоуренса продолжает испытывать влияние и определяется формулой:

где v — скорость электрически заряженной частицы, q — ее заряд, B и E — силы магнитного и электрического полей поля.

Единицы силы Лоренца

Как и большинство других физических величин, которые действуют на тело и изменяют его состояние, оно измеряется в ньютонах и обозначается буквой N.

Концепция напряженности электрического поля

Электромагнитное поле фактически состоит из двух половин — электрической и магнитной. Это точно близнецы, у которых все одинаково, но характер у них другой. А если присмотреться, можно заметить небольшие отличия во внешнем виде.


То же самое и с силовыми полями.Электрическое поле также имеет напряженность — векторную величину, которая является силовой характеристикой. Он воздействует на неподвижные в нем частицы. Сама по себе это не сила Лоренца, ее просто нужно учитывать при расчете воздействия на частицу в присутствии электрического и магнитного полей.

Напряженность электрического поля

Напряженность электрического поля влияет только на неподвижный заряд и определяется по формуле:

Единица измерения — Н / К или В / м.

Примеры задач

Задача 1

На заряд 0,005 Кл, который движется в магнитном поле с индукцией 0,3 Тл, действует сила Лоренца. Вычислите это, если скорость заряда составляет 200 м / с, и он движется под углом 450 к линиям магнитной индукции.

Задача 2

Определить скорость тела, которое имеет заряд и которое движется в магнитном поле с индукцией 2 Тл под углом 900. Величина, с которой поле действует на тело, составляет 32 Н, заряд тела равен 5 × 10-3 С.

Задача 3

Электрон движется в однородном магнитном поле под углом 900 к его силовым линиям. Величина, с которой поле действует на электрон, составляет 5 × 10-13 Н. Величина магнитной индукции составляет 0,05 Тл. Определите ускорение электрона.

ac = v2R = 6 × 10726,8 × 10-3 = 5 × 1017 мс2

Электродинамика оперирует понятиями, аналогию которым в обычном мире трудно найти. Но это вовсе не означает, что их невозможно понять.С помощью различных визуальных экспериментов и природных явлений процесс познания мира электричества может стать поистине захватывающим.

  • Определение силы Лоренца

    Определение силы Лоренца

    Сила Лоренца — это комбинация магнитной и электрической силы, действующей на точечный заряд, которая вызывается электромагнитными полями. Другими словами, сила Лоренца — это сила, действующая на любую заряженную частицу, которая падает в магнитном поле с определенной скоростью.Его величина зависит от величины магнитной индукции В , электрического заряда частицы q и скорости, с которой частица попадает в поле — В … О том, какая формула для расчета Лоренца сила, а также ее практическое значение в физике, читайте дальше.

    Немного истории

    Первые попытки описать электромагнитную силу были сделаны в 18 веке. Ученые Генри Кавендиш и Тобиас Мейер предположили, что сила на магнитных полюсах и электрически заряженных объектах подчиняется закону обратных квадратов.Однако экспериментальные доказательства этого факта не были полными и убедительными. Только в 1784 году Шарль Августин де Кулон с помощью своих торсионных весов смог окончательно доказать это предположение.

    В 1820 году физик Эрстед обнаружил, что на магнитную стрелку компаса действует вольт-ток, и Андре-Мари Ампер в том же году смог разработать формулу угловой зависимости между двумя элементами тока. Фактически эти открытия легли в основу современной концепции электрического и магнитного полей.Сама концепция получила дальнейшее развитие в теориях Майкла Фарадея, особенно в его концепции силовых линий. Лорд Кельвин и Джеймс Максвелл дополнили теории Фарадея подробными математическими описаниями. В частности, Максвелл создал так называемое «уравнение поля Максвелла» — систему дифференциальных и интегральных уравнений, описывающих электромагнитное поле и его связь с электрическими зарядами и токами в вакууме и сплошных средах.

    Дж. Дж. Томпсон был первым физиком, который попытался вывести из уравнения поля Максвелла электромагнитную силу, действующую на движущийся заряженный объект.В 1881 году он опубликовал свою формулу F = q / 2 v x B. Но из-за некоторых просчетов и неполного описания тока смещения она оказалась не совсем правильной.

    И, наконец, в 1895 году голландский ученый Хендрик Лоренц вывел правильную формулу, которая используется до сих пор и также носит его имя, а также сила, действующая на летящую частицу в магнитном поле, теперь называется «Сила Лоренца».

    Формула силы Лоренца

    Формула для расчета силы Лоренца выглядит следующим образом:

    Где q — электрический заряд частицы, V — ее скорость, а B — величина магнитной индукции частицы. магнитное поле.

    В этом случае поле B действует как сила, перпендикулярная направлению вектора скорости V нагрузки и направлению вектора B. Это можно проиллюстрировать на диаграмме:

    Правило левой руки позволяет физикам определять направление и возврат вектора магнитной (электродинамической) энергии. Представьте, что наша левая рука расположена таким образом, что линии магнитного поля направлены перпендикулярно внутренней поверхности руки (так, чтобы они проникали в руку), а все пальцы, кроме большого, указывают направление положительного тока. отклоненный большой палец указывает направление электродинамической силы, действующей на положительный заряд, помещенный в это поле.

    Вот так это будет выглядеть схематично.

    Существует также второй способ определения направления электромагнитной силы. Он заключается в размещении большого, указательного и среднего пальцев под прямым углом. В этом случае указательный палец покажет направление силовых линий магнитного поля, средний палец покажет направление движения тока, а большой палец покажет направление электродинамической силы.

    Применение силы Лоренца

    Сила Лоренца и ее вычисления находят свое практическое применение при создании как специальных научных инструментов — масс-спектрометров, которые служат для идентификации атомов и молекул, так и создания многих других устройств широкий спектр приложений.Устройства включают электродвигатели, громкоговорители и рельсовые пистолеты.

  • Основные законы динамики. Законы Ньютона — первый, второй, третий. Принцип относительности Галилея. Закон всемирного тяготения. Сила тяжести. Упругие силы. Вес. Силы трения — покой, скольжение, качение + трение в жидкостях и газах.
  • Кинематика. Базовые концепты. Равномерное прямолинейное движение. Равно ускоренное движение. Равномерное круговое движение. Справочная система. Траектория, перемещение, путь, уравнение движения, скорость, ускорение, соотношение между линейной и угловой скоростью.
  • Простые механизмы. Рычаг (рычаг первого класса и рычаг второго класса). Блок (фиксированный блок и подвижный блок). Наклонная плоскость. Гидравлический пресс. Золотое правило механики
  • Законы сохранения в механике. Механическая работа, мощность, энергия, закон сохранения количества движения, закон сохранения энергии, равновесие твердых тел
  • Круговое движение. Уравнение движения по окружности. Угловая скорость. Нормальный = центростремительное ускорение. Период, частота вращения (вращения).Соотношение линейной и угловой скоростей
  • Механические колебания. Свободные и вынужденные колебания. Гармонические колебания. Упругие колебания. Математический маятник. Преобразования энергии при гармонических колебаниях
  • Механические волны. Скорость и длина волны. Уравнение бегущей волны. Волновые явления (дифракция, интерференция …)
  • Гидромеханика и аэромеханика. Давление, гидростатическое давление. Закон Паскаля. Основное уравнение гидростатики. Сообщающиеся сосуды.Закон Архимеда. Условия купания тел. Поток жидкости. Закон Бернулли. Формула Торричели
  • Молекулярная физика. Основные положения ИКТ. Основные понятия и формулы. Идеальные газовые свойства. Основное уравнение МКТ. Температура. Уравнение состояния идеального газа. Уравнение Менделеева-Клиперона. Законы газа — изотерма, изобара, изохора
  • Волновая оптика. Корпускулярно-волновая теория света. Волновые свойства света. Рассеивание света. Легкие помехи. Принцип Гюйгенса-Френеля.Дифракция света. Поляризация света
  • Термодинамика. Внутренняя энергия. Работа. Количество тепла. Тепловые явления. Первый закон термодинамики. Применение первого закона термодинамики к различным процессам. Уравнение теплового баланса. Второй закон термодинамики. Тепловые двигатели
  • Электростатика. Базовые концепты. Электрический заряд. Закон сохранения электрического заряда. Закон Кулона. Принцип суперпозиции. Теория ближнего действия. Потенциал электрического поля.Конденсатор.
  • Постоянный электрический ток. Закон Ома для участка цепи. Работа и мощность постоянного тока. Закон Джоуля-Ленца. Закон Ома для полной цепи. Закон электролиза Фарадея. Электрические схемы — последовательное и параллельное соединение. Кирхгоф правил.
  • Колебания электромагнитные. Свободные и вынужденные электромагнитные колебания. Колебательный контур. Переменный электрический ток. Конденсатор в цепи переменного тока. Индуктор («соленоид») в цепи переменного тока.
  • Электромагнитные волны.Понятие об электромагнитной волне. Свойства электромагнитных волн. Волновые явления
  • Вы сейчас здесь: Магнитное поле. Вектор магнитной индукции. Правило буравчика. Закон Ампера и сила Ампера. Сила Лоренца. Правило левой руки. Электромагнитная индукция, магнитный поток, правило Ленца, закон электромагнитной индукции, самоиндукция, энергия магнитного поля
  • Квантовая физика. Гипотеза Планка. Явление фотоэлектрического эффекта.Уравнение Эйнштейна. Фотоны. Квантовые постулаты Бора.
  • Элементы теории относительности. Постулаты теории относительности. Относительность одновременности, расстояний, временных интервалов. Релятивистский закон сложения скоростей. Скорость в зависимости от массы. Основной закон релятивистской динамики …
  • Погрешности прямых и косвенных измерений. Абсолютная относительная ошибка. Систематические и случайные ошибки. Стандартное отклонение (ошибка). Таблица для определения погрешностей косвенных измерений различных функций.
  • Сила Лоренца называется силой, действующей из электромагнитного поля на движущийся электрический заряд. Нередко только магнитная составляющая этого поля называется силой Лоренца. Формула для определения:

    F = q (E + vB),

    где q — заряд частицы; E — напряженность электрического поля; B — магнитная индукция поля; v — скорость частиц.

    Сила Лоренца в принципе очень похожа на, разница в том, что последняя действует на весь проводник, который обычно является электрически нейтральным, а сила Лоренца описывает действие электромагнитного поля только для одного движущегося заряда .

    Характеризуется тем, что не изменяет скорость движения зарядов, а только влияет на вектор скорости, то есть способен изменять направление движения заряженных частиц.

    В природе сила Лоренца позволяет защитить Землю от воздействия космического излучения. Под его воздействием заряженные частицы, падающие на планету, отклоняются от прямой траектории из-за наличия магнитного поля Земли, вызывая полярное сияние.

    В технике сила Лоренца используется очень часто: во всех двигателях и генераторах, именно она приводит во вращение ротор под действием электромагнитного поля статора.

    Таким образом, в любых электродвигателях и электроприводах основным видом силы является лоренцевский. Кроме того, он используется в ускорителях частиц, а также в электронных пушках, которые ранее устанавливались в ламповых телевизорах. В ЭЛТ электроны, испускаемые пушкой, отклоняются под действием электромагнитного поля, которое возникает с участием силы Лоренца.

    Кроме того, эта сила используется в масс-спектрометрии и масс-электрографии для устройств, способных сортировать заряженные частицы в зависимости от их удельного заряда (отношения заряда к массе частицы). Это позволяет определять массу частицы с высокой точностью. Он также находит применение в другом оборудовании, например, в бесконтактном методе измерения расхода электропроводных жидких сред (расходомеры). Это очень важно, если жидкая среда имеет очень высокую температуру (расплав металлов, стекла и т. Д.).

    Магнитный момент равен. Экспериментальное определение магнитных моментов. Магнитный момент. Силы, действующие на магнитный момент и его энергию в магнитном поле

    МАГНИТНЫЙ МОМЕНТ — физическая величина, характеризующая магн. системные свойства заряжены. частицы (или отдельные частицы) и определяющие, наряду с другими мультипольными моментами (электрический дипольный момент, квадрупольный момент и т. д., см. Multipoli ) взаимодействие системы с доп.эль — магн. поля и другие подобные системы.

    По взглядам классика. , магн. поле создается движущимся электрическим током. … Хотя современный. теория не отвергает (и даже не предсказывает) существование частиц с магн. заряда ( магнитных монополей) , такие частицы еще не наблюдались экспериментально и отсутствуют в обычном веществе. Следовательно, элементарная характеристика магн. свойства оказывается именно магнитным m.Система, обладающая магнитным полем m. (осевой вектор) создает величину на больших расстояниях от системы. поле


    (- радиус-вектор точки наблюдения). Электрооборудование имеет похожую форму. поле диполя, состоящего из двух близко расположенных электриков. заряды противоположного знака. Однако в отличие от электрического. дипольный момент. М. М. Создается не точечной системой «магнит. Заряды», а электрической. токи, протекающие внутри системы. Если закрыт электр.ток течет в ограниченном объеме В , то создаваемая им М. м определяется ф-лой

    В простейшем случае замкнутый круговой ток I течет по плоскому витку площади s, а вектор М. м. направлен по правой нормали к повороту.

    Если ток создается стационарным движением точки электрического. зарядов с массами, имеющими скорости, то возникающая М. м., как следует из ф-лы (1), имеет вид


    , где под усреднением подразумевается микроскопическое.количества с течением времени. Поскольку векторное произведение в правой части пропорционально вектору момента числа движения частицы (предполагается, что скорость), то вклад деп. частиц в М. м. и по времени количество перемещений пропорционально:

    Соотношение сторон е / 2ц называется; эта величина характеризует универсальную связь между магн. и механические свойства заряда. частицы в классике. электродинамика.Однако движение элементарных носителей заряда в веществе (электронов) подчиняется законам, которые вносят коррективы в классические. рисунок. Помимо орбитальной механики. момент движения L электрон имеет внутренний механический. момент — , раскрутка … Суммарная М. м. Количество электрона равно сумме орбитальных М. м. (2) и спин М. м.

    Как видно из этой ф-лы (вытекающей из релятивистского уравнения Дирака для электрона), гиромагнетик.отношение для спина оказывается ровно вдвое больше, чем для орбитального углового момента. Особенность квантовой концепции магн. и механических моментов — это еще и тот факт, что векторы не могут иметь определенного направления в пространстве из-за некоммутативности операторов проекции этих векторов на координатную ось.

    Спин М. м. Плата. частицы, определяемые ф-лой (3), называются. нормальный, для электрона он равен магнетону бора. Однако опыт показывает, что величина электрона отличается от (3) на порядок (- постоянная тонкой структуры).Аналогичное дополнение под названием

    1. Магнитный момент — см. Магнетизм. Энциклопедический словарь Брокгауза и Ефрона
    2. магнитный момент — МАГНИТНЫЙ МОМЕНТ — это векторная величина, характеризующая магн. свойства вещества. Мм. обладают все элементарные частицы и образованные из них системы (атомные ядра, атомы, молекулы). Мм. атомы, молекулы и др. Химическая энциклопедия
    3. МАГНИТНЫЙ МОМЕНТ — Основное значение, характеризующее магн. свойства островов.Источник магнетизма (М. м.), Согласно класс. теория е-магн. явления, явл. макро- и микро (атомные) — электрические. токи. Элем. источником магнетизма считается замкнутый ток. Из опыта и классики. Физический энциклопедический словарь
    4. МАГНИТНЫЙ МОМЕНТ — МАГНИТНЫЙ МОМЕНТ, измеряющий силу постоянного магнита или токоведущей катушки. Это максимальная сила вращения (крутящий момент), приложенная к магниту, катушке или электрическому заряду в МАГНИТНОМ ПОЛЕ, деленная на напряженность поля.Заряженные частицы и атомные ядра также обладают магнитным моментом. Научно-технический словарь
    5. МАГНИТНЫЙ МОМЕНТ — МАГНИТНЫЙ МОМЕНТ — векторная величина, характеризующая вещество как источник магнитного поля … Макроскопический магнитный момент создается замкнутыми электрическими токами и упорядоченно ориентированными магнитными моментами атомных частиц. Большой энциклопедический словарь

    Кикоин А.К. Магнитный момент тока // Квант. — 1986. — №3. — С. 22-23.

    По специальному соглашению с редколлегией и редакцией журнала «Квант»

    Из курса физики девятого класса (Физика 9, § 88) известно, что прямой провод длиной l с током I , если его поместить в однородное магнитное поле с индукцией \ (~ \ vec B \ ) действует сила \ (~ \ vec F \), равная по величине

    \ (~ F = BIl \ sin \ alpha \),

    , где α — угол между направлением тока и вектором магнитной индукции.Эта сила направлена ​​перпендикулярно как полю, так и току (согласно правилу левой руки).

    Прямой проводник — это только часть электрической цепи, поскольку электрический ток всегда замкнут. А как магнитное поле влияет на замкнутый ток, а точнее на замкнутый контур с током?

    На фиг.1 в качестве примера показан контур в виде прямоугольной рамки со сторонами a и b , по которой ток течет в направлении, указанном стрелками I .

    Рамка помещается в однородное магнитное поле с индукцией \ (~ \ vec B \) так, чтобы в начальный момент вектор \ (~ \ vec B \) лежал в плоскости рамки и был параллелен ее двум сторонам. . Рассматривая каждую сторону рамы отдельно, обнаружим, что на боковых сторонах (длина a ) действуют силы, равные по модулю F = BIa и направленные в противоположные стороны. На две другие стороны силы не действуют (для них sin α = 0).Каждая из сил F вокруг оси, проходящей через середины верхней и нижней сторон рамы, создает момент силы (крутящий момент), равный \ (~ \ frac (BIab) (2) \) (\ ( ~ \ frac (b) (2) \) — сила плеча). Знаки моментов одинаковы (обе силы вращают раму в одном направлении), поэтому суммарный крутящий момент M равен BIab , или, поскольку произведение ab равно площади S каркаса,

    \ (~ M = BIab = BIS \).

    Под действием этого момента рамка начнет вращаться (если смотреть сверху, то по часовой стрелке) и будет вращаться, пока не станет своей плоскостью, перпендикулярной вектору индукции \ (~ \ vec B \) (рис. 2) .

    В этом положении сумма сил и сумма моментов сил равны нулю, и рама находится в состоянии устойчивого равновесия. (Фактически, рамка не остановится немедленно — некоторое время она будет колебаться вокруг своего положения равновесия.)

    Легко показать (сделай сам), что в любом промежуточном положении, когда нормаль к плоскости контура составляет произвольный угол β с индукцией магнитного поля, крутящий момент

    \ (~ M = BIS \ sin \ beta \).

    Из этого выражения видно, что при заданном значении индукции поля и в определенном положении цепи с током крутящий момент зависит только от произведения площади цепи S на силу тока Я в нем.Величина IS и называется магнитным моментом токовой петли. Точнее, IS — это модуль вектора магнитного момента. Причем этот вектор направлен перпендикулярно плоскости контура и притом так, что если мысленно повернуть большой палец в направлении тока в петле, то направление поступательного движения большого пальца будет указывать направление движения большого пальца. магнитный момент. Например, магнитный момент контура, изображенного на рисунках 1 и 2, направлен от нас за плоскость страницы.Магнитный момент измеряется в А · м 2.

    Теперь мы можем сказать, что цепь с током в однородном магнитном поле настроена так, что ее магнитный момент «смотрит» в направлении поля, вызвавшего ее вращение.

    Известно, что не только цепи с током обладают свойством создавать собственное магнитное поле и вращаться во внешнем поле. Такие же свойства наблюдаются и у намагниченного стержня, например, у стрелки компаса.

    Еще в 1820 году замечательный французский физик Ампер высказал мысль, что сходство поведения магнита и цепи с током объясняется тем, что в частицах магнита существуют замкнутые токи.Теперь известно, что в атомах и молекулах действительно есть мельчайшие электрические токи, связанные с движением электронов по их орбитам вокруг ядер. Из-за этого атомы и молекулы многих веществ, например парамагнетиков, обладают магнитными моментами. Вращение этих моментов во внешнем магнитном поле приводит к намагничиванию парамагнитных веществ.

    Выяснилось другое. Все частицы, составляющие атом, также обладают магнитными моментами, которые совершенно не связаны ни с какими движениями зарядов, то есть с токами.Для них магнитный момент — это такое же «врожденное» качество, как заряд, масса и т. Д. Магнитным моментом обладает даже частица, не имеющая электрического заряда — нейтрон, составная часть атомных ядер. Следовательно, атомные ядра тоже обладают магнитным моментом.

    Таким образом, магнитный момент — одно из важнейших понятий физики.

    ; замкнутый ток считается элементарным источником магнетизма). Элементарные частицы, атомные ядра, электронные оболочки атомов и молекул обладают магнитными свойствами.Магнитный момент элементарных частиц (электронов, протонов, нейтронов и др.), Как показывает квантовая механика, обусловлен существованием их собственного механического момента — спина.

    Магнитный момент
    м → знак равно I S N → (\ Displaystyle (\ vec (m)) = IS (\ vec (п)))
    Размер л 2 я
    SI ⋅ 2
    количество векторов

    Магнитный момент измеряется в ⋅ 2, или в Вб * м, или Дж / Тл (СИ), или эрг / Гс (СГС), 1 эрг / Г = 10 −3 Дж / Тл.Специфической единицей элементарного магнитного момента является магнетон Бора.

    Формулы для расчета магнитного момента

    В случае плоского контура с поражением электрическим током магнитный момент рассчитывается как

    m = IS N (\ Displaystyle \ mathbf (m) = IS \ mathbf (n)),

    где I (\ displaystyle I) — сила тока в цепи, S (\ displaystyle S) — площадь контура, n (\ displaystyle \ mathbf (n)) — единичный вектор нормали к контурной плоскости. Направление магнитного момента обычно определяется в соответствии с правилом подвеса: если повернуть ручку подвеса в направлении тока, направление магнитного момента будет совпадать с направлением поступательного движения подвеса.

    Для произвольного замкнутого контура магнитный момент находится по формуле:

    м знак равно я 2 ∮ ⁡ [р, dl] (\ displaystyle \ mathbf (m) = (I \ over 2) \ oint [\ mathbf (r), d \ mathbf (l)]),

    где r (\ displaystyle \ mathbf (r)) — радиус-вектор, проведенный от начала координат до элемента длины контура dl (\ displaystyle d \ mathbf (l)).

    В общем случае произвольного распределения токов в среде:

    м знак равно 1 2 ∫ В [р, J] d В (\ Displaystyle \ mathbf (m) = (1 \ над 2) \ int \ limits _ (V) [\ mathbf (r), \ mathbf (j)] dV ),

    , где j (\ displaystyle \ mathbf (j)) —

    Любое вещество.Источником образования магнетизма, согласно классической электромагнитной теории, являются микротоки, возникающие при движении электрона по его орбите. Магнитный момент — непременное свойство всех без исключения ядер, электронных оболочек и молекул атомов.

    Магнетизм, присущий всем элементарным частицам, в зависимости от наличия у них механического момента, называется спином (собственный механический импульс квантовой природы). Магнитные свойства атомного ядра складываются из спиновых моментов составных частей ядра — протонов и нейтронов.Электронные оболочки (внутриатомные орбиты) также обладают магнитным моментом, который представляет собой сумму магнитных моментов электронов на них.

    Другими словами, магнитные моменты элементарных частиц возникают из-за внутриатомного квантово-механического эффекта, известного как спиновый момент. Этот эффект аналогичен угловому моменту вращения вокруг собственной центральной оси. Спиновый момент измеряется постоянной Планка, фундаментальной постоянной квантовой теории.

    Все нейтроны, электроны и протоны, из которых фактически состоит атом, согласно Планку, имеют спин, равный ½.В структуре атома электроны, вращающиеся вокруг ядра, помимо спинового момента, имеют еще и орбитальный угловой момент. Ядро, хотя и занимает статическое положение, также имеет угловой момент, который создается эффектом ядерного спина.

    Магнитное поле, которое генерирует атомный магнитный момент, определяется различными формами этого углового момента. Наиболее заметный вклад в творчество вносит спиновой эффект. Согласно принципу Паули, согласно которому два идентичных электрона не могут одновременно находиться в одном квантовом состоянии, связанные электроны сливаются, а их спиновые моменты приобретают диаметрально противоположные проекции.В этом случае магнитный момент электрона уменьшается, что снижает магнитные свойства всей конструкции. У некоторых элементов, имеющих четное число электронов, этот момент уменьшается до нуля, и вещества перестают иметь магнитные свойства. Таким образом, магнитный момент отдельных элементарных частиц оказывает непосредственное влияние на магнитные свойства всей ядерно-атомной системы.

    Ферромагнитные элементы с нечетным числом электронов всегда будут иметь ненулевой магнетизм из-за неспаренного электрона.В таких элементах соседние орбитали перекрываются, и все спиновые моменты неспаренных электронов принимают одинаковую ориентацию в пространстве, что приводит к достижению состояния с наименьшей энергией. Этот процесс называется обменным взаимодействием.

    При таком совмещении магнитных моментов ферромагнитных атомов возникает магнитное поле. А парамагнитные элементы, состоящие из атомов с разориентированными магнитными моментами, не имеют собственного магнитного поля. Но если на них воздействовать внешним источником магнетизма, то магнитные моменты атомов выровняются, и эти элементы также приобретут магнитные свойства.

    На очереди

    ветеранов, которые получат отсроченные стимулирующие чеки

    Вот что вам нужно знать:

    Более 25 миллионов американцев с низкими доходами, чьи стимулирующие выплаты были отложены, наконец, получили их в среду. И одна группа, все еще ожидающая — некоторые ветераны и их бенефициары — могут рассчитывать на получение выплат на следующей неделе, сообщила Налоговая служба.

    Выплаты произведены группами, первая партия поступила на счета 17 марта.Но многие люди, которые получают государственные пособия и не достигают порогового уровня дохода, необходимого для подачи налоговой декларации, не получили денег, потому что I.R.S. у них не было файлов, необходимых для обработки платежей. В их число входили американцы, получающие пособия по социальному обеспечению, дополнительному страховому доходу, пенсионному совету железной дороги и по делам ветеранов.

    В среду было получено 25 миллионов просроченных платежей на сумму около 36 миллиардов долларов. Самый большой блок, или 26 миллиардов долларов, достался более чем 19 миллионам получателей социального обеспечения, включая тех, кто получает пенсионные пособия, пособия по случаю потери кормильца или инвалидности.Еще три миллиона платежей на сумму почти 5 миллиардов долларов были отправлены получателям дополнительного дохода по безопасности. И около 85 000 выплат, или 119 миллионов долларов, пошли получателям пенсионного совета Железной дороги.

    Некоторые получатели помощи по делам ветеранов все еще ждут. Но до тех пор, пока не возникнет никаких проблем, ветераны, не подающие документы, и их бенефициары, которые получают компенсацию и пенсионные выплаты, могут ожидать, что их деньги появятся 14 апреля. Статус их выплаты должен быть доступен в I.Инструмент Get My Payment от R.S. в субботу или воскресенье. Пакет

    в среду также включал более одного миллиона платежей американцам, которые уже получили один в марте, но имели право на получение новой или большей суммы на основании своей налоговой декларации за 2020 год. Эти так называемые дополнительные выплаты были оценены более чем в 2 миллиарда долларов.

    Круизный лайнер Carnival пришвартовался в прошлом году в Лонг-Бич, штат Калифорния. Круизная линия пригрозила вывести свои суда за пределы портов США. Фото… Люси Николсон / Reuters

    Carnival Cruise Line, крупнейший круизный оператор в Соединенных Штатах, сказал в среду, что было оптимистично, что некоторые из его U.Линии на базе S. будут запущены к июлю.

    Объявление было сделано на следующий день после того, как компания была вынуждена отменить свои рейсы до 30 июня и пригрозила вывести свои корабли из портов США. Отрасль изо всех сил пыталась возобновить работу через год после того, как пандемия остановила круизы.

    «Хотя у нас не было планов по перемещению судов Carnival Cruise Line за пределы наших портов базирования в США, у нас может не быть другого выбора, кроме как сделать это, чтобы возобновить нашу работу», — сказала Кристин Даффи, президент Carnival Cruise Line. в заявлении, опубликованном во вторник на сайте компании.

    Центры по контролю и профилактике заболеваний рекомендуют людям избегать круизов по всему миру из-за высокого риска заражения коронавирусом на борту лайнера. В пятницу он выпустил условные заказы на круизы для круизных линий, включая плановое тестирование членов экипажа и моделирование рейсов для отработки процедур безопасности.

    “C.D.C. стремится работать с круизной индустрией и партнерами по морским портам, чтобы возобновить круизы, когда это будет безопасно », — говорится в заявлении агентства.

    Гости карнавала получили возможность получить кредит или полный возврат средств за отмененные круизы.

    Disney Cruise Line сообщила во вторник, что также приостановит отправления до июня после проверки C.D.C. руководство. Он также отменил рейсы в Европу до 18 сентября.

    Клиенты, похоже, хотят снова отправиться в плавание. Объем бронирования будущих круизов Carnival в первом квартале 2021 года был примерно на 90% выше, чем в предыдущем квартале, «что отражает как значительный отложенный спрос, так и долгосрочный потенциал круизов», — сказал Арнольд Дональд, исполнительный директор Carnival Corporation. , говорится в заявлении материнской компании круизной линии в среду.

    Carnival сообщил, что количество бронирований на 2022 год превысило количество бронирований в 2019 году, добавив, что шесть из девяти его брендов, как ожидается, возобновят ограниченные гостевые круизы к лету.

    Компания сообщила о чистом убытке в размере 2 миллиардов долларов за первый квартал 2021 года.

    Чиновники Федеральной резервной системы воодушевились восстановлением экономики на своем заседании в прошлом месяце, как показали опубликованные в среду протоколы, но инфляция и рынок труда по-прежнему сильно отстают. своих целей, а политики продолжали видеть «повышенную» неопределенность в отношении перспектив роста.

    «Участники согласились, что развитие экономики будет в значительной степени зависеть от распространения вируса, включая прогресс в вакцинации», — говорится в отчете о встрече 16-17 марта. На этом заседании ФРС оставила процентные ставки неизменными на уровне, близком к нулю, и продолжила покупать облигации со скоростью 120 миллиардов долларов в месяц — две политики, направленные на стимулирование расходов за счет удержания заимствований на низком уровне.

    В прошлом году ФРС предприняла радикальные меры для поддержки экономики, пострадавшей от пандемии, и теперь инвесторы следят за любым намеком на то, когда она может начать откатывать некоторые из этих мер политики.Поскольку ожидается, что чиновники замедлят покупку своих облигаций, прежде чем они поднимут процентные ставки, инвесторы внимательно следят за любыми признаками того, что покупки могут прекратиться.

    Должностные лица ФРС заявили, что хотят увидеть «существенный дальнейший прогресс» в достижении своих целей в области занятости и инфляции, прежде чем замедлять программу, хотя они не определили, что можно было бы квалифицировать как существенное.

    Должностные лица в прошлом месяце «отметили, что, вероятно, пройдет некоторое время, прежде чем будет достигнут существенный дальнейший прогресс в достижении целей комитета по максимальной занятости и стабильности цен», — говорится в протоколе, добавив, что было бы важно сообщить «хорошо» заранее. о внесении каких-либо изменений в программу облигаций.

    Когда дело доходит до процентной ставки, директивные органы ФРС были более четкими. Они заявили, что ФРС будет поддерживать ставку около нуля до тех пор, пока инфляция не превысит 2 процента и, похоже, будет оставаться на более высоком уровне в течение некоторого времени и до тех пор, пока рынок труда не вернется к полной занятости.

    После мартовского заседания ФРС вакцинация в Соединенных Штатах продолжала неуклонно расти, а мартовский отчет о занятости показал, что работодатели повторно нанимают на работу по мере возобновления экономики штатов и местных экономик.Тем не менее, по сравнению с февралем 2020 года, когда началась пандемия, не хватает около 8,4 миллиона рабочих мест.

    Несколько официальных лиц на заседании ФРС отметили, что недавно принятая программа стимулирования экономики на 1,9 триллиона долларов «может ускорить восстановление, что может помочь ограничить долгосрочный ущерб рынкам труда, вызванный пандемией», говорится в протоколе.

    Но центральный банк не беспокоит безудержная инфляция, поскольку правительство тратит.

    Хотя многие политики ФРС ожидают роста инфляции в этом году, отчасти по мере открытия экономики и скачка предложения, чтобы не отставать от спроса, «участники в целом ожидали, что годовые показатели инфляции снизятся в следующем году.И они охарактеризовали риски для прогноза инфляции — в основном шансы на более высокие или более низкие, чем ожидалось, цифры — как «в целом сбалансированные».

    Профсоюзы, представляющие сотрудников двух известных подкастинговых компаний, принадлежащих Spotify, гиганту аудиостриминга, объявили в среду, что они ратифицировали свои первые трудовые контракты.

    Самый крупный из двух профсоюзов, насчитывающий 65 сотрудников, находится на сайте The Ringer, посвященном спорту и поп-культуре, с сетью подкастинга. Во втором профсоюзе, в компании Gimlet Media, занимающейся производством подкастов, работает чуть менее 50 сотрудников.Эти две группы были одними из первых в индустрии подкастинга, которые объединились в профсоюзы, и обе представлены Гильдией писателей Америки, Восток.

    Лоуэлл Петерсон, исполнительный директор гильдии, сказал, что контракты показали, что писатели, продюсеры и редакторы компаний «приносят огромную пользу основным платформам, для которых они создают контент».

    Контракты устанавливают минимальную базовую заработную плату в размере 57 000 долларов для членов профсоюзов в The Ringer и 73 000 долларов в Gimlet Media, ежегодное увеличение заработной платы не менее чем на 2 процента и минимум 11 недель выходного пособия.

    Соглашения включают положения, ограничивающие использование подрядчиков и позволяющие рабочим получать титулы, отражающие их трудовой стаж.

    Обе компании создадут комитеты по разнообразию, в состав которых войдут менеджеры и члены профсоюзов, и потребуют, чтобы по крайней мере половина кандидатов, серьезно рассматриваемых на профсоюзные должности, открытые для посторонних, были из недопредставленных групп, таких как расовые меньшинства или люди с ограниченными возможностями.

    The Ringer и Gimlet Media в прошлом году занимались внутренней борьбой, связанной с гонкой.Сотрудники The Ringer жаловались на нехватку чернокожих писателей и редакторов после того, как основатель компании Билл Симмонс устроил подкаст, в котором коллега неуклюже обсудил последствия убийства Джорджа Флойда и похвалил г-на Симмонса за приверженность разнообразию.

    В Gimlet компания недавно отменила последние два эпизода четырехсерийного сериала о расовом неравенстве в журнале Bon Appétit о еде после того, как сотрудники жаловались, что Gimlet сам страдает от подобных проблем.

    Сотрудники обеих компаний объединились в профсоюзы в 2019 году, и переговоры по контракту иногда были спорными. Руководство отказалось предоставить основание для главного приоритета профсоюзов — прав на работы, которые создают сценаристы и подкастеры, которые компании сохранят, — но профсоюзы, тем не менее, единогласно ратифицировали контракты, согласно гильдии писателей.

    «Мы начали этот процесс с целью улучшения условий труда и компенсации в компании, особенно для наших низкооплачиваемых членов», — говорится в заявлении Союза Рингеров.«Мы очень рады, что достигли этой цели с помощью этого контракта».

    Spotify не сразу ответил на запрос о комментарии.

    «Наши налоговые поступления уже находятся на самом низком уровне за многие поколения», — заявила министр финансов Джанет Л. Йеллен. «Если они продолжат падать ниже, у нас будет меньше денег для инвестирования в дороги, мосты, широкополосную связь и исследования и разработки». Кредит … Аль Драго для The New York Times

    Администрация Байдена обнародовала свой план по пересмотру корпоративного налогового кодекса на В среду он предлагает ряд предложений, которые потребуют от крупных компаний платить более высокие налоги, чтобы помочь финансировать экономическую повестку дня Белого дома.

    План, если он будет принят, принесет 2,5 триллиона долларов дохода за 15 лет. Это будет сделано путем внесения серьезных изменений в американские компании, которые уже давно используют особенности налогового кодекса, позволяющие им снижать или устранять свои налоговые обязательства, часто за счет перевода прибыли за границу. План также включает усилия по борьбе с изменением климата, предлагая заменить субсидии на ископаемое топливо налоговыми льготами, которые способствуют производству чистой энергии.

    Некоторые корпорации выразили готовность платить больше в виде налогов, но общий объем предложения, вероятно, вызовет негативную реакцию со стороны бизнес-сообщества, которое в течение многих лет извлекало выгоду из лазеек в налоговом кодексе и смягченного подхода к обеспечению соблюдения.

    Министр финансов Джанет Л. Йеллен заявила во время брифинга с журналистами в среду, что этот план положит конец глобальной «гонке на дно» корпоративного налогообложения.

    «Наши налоговые поступления уже находятся на самом низком уровне за многие поколения», — сказала г-жа Йеллен. «Если они продолжат падать ниже, у нас будет меньше денег для инвестирования в дороги, мосты, широкополосную связь и НИОКР».

    План, объявленный Министерством финансов, повысит ставку корпоративного налога с 21 процента до 28 процентов.Администрация заявила, что повышение приведет к более близкому соответствию ставки корпоративного налога в Америке с другими странами с развитой экономикой и уменьшит неравенство. Она также останется ниже, чем была до снижения налогов Трампом в 2017 году, когда ставка составляла 35 процентов.

    Белый дом также предложил значительные изменения в нескольких международных налоговых положениях, включенных в снижение налогов Трампа, которое администрация Байдена охарактеризовала в отчете как политику, которая ставит «Америку на последнее место», принося пользу иностранцам.Одним из самых больших изменений будет удвоение фактического глобального минимального налога до 21 процента и его ужесточение, чтобы вынудить компании платить налог на более широкий диапазон доходов в разных странах.

    Это, в частности, вызвало обеспокоенность в деловом сообществе, поскольку Джошуа Болтен, исполнительный директор Business Roundtable, заявил в заявлении на этой неделе, что это «угрожает поставить США в невыгодное положение с точки зрения конкуренции».

    Однако некоторые компании выразили готовность принять новые предложения в среду.

    Джон Циммер, президент и соучредитель Lyft, сказал CNN, что поддерживает предложенную Байденом ставку корпоративного налога в размере 28 процентов.

    «Я считаю важным снова инвестировать в страну и экономику», — сказал г-н Циммер.

    Администрация Байдена также дала понять, что это предложение было чем-то вроде вступительной заявки и что будет место для переговоров.

    Министр торговли Джина Раймондо призвала законодателей в среду не отказываться от плана, предложив им провести «обсуждение» — даже несмотря на то, что она предположила, что основные параметры предложения останутся в силе.

    «Мы хотим компромисса, — сказала она во время брифинга в Белом доме. «Чего мы не можем сделать, и чего я умоляю бизнес-сообщество, так это сказать:« Нам не нравится 28 лет. Мы уходим. Мы не обсуждаем ». Это недопустимо».

    План также отменит положения, введенные во время администрации Трампа, которые, по словам администрации Байдена, не смогли сдержать перемещение прибыли и корпоративные инверсии, которые предполагают слияние американской компании с иностранной фирмой и превращения ее в ее дочернюю компанию, что фактически приводит к перемещению ее штаб-квартиры за границу. для налоговых целей.Он заменит их более жесткими антиинверсионными правилами и более строгими штрафами за так называемое лишение прибыли.

    План не полностью сосредоточен на международной стороне корпоративного налогового кодекса. Он пытается расправиться с крупными прибыльными компаниями, которые платят небольшие налоги на прибыль или совсем не платят, но сообщают о больших прибылях своей «балансовой стоимостью». Чтобы сократить это неравенство, компаниям придется платить минимальный налог в размере 15 процентов с бухгалтерской прибыли, о которой компании сообщают инвесторам и которая часто используется для оценки выплат акционерам и руководству.

    VideoPresident Байден сказал, что готов пойти на компромисс в своем предложении по инфраструктуре на 2,3 триллиона долларов, но отверг критиков, которые утверждали, что часть плана не является необходимой инфраструктурой, включая расширение широкополосного Интернета. CreditCredit … Амр Альфики / The New York Times

    Президент Байден в среду заявил о своей готовности к «добросовестным переговорам» по предложению об инфраструктуре на сумму 2,3 триллиона долларов, но прямо предупредил республиканских оппонентов плана, что он «не будет готов ничего не делать.

    Г-н Байден выступил против критиков, которые утверждали, что его обширный план содержит элементы — такие как ремонт больниц для ветеранов, расширение широкополосного интернета и программы борьбы с бедностью — которые не соответствуют традиционному определению инфраструктуры.

    «Автоматически говорить, что единственное, что является инфраструктурой, — это шоссе, мост или что-то еще, это просто нерационально», — сказал г-н Байден, который призвал республиканцев спросить у рабочего класса американцев, «какая инфраструктура им нужна, чтобы построить лучшую инфраструктуру. жизнь, чтобы иметь возможность немного подышать », вместо того, чтобы сразу же отвергнуть его предложение.

    «Я не знаю, почему мы этого не понимаем», — добавил г-н Байден в сопровождении вице-президента Камалы Харрис, выступая с речами в административном здании Эйзенхауэра, неоднократно отклоняясь от сценария, чтобы выразить страстную, временами раздраженная мольба о поддержке.

    Речь г-на Байдена была явно нацелена на республиканцев в Конгрессе во главе с сенатором Митчем МакКоннеллом от Кентукки, лидером меньшинства, которые выразили почти единодушное несогласие с планом.

    Но он также нацелился на красных и колеблющихся избирателей штата, которые поддерживают проекты в своих общинах, и разговаривал с умеренными демократами, такими как сенатор Джо Манчин III от Западной Вирджинии, которые предположили, что они могут согласиться на повышение корпоративного налога, но не такой же большой, как 28 процентов Mr.Байден сделал предложение. Текущая ставка — 21 процент.

    На вопрос, готов ли он пойти на компромисс по корпоративной ставке в своем плане — возможно, до 25 процентов — г-н Байден ответил: «Я готов вести переговоры», добавив, что он «широко открыт» для новых предложений, которые принесут прибыль. за его план.

    «Дебаты приветствуются, компромисс неизбежен, изменения неизбежны», — сказал он. «В следующие несколько недель мы с вице-президентом встретимся с республиканцами и демократами, чтобы услышать мнение всех. И мы будем слушать, будем открыты для хороших идей и добросовестных переговоров.Но вот к чему мы не будем открыты: мы не будем открыты для безделья ».

    Демократов на Капитолийском холме поддержало в понедельник постановление парламентария Сената, согласно которому демократы могут использовать ускоренный процесс согласования бюджета во второй раз в этом финансовом году. Постановление означает, что демократы могут по существу повторно открыть бюджетный план, принятый ими в феврале, и добавить директивы для введения в действие пакета инфраструктуры или других инициатив. Если они решат использовать этот ход, это защитит их от пирата, для преодоления которого требуется 60 голосов.

    Представители министерства финансов заявили в среду, что полный налоговый план г-на Байдена, который также исключает налоговые субсидии для компаний, работающих на ископаемом топливе, принесет 2,5 триллиона долларов новых доходов в течение следующих 15 лет.

    Непартийная модель бюджета Пенна Уортона, разработанная в Университете Пенсильвании, оценила в среду, что налоговые планы Байдена позволят собрать 2,1 триллиона долларов в течение десятилетия. По оценкам аналитиков группы, за десять лет в рамках этого плана будет израсходовано 2,7 триллиона долларов, а программы, в которые он инвестирует, помогут экономике функционировать более продуктивно.

    Но они подсчитали, что сочетание увеличения налогов и дополнительного государственного долга, понесенного планом, немного замедлит экономический рост, в результате чего в 2050 году экономика станет меньше на 0,8 процента, чем она была бы в противном случае.

    Представители министерства финансов заявили в среду, что они все еще рассматривают анализ, но не согласны с его выводом, настаивая на том, что планы г-на Байдена будут способствовать росту. Цель

    сказала, что ее приверженность добавила к своим другим движениям улучшить расовую справедливость в прошлом году.Кредит … Кендрик Бринсон для The New York Times

    Target потратит более 2 миллиардов долларов на предприятия, принадлежащие черным, к 2025 году, объявил он в среду, присоединившись к растущему списку розничных торговцев, которые пообещали увеличить свою экономическую поддержку таких компаний в стремление продвинуть расовое равенство в Соединенных Штатах.

    Target, базирующаяся в Миннеаполисе, добавит больше продуктов от компаний, принадлежащих черным предпринимателям, будет тратить больше средств на маркетинговые агентства, принадлежащие черным, и строительные компании, а также представит новые ресурсы, которые помогут поставщикам, принадлежащим черным, ориентироваться в процессе создания продуктов для — говорится в сообщении компании.

    После прошлогодних протестов против жестокости полиции волна американских розничных торговцев, от Sephora до Macy’s, взяла на себя обязательство тратить больше денег на предприятия, принадлежащие черным. Многие из них присоединились к движению, известному как «Клятва 15 процентов», которое поддерживает выделение достаточного количества места на полках для предприятий, принадлежащих чернокожим, чтобы соответствовать процентной доле афроамериканцев в национальном населении.

    Объявление Target, похоже, не связано с этим обещанием. Он сказал, что его приверженность усилила другие инициативы по обеспечению расового равенства и социальной справедливости в прошлом году, включая усилия по улучшению представительства среди его рабочей силы.

    Ежегодное письмо, которое Джейми Даймон, управляющий JPMorgan Chase, опубликовал в среду, было на 66 страницах самым длинным за все время. Кредит … Jeenah Moon / Reuters

    Годовое письмо к акционерам генерального директора JPMorgan Chase Джейми Даймона было опубликовано рано в среду. Письмо, которое широко читают на Уолл-стрит, не только представляет собой обзор бизнеса банка, но и охватывает мысли г-на Даймона по всему, от уроков лидерства до предписаний государственной политики.

    «У.Южная экономика, скорее всего, будет процветать ». Сочетание избыточных сбережений, дефицита расходов, вакцинации и «эйфории по окончании пандемии», — писал г-н Даймон, — может вызвать бум, который «легко может длится до 2023 года». Это могло оправдать высокую оценку акций, но не цену долга США, учитывая, что «огромное предложение» скоро появится на рынке. По его словам, существует вероятность того, что рост инфляции будет «более чем временным», что вынудит Федеральный резерв агрессивно повышать процентные ставки. «Быстрое повышение ставок для компенсации перегрева экономики — типичная причина рецессии», — писал он, но надеется на «сценарий Златовласки», предусматривающий быстрый рост, мягкое повышение инфляции и умеренное повышение процентных ставок.

    «Банки играют все меньшую роль в финансовой системе». Г-н Даймон сослался на конкуренцию со стороны уже крупной теневой банковской системы и финтех-компаний, а также «Amazon, Apple, Facebook, Google, а теперь и Walmart». Он утверждал, что этих небанковских конкурентов следует более строго регулировать; По его словам, их рост «частично стал возможным» благодаря игнорированию банковских правил. А когда дело доходит до ужесточения регулирования крупных банков, писал он, «затраты экономики на наличие отказоустойчивых банков могут не окупаться.

    «Руководители Китая считают, что Америка находится в упадке». Соединенные Штаты и раньше сталкивались с тяжелыми временами, но сегодня «китайцы видят Америку, которая теряет свои позиции в технологиях, инфраструктуре и образовании, — нацию, раздираемую и искалеченную политикой, а также расовым неравенством и неравенством доходов — и страну, неспособную согласованно координировать государственную политику (фискальную, денежно-кредитную, промышленную, регулирующую) для достижения национальных целей », — написал он. «К сожалению, в последнее время в этом есть много правды.»

    « Решение не так просто, как отказ от ископаемого топлива ». Решение проблемы изменения климата не означает «отказ от компаний, производящих и использующих ископаемое топливо», — писал г-н Даймон, — а работа с ними для уменьшения их воздействия на окружающую среду. Он видит «огромные возможности в устойчивых и низкоуглеродных технологиях и предприятиях» и планирует оценивать прогресс клиентов в соответствии с сокращением углеродоемкости — выбросов на единицу продукции — с поправкой на такие факторы, как размер.

    Другие примечательные новости (и просмотры) из письма:

    • При более широком распространении удаленной работы JPMorgan может потребоваться только 60 рабочих мест на каждые 100 сотрудников. «Это значительно снизит нашу потребность в недвижимости», — написал г-н Даймон.

    • JPMorgan тратит более 600 миллионов долларов в год на кибербезопасность.

    • Г-н Даймон процитировал налоговые лазейки, без которых, по его мнению, Соединенные Штаты могли бы обойтись: начисленные проценты; налоговые льготы для гоночных автомобилей, частных самолетов и скачек; и льгота по налогу на охрану земель для полей для гольфа.

    Это было самое длинное письмо г-на Даймона на данный момент: 35 000 слов на 66 страницах. Неуклонно расширяющиеся письма — за исключением более короткого выпуска в прошлом году, через несколько недель после того, как г-н Даймон перенес экстренную операцию на сердце — можно рассматривать как отражение круга проблем, которые высшие руководители теперь ожидают или вынуждены решать.

    Сенатор Берни Сандерс выступил на митинге в Алабаме 26 марта в поддержку акции профсоюзов на складе Amazon. Кредит … Благотворительность Рашель для The New York Times

    Голосование на выборах профсоюзов на складе Amazon в Бессемере, штат Алабама., закончился 29 марта, а подсчет начался на следующий день, но результат пока неизвестен. В чем дело? Дело не столько в количестве бюллетеней, сколько в их подсчете.

    Ставки высоки как для Amazon, так и для рабочего движения. Прогрессивные лидеры, такие как сенатор Берни Сандерс, независимый представитель штата Вермонт, утверждали, что победа профсоюза, первая на предприятии Amazon в Соединенных Штатах, может вдохновить рабочих в других странах на объединение в профсоюзы. А Amazon сталкивается с повышенным вниманием к своей рыночной силе и трудовой деятельности.

    Несмотря на важность, право голоса имела лишь небольшая часть сотрудников Amazon. Около 5800 рабочих имели право отправить свои бюллетени по почте в Бирмингемский офис Национального совета по трудовым отношениям. Подсчет каждого голоса включает два конверта: один с идентификацией рабочих, а внутри него другой запечатанный конверт с анонимным бюллетенем. Обработка их была кропотливым процессом:

    • На частной видеоконференции сотрудник N.L.R.B. Сотрудник читает имена работников, указанные на внешних конвертах.У Amazon и профсоюза есть возможность оспорить право каждого работника на участие в программе.

    • После того, как Amazon и профсоюз начали обсуждать спорных избирателей, N.L.R.B. подсчитывает неопрошенные бюллетени анонимно и вручную на видеоконференции, открытой для журналистов. Это может начаться сегодня.

    Магазин Samsung в Сеуле. Серия телефонов Galaxy S21 компании хорошо продается в Соединенных Штатах с момента их появления в январе. Кредит … Jung Yeon-Je / Agence France-Presse — Getty Images

    Продажи Samsung в первом квартале выросли примерно на 17 процентов по сравнению с годом ранее, а операционная прибыль увеличилась на 44 процента, сообщила компания в среду.Росту южнокорейского гиганта электроники во время пандемии способствовал высокий спрос на телевизоры, компьютерные мониторы и другие предметы домашнего обихода.

    В январе компания выпустила свои последние флагманские смартфоны серии Galaxy S21. По данным Counterpoint Research, в США эти устройства легко превзошли последнюю линейку телефонов премиум-класса от Samsung за первые шесть недель их выхода на рынок.

    Бизнесу Samsung по производству мобильных телефонов в последнее время способствовала также кампания США против Huawei, одного из основных конкурентов компании в области смартфонов. Продажи устройств китайского технологического гиганта резко упали, поскольку американские санкции не позволяют использовать на его телефонах популярные приложения и сервисы Google, что ограничивает их привлекательность для многих покупателей.

    Другой конкурент, LG Electronics, заявил на этой неделе, что уходит из бизнеса смартфонов, чтобы сосредоточиться на других продуктах.

    Выручка Samsung в первом квартале, вероятно, пострадала из-за февральского шторма в Техасе, из-за которого компания на время приостановила производство на своих производственных предприятиях в Остине.

    Ожидается, что компания представит подробные финансовые результаты в конце этого месяца.

    Бывший торговый центр в Вест-Ориндж, штат Нью-Джерси, теперь является центром вакцинации от коронавируса. Международный валютный фонд заявил, что успешные программы вакцинации улучшили перспективы роста стран. Кредит … Джеймс Эстрин / The New York Times

    Фондовые индексы США в среду в основном выросли после потока в основном оптимистичных экономических данных и прогресса в вакцинации.

    S&P 500 прибавил 0.2 процента, в то время как композитный индекс Nasdaq упал менее чем на 0,1 процента. Индексы Stoxx Europe 600 и DAX в Германии упали примерно на 0,2 процента после роста до новых максимумов во вторник.

    Во вторник Международный валютный фонд обновил свой прогноз глобального экономического роста и заявил, что некоторые из самых богатых стран мира возглавят восстановление, особенно Соединенные Штаты, где в настоящее время прогнозируется рост экономики на 6,4 процента в этом году.

    Внедрение вакцин является основной причиной более радужных прогнозов в некоторых странах, в том числе вакцины I.М.Ф. сказал. Президент Байден сказал, что он хочет, чтобы штаты предоставили всем взрослым людям право на вакцинацию к 19 апреля, на две недели раньше, чем он был ранее крайний срок. В Великобритании вакцина Moderna была впервые введена в среду, что сделало ее третьей доступной вакциной.

    Тем не менее, I.M.F. предупредил во вторник о неравномерном выздоровлении из-за неравномерного распределения вакцин по всему миру, при этом ожидается, что некоторые страны с более низким уровнем дохода не смогут вакцинировать свое население в этом году.

    Доходность 10-летних облигаций США выросла до 1,67 процента.

    Цены на нефть незначительно выросли: фьючерсы на американский стандарт West Texas Intermediate поднялись на 0,7% до 59,77 доллара за баррель.

    • Saks Fifth Avenue прекратит продажу изделий из меха животных к концу 2022 финансового года и закроет все свои меховые салоны к концу 2021 финансового года, сообщил в среду ритейлер. Финансовые годы розничных продавцов обычно заканчиваются в январе или феврале, что соответствует сезону праздничных продаж.Розничный торговец заявил, что откажется от продуктов, сделанных из животных, выращенных для использования их меха или сделанных из меха диких животных, но продолжит продавать короткую шерсть, козью кожу, шкуры крупного рогатого скота, пух, перья, изделия из кожи и искусственного меха. . Это последний розничный торговец, выступивший против меха, присоединившись к Macy’s, Michael Kors, Gucci и California.

    Изменение магнитной индукции. Индукционный ток

    В нашем мире все типы существующих сил, за исключением сил гравитации, представлены электромагнитными взаимодействиями.Во Вселенной, несмотря на удивительное разнообразие воздействий тел друг на друга, в любых веществах, живых организмах всегда есть проявление электромагнитных сил … Как происходило открытие электромагнитной индукции (ЭИ), описано ниже. .

    В контакте с

    Открытие EI

    Вращение магнитной стрелки около проводника с током в экспериментах Эрстеда было первым, указавшим на связь между электрическими и магнитными явлениями.Очевидно: электрический ток «окружает» себя магнитным полем.

    Так можно ли добиться его появления с помощью магнитного поля — аналогичную задачу поставил Майкл Фарадей. В 1821 году он отметил это свойство в своем дневнике о превращении магнетизма в.

    Успех пришел к ученому не сразу. Только глубокая уверенность в единстве природных сил и упорный труд привели его десять лет спустя к новому великому открытию.

    Долгое время решение проблемы не давалось Фарадею и другим его коллегам, поскольку они пытались получить электричество в неподвижной катушке, используя действие постоянного магнитного поля.Между тем позже выяснилось: количество силовых линий, пронизывающих провода, меняется, и возникает электричество.

    Феномен EI

    Процесс появления электричества в катушке в результате изменения магнитного поля характерен для электромагнитной индукции и определяет это понятие. Совершенно естественно, что разнообразие, возникающее в ходе этого процесса, называется индукцией. Эффект будет сохраняться, если саму катушку оставить неподвижной, но при этом перемещать магнит.При использовании второй катушки можно вообще обойтись без магнита.

    Если пропустить электричество через одну из катушек, то при их совместном движении во второй появится индукционный ток … Вы можете надеть одну катушку на другую и изменить значение напряжения одной из них, замыкая и открывая ключ . В этом случае магнитное поле, пронизывающее катушку, на которое воздействует ключ, изменяется, и это становится причиной возникновения индукционного тока во втором.

    Закон

    В ходе экспериментов легко обнаружить, что количество силовых линий, пронизывающих катушку, увеличивается — стрелка используемого прибора (гальванометра) смещается в одну сторону, уменьшается в другую.Более подробное исследование показывает, что сила индукционного тока прямо пропорциональна скорости изменения количества силовых линий. Это основной закон электромагнитной индукции.

    Этот закон выражает формулу:

    Применяется, если за период времени t магнитный поток изменяется на ту же величину, когда скорость изменения магнитного потока Ф / t постоянна.

    Важно! Для индукционных токов действует закон Ома: I = / R, где — ЭДС индукции, которая находится по закону ЭИ.

    Замечательные эксперименты, когда-то проведенные известным английским физиком и ставшие основой открытого им закона, сегодня могут без особого труда провести любой школьник. Для этих целей используются:

    • магнит,
    • две катушки с проволокой,
    • источник электроэнергии,
    • Гальванометр
    • .

    Зафиксируем магнит на подставке и подведем к нему катушку концами, присоединенными к гальванометру.

    Поворачивая, наклоняя и перемещая его вверх и вниз, мы изменяем количество силовых линий магнитного поля, пронизывающих его катушки.

    Гальванометр регистрирует появление электричества с постоянно меняющейся величиной и направлением в процессе опыта.

    Катушка и магнит, покоящиеся друг относительно друга, не создают условий для возникновения электричества.

    Прочие законы Фарадея

    На основе проведенных исследований сформировались еще два одноименных закона:

    1. Суть первого состоит в следующем: масса вещества m , генерируемая электрическим напряжением на электроде, пропорциональна количеству электричества Q, прошедшего через электролит.
    2. Определение второго закона Фарадея или зависимости электрохимического эквивалента от атомного веса элемента и его валентности формулируется следующим образом: электрохимический эквивалент вещества пропорционален его атомному весу, а обратно пропорционален валентности. .

    Из всех существующих видов индукции большое значение имеет отдельный вид этого явления — самоиндукция. Если взять катушку, у которой большое количество витков, то при замыкании цепи лампочка загорается не сразу.

    Этот процесс может занять несколько секунд. На первый взгляд очень удивительный факт. Чтобы понять, что здесь происходит, нужно разобраться, что происходит в цепи , замыкающий момент … Замкнутая цепь как бы «пробуждает» электрический ток, который начинает свое движение по виткам провода. В то же время в пространстве вокруг него мгновенно создается увеличивающееся магнитное поле.

    В витки катушки действует изменяющееся электромагнитное поле, которое концентрируется сердечником.Индукционный ток, возбуждаемый в витках катушки с увеличением магнитного поля (в момент замыкания цепи), противодействует основному. Мгновенное достижение его максимального значения в момент замыкания цепи невозможно, оно постепенно «растет». Вот объяснение того, почему лампочка не мигает сразу. Когда цепь разомкнута, основной ток усиливается индукцией в результате явления самоиндукции, и лампочка ярко мигает.

    Важно! Суть явления, называемого самоиндукцией, характеризуется зависимостью изменения возбуждающего индукционный ток электромагнитного поля от изменения силы электрического тока, протекающего по цепи.

    Направление тока самоиндукции определяет правило Ленца. Самоиндукция легко сопоставима с инерцией в области механики, поскольку оба явления имеют схожие характеристики. Действительно, в в результате инерции под действием силы тело приобретает определенную скорость постепенно, а не мгновенно.Не сразу — под действием самоиндукции — при включении аккумулятора в цепь тоже появляется электричество. Продолжая сравнение со скоростью, отметим, что он тоже не умеет мгновенно исчезать.

    Вихревые токи

    Наличие вихревых токов в массивных проводниках может служить еще одним примером электромагнитной индукции.

    Специалисты знают, что металлические сердечники трансформаторов, якоря генераторов и электродвигатели никогда не бывают твердыми. Когда они изготовлены, на отдельные тонкие листы, из которых они состоят, наносится слой лака, изолирующий один лист от другого.

    Нетрудно понять какая сила заставляет человека создать именно такое устройство … Под действием электромагнитной индукции в переменном магнитном поле сердечник протыкается силовыми линиями вихревого электрического поля.

    Представим, что сердечник сделан из цельного металла. Поскольку его электрическое сопротивление невелико, возникновение большого индукционного напряжения было бы вполне объяснимо. Ядро в конечном итоге нагревается, и большая часть электричества расходуется напрасно.Кроме того, потребуются специальные меры по охлаждению. А изоляционные слои не позволяют достичь больших значений .

    Индукционные токи, присущие массивным проводникам, не случайно называют вихревыми токами — их линии замкнуты, как силовые линии электрического поля, где они возникают. Чаще всего вихревые токи используются при работе индукционных металлургических печей для плавки металлов. Взаимодействуя с создавшим их магнитным полем, они иногда становятся причиной занимательных явлений.

    Возьмем мощный электромагнит и поместим между его вертикально расположенными полюсами, например, пятикопеечную монету. Вопреки ожиданиям не упадет, а будет медленно спускаться. Ей потребуется несколько секунд, чтобы преодолеть несколько сантиметров.

    Поместим, например, пятикопеечную монету между вертикально расположенными полюсами мощного электромагнита и отпустим ее.

    Вопреки ожиданиям не упадет, а будет медленно спускаться. Ей потребуется несколько секунд, чтобы преодолеть несколько сантиметров. Движение монеты напоминает движение тела в вязкой среде. Почему это происходит.

    Согласно правилу Ленца, направления вихревых токов, возникающих при движении монеты в неоднородном магнитном поле, таковы, что поле магнита толкает монету вверх. Эта функция используется для «успокоения» стрелок в измерительных приборах. К стрелке прикреплена алюминиевая пластина, расположенная между полюсами магнита, и возникающие в ней вихревые токи способствуют быстрому гашению колебаний.

    Демонстрация феномена удивительной красоты электромагнитной индукции , предложенная профессором МГУ В.К. Аркадьев. Возьмите чашу из сверхпроводящего свинца и попробуйте опустить на нее магнит. Он не упадет, а будет «парить» над чашей. Объяснение здесь простое: нулевое электрическое сопротивление сверхпроводника способствует появлению в нем электричества большой величины, которое может сохраняться длительное время и «удерживать» магнит над чашей.Согласно правилу Ленца, направление их магнитного поля таково, что оно отталкивает магнит и не позволяет ему упасть.

    Изучаем физику — закон электромагнитной индукции

    Формулировка закона Фарадея верна

    Выход

    Электромагнитные силы — это силы, которые позволяют людям видеть окружающий мир, и они чаще встречаются в природе, например, свет также является примером электромагнитных явлений. Без этого явления невозможно представить жизнь человечества.

    На рисунке показано направление индукционного тока, который возникает в катушке с короткозамкнутым проводом при перемещении относительно нее.

    магнит. Обратите внимание, какие из следующих утверждений верны, а какие нет.
    A. Магнит и катушка притягиваются друг к другу.
    B. Внутри катушки магнитное поле индукционного тока направлено вверх.
    B. Внутри катушки линии магнитной индукции магнитного поля направлены вверх.
    D. Магнит снимается с катушки.

    1. Первый закон Ньютона?

    2. Какие системы отсчета являются инерционными и неинерциальными? Приведите примеры.
    3. Какое свойство тел называется инертностью? Каков размер инерции?
    4. Какова связь между массами тел и модулями ускорений, которые они получают при взаимодействии?
    5. Что такое сила и как она характеризуется?
    6. Формулировка 2 закона Ньютона? Каковы его математические обозначения?
    7. Как закон Ньютона 2 формулируется в импульсной форме? Его математические обозначения?
    8.Что такое 1 Ньютон?
    9. Как движется тело, если к нему приложена сила, постоянная по величине и направлению? Как направлено ускорение, вызванное действующей на него силой?
    10. Как определяется равнодействующая сил?
    11. Как сформулированы и записаны 3 закона Ньютона?
    12. Как направлены ускорения взаимодействующих тел?
    13. Приведите примеры проявления 3 закона Ньютона.
    14. Каковы пределы применимости всех законов Ньютона?
    15.Почему мы можем рассматривать Землю как инерциальную систему отсчета, если она движется с центростремительным ускорением?
    16. Что такое деформация, какие виды деформации вам известны?
    17. Какая сила называется силой упругости? Какова природа этой силы?
    18. Каковы особенности силы упругости?
    19. Как направлена ​​сила упругости (сила реакции опоры, сила натяжения нити?)
    20. Как формулируется и записывается закон Гука? Каковы пределы его применимости? Постройте график, чтобы проиллюстрировать закон Гука.
    21. Как формулируется и записывается закон всемирного тяготения, когда он применим?
    22. Опишите эксперименты по определению значения гравитационной постоянной?
    23. Что такое гравитационная постоянная, каков ее физический смысл?
    24. Зависит ли работа силы тяжести от формы траектории? Что такое работа силы тяжести в замкнутом контуре?
    25. Зависит ли работа упругой силы от формы траектории?
    26. Что вы знаете о гравитации?
    27.Как рассчитывается ускорение свободного падения на Земле и других планетах?
    28. Какая первая космическая скорость? Как рассчитывается?
    29. Что называется свободным падением? Зависит ли ускорение свободного падения от веса тела?
    30. Опишите опыт Галилео Галилея, доказавший, что все тела в вакууме падают с одинаковым ускорением.
    31. Какая сила называется силой трения? Типы сил трения?
    32. Как рассчитывается сила трения скольжения и качения?
    33.Когда возникает сила статического трения? Чему это равно?
    34. Зависит ли сила трения скольжения от площади соприкасающихся поверхностей?
    35. От каких параметров зависит сила трения скольжения?
    36. От чего зависит сила сопротивления движению тела в жидкостях и газах?
    37. Что называется массой тела? В чем разница между массой тела и силой тяжести, действующей на тело?
    38. В каком случае вес тела численно равен модулю силы тяжести?
    39.Что такое невесомость? Что такое перегрузка?
    40. Как рассчитать вес тела при его ускоренном движении? Изменится ли вес тела, если оно движется по фиксированной горизонтальной плоскости с ускорением?
    41. Как изменяется вес тела при движении по выпуклой и вогнутой частях окружности?
    42. Каков алгоритм решения задач, когда тело движется под действием нескольких сил?
    43. Какая сила называется силой Архимеда или силой плавучести? От каких параметров зависит эта сила?
    44.По каким формулам можно рассчитать силу Архимеда?
    45. При каких условиях тело в жидкости плавает, тонет, плавает?
    46. Как зависит глубина погружения плавающего тела в жидкость от его плотности?
    47. Почему воздушные шары наполнены водородом, гелием или горячим воздухом?
    48. Объясните влияние вращения Земли вокруг своей оси на величину ускорения свободного падения.
    49. Как изменяется величина силы тяжести, когда: а) тело удаляется от поверхности Земли, Б) когда тело движется по меридиану, параллельному

    электрическая схема?

    3.В чем физический смысл ЭМП? Определите вольт.

    4. Подключите вольтметр на короткое время к источнику электрической энергии, соблюдая полярность. Сравните его показания с расчетом по результатам эксперимента.

    5. От чего зависит напряжение на выводах источников тока?

    6. По результатам измерений определить напряжение на внешней цепи (если работа ведется по I способу), сопротивление внешней цепи (если работа выполняется по II методу).

    6 вопросов по вычислению вложенности

    Помогите пожалуйста!

    1. При каких условиях возникают силы трения?
    2. Что определяет модуль и направление силы статического трения?
    3. В каких пределах может изменяться сила трения покоя?
    4. Какая сила сообщает ускорение автомобилю или тепловозу?
    5. Может ли сила трения скольжения увеличить скорость тела?
    6. В чем основное отличие силы сопротивления в жидкостях и газах от силы трения между двумя твердыми телами?
    7.Приведите примеры полезных и вредных эффектов сил трения всех типов

    ИНДУКЦИОННЫЙ ТОК — электрический ток, возникающий при изменении потока магнитной индукции в замкнутом проводящем контуре. Это явление называется электромагнитной индукцией. Вы хотите знать, в каком направлении индукционный ток? Росиндуктор — это торговый информационный портал, где вы найдете информацию о текущих ценах.

    Правило, определяющее направление индукционного тока, выглядит следующим образом: «Индукционный ток направлен так, чтобы противодействовать изменению магнитного потока, вызванному его магнитным полем.«Правая рука повернута ладонью к магнитным силовым линиям, а большой палец направлен в сторону движения проводника, а четыре пальца показывают, в каком направлении будет течь индукционный ток. Перемещая проводник, мы движемся вместе с проводником. проводят все содержащиеся в нем электроны, и при движении в магнитном поле электрических зарядов на них будет действовать сила по правилу левой руки.

    Направление индукционного тока, как и его величина, определяется правилом Ленца, согласно которому направление индукционного тока всегда ослабляет влияние фактора, возбуждающего ток.Когда поток магнитного поля изменяется в цепи, направление индукционного тока будет таким, чтобы компенсировать эти изменения. Когда магнитное поле, возбуждающее ток в цепи, создается в другой цепи, направление индукционного тока зависит от характера изменений: с увеличением внешнего тока индукционный ток имеет противоположное направление, с уменьшением, он направлен в том же направлении и имеет тенденцию увеличивать поток.

    Катушка индукционного тока имеет два полюса (северный и южный), которые определяются в зависимости от направления тока: индукционные линии выходят из северного полюса.Приближение магнита к катушке создает ток в направлении, которое отталкивает магнит. Когда магнит удален, ток в катушке имеет направление, которое будет притягивать магнит.


    Индукционный ток возникает в замкнутом контуре в переменном магнитном поле. Контур может быть как неподвижным (помещенный в изменяющийся поток магнитной индукции), так и движущимся (движение контура вызывает изменение магнитного потока). Возникновение индукционного тока вызывает вихревое электрическое поле, которое возбуждается магнитным полем.

    Научиться создавать кратковременный индукционный ток можно из школьного курса физики.

    Это можно сделать несколькими способами:

    • — перемещение постоянного магнита или электромагнита относительно катушки,
    • — движение сердечника относительно вставленного в катушку электромагнита,
    • — замыкание и размыкание цепи,
    • — регулировка тока в цепи.


    Основной закон электродинамики (закон Фарадея) гласит, что сила индукционного тока для любой цепи равна скорости изменения магнитного потока, проходящего через цепь, взятую со знаком минус.Сила индукционного тока называется электродвижущей силой.


    Возникновение в проводнике индукции ЭДС

    Если вы поместите его в проводник и переместите так, чтобы он пересекал силовые линии во время своего движения, то в проводнике появится ЭДС индукции.

    ЭДС индукции будет возникать в проводнике, даже если сам проводник остается неподвижным, и магнитное поле будет двигаться, пересекая проводник с его силовыми линиями.

    Если проводник, в котором индуцируется ЭДС индукции, замкнут на какую-либо внешнюю цепь, то под действием этой ЭДС через цепь будет протекать ток, называемый индукционным током.

    Явление индукции ЭДС в проводнике, когда он пересекает силовые линии его магнитного поля, называется электромагнитной индукцией.

    Электромагнитная индукция — это обратный процесс, то есть преобразование механической энергии в электрическую.

    Явление электромагнитной индукции нашло широкое применение в России.На его использовании основано устройство различных электрических машин.

    Величина и направление индукции ЭДС

    Давайте теперь посмотрим, какими будут величина и направление ЭДС, индуцированной в проводнике.

    Величина ЭДС индукции зависит от количества силовых линий поля, пересекающих проводник в единицу времени, то есть от скорости проводника в поле.

    Величина наведенной ЭДС прямо пропорциональна скорости движения проводника в магнитном поле.

    Величина наведенной ЭДС также зависит от длины той части проводника, через которую проходят силовые линии поля. Чем больше часть проводника пересекается силовыми линиями, тем больше ЭДС наводится в проводнике. И, наконец, чем сильнее магнитное поле, то есть чем больше его индукция, тем большая ЭДС возникает в проводнике, пересекающем это поле.

    Итак, величина ЭДС индукции, которая возникает в проводнике, когда он движется в магнитном поле, прямо пропорциональна индукции магнитного поля, длине проводника и скорости его движения.

    Эта зависимость выражается формулой E = Blv,

    где E — ЭДС индукции; Б — магнитная индукция; I — длина проводника; v — скорость движения проводника.

    Следует твердо помнить, что в проводнике, движущемся в магнитном поле, ЭДС индукции возникает только в том случае, если этот проводник пересекается силовыми линиями магнитного поля. Если проводник движется по силовым линиям поля, то есть не пересекает, а как бы скользит по ним, то ЭДС в нем не наводится.Следовательно, приведенная выше формула верна только тогда, когда проводник движется перпендикулярно силовым линиям магнитного поля.

    Направление наведенной ЭДС (а также тока в проводнике) зависит от того, в каком направлении движется проводник. Существует правило правой руки для определения направления наведенной ЭДС.

    Если держать ладонь правой руки так, чтобы в нее входили силовые линии магнитного поля, а согнутый большой палец указывал бы направление движения проводника, то четыре вытянутых пальца укажут направление действия наведенной ЭДС и направление тока в проводнике.

    Правое правило

    Индукция ЭДС в катушке

    Мы уже говорили, что для создания ЭДС индукции в проводнике необходимо перемещать либо сам проводник, либо магнитное поле в магнитном поле. В обоих случаях провод должен пересекаться силовыми линиями магнитного поля, иначе ЭДС не будет индуцироваться. Индуцированная ЭДС и, следовательно, индукционный ток могут быть получены не только в прямом проводе, но и в проводнике, скрученном в катушку.

    При движении внутри постоянного магнита в нем наводится ЭДС из-за того, что магнитный поток магнита пересекает витки катушки, то есть так же, как это было при движении прямолинейного проводника в поле магнита.

    Если опускать магнит в катушку медленно, то возникающая в нем ЭДС будет настолько мала, что стрелка прибора может даже не отклониться. Если же наоборот, быстро вставить магнит в катушку, то отклонение стрелки будет большим.Это означает, что величина наведенной ЭДС и, следовательно, сила тока в катушке зависит от скорости магнита, то есть от того, как быстро силовые линии пересекают витки катушки. Если теперь поочередно ввести в катушку сначала сильный, а затем слабый магнит с одинаковой скоростью, то вы заметите, что при сильном магните стрелка устройства отклонится на больший угол. Значит, величина наведенной ЭДС, а, следовательно, и сила тока в катушке зависит от величины магнитного потока магнита.

    И, наконец, если тот же магнит ввести с той же скоростью сначала в катушку с большим числом витков, а затем с гораздо меньшим числом, то в первом случае стрелка устройства отклонится на большее угол, чем во втором. Это означает, что величина наведенной ЭДС, а, следовательно, и сила тока в катушке зависит от количества ее витков. Такие же результаты можно получить, если использовать электромагнит вместо постоянного магнита.

    Направление индукции ЭДС в катушке зависит от направления движения магнита.Как определить направление ЭДС индукции, гласит закон, установленный Э. Х. Ленцем.

    Закон Ленца для электромагнитной индукции

    Любое изменение магнитного потока внутри катушки сопровождается появлением в ней ЭДС индукции, и чем быстрее изменяется магнитный поток, проникающий в катушку, тем больше в ней индуцируется ЭДС.

    Если катушка, в которой создается ЭДС индукции, замкнута на внешнюю цепь, то по ее виткам протекает индукционный ток, создавая вокруг проводника магнитное поле, благодаря которому катушка превращается в соленоид.Получается, что изменяющееся внешнее магнитное поле вызывает индукционный ток в катушке, который, в свою очередь, создает вокруг катушки собственное магнитное поле — текущее поле.

    Изучая это явление, Э. Х. Ленц установил закон, определяющий направление индукционного тока в катушке, а, следовательно, и направление индукционной ЭДС. ЭДС индукции, возникающая в катушке при изменении в ней магнитного потока, создает в катушке ток в таком направлении, при котором магнитный поток катушки, создаваемый этим током, предотвращает изменение постороннего магнитного потока.

    Закон Ленца справедлив для всех случаев индукции тока в проводниках, независимо от формы проводников и того, как достигается изменение внешнего магнитного поля.


    Когда постоянный магнит движется относительно проволочной катушки, соединенной с выводами гальванометра, или когда катушка движется относительно магнита, генерируется индукционный ток.

    Индукционные токи в массивных проводниках

    Изменяющийся магнитный поток способен вызывать ЭДС не только в витках катушки, но и в массивных металлических проводниках.Проникая в толщу массивного проводника, магнитный поток наводит в нем ЭДС, которая создает индукционные токи. Эти так называемые распространяются по массивному проводнику и замыкаются в нем накоротко.

    Сердечники трансформаторов, магнитопроводы различных электрических машин и устройств — это как раз те массивные проводники, которые нагреваются возникающими в них индукционными токами. Это явление нежелательно, поэтому для уменьшения величины индукционных токов детали электрических машин и сердечники трансформаторов делают не массивными, а состоящими из тонких листов, изолированных друг от друга бумагой или слоем изоляционного лака.За счет этого блокируется путь распространения вихревых токов по массе проводника.

    Но иногда на практике вихревые токи также используются как полезные токи. Использование этих токов основано, например, на работе так называемых магнитных демпферов движущихся частей электроизмерительных приборов.

    Тема 11. ЯВЛЕНИЕ ЭЛЕКТРОМАГНИТНОЙ ИНДУКЦИИ.

    11.1. Эксперименты Фарадея. Индукционный ток. Правило Ленца. 11.2. Величина индукции ЭДС.

    11.3. Природа индукции ЭДС.

    11,4. Циркуляция вектора напряженности электрического поля вихря.

    11,5. Бетатрон.

    11.6. Токи Фуко.

    11.7. Скин-эффект. №

    С того момента, как была обнаружена связь магнитного поля с током (что является подтверждением симметрии законов природы), были предприняты многочисленные попытки получить ток с помощью магнитного поля. Проблема была решена Майклом Фарадеем в 1831 году.(Американец Джозеф Генри также обнаружил, но не смог опубликовать свои результаты. Ампер также заявил об открытии, но не смог представить свои результаты).

    ФАРАДЕЙ Майкл (1791 — 1867) — известный английский физик. Исследования в области электричества, магнетизма, магнитооптики, электрохимии. Создан лабораторный образец электродвигателя. Я открывал лишние токи при замыкании и размыкании цепи и задавал их направление. Он открыл законы электролиза, первым ввел понятия поля и диэлектрической проницаемости, в 1845 году употребил термин «магнитное поле».

    Среди прочего М. Фарадей открыл явления диам и парамагнетизма. Он обнаружил, что все материалы в магнитном поле ведут себя по-разному: они ориентированы вдоль поля (пар и ферромагнетики) или через

    полей — диамагнетики.

    Из школьного курса физики хорошо известны опыты Фарадея: катушка и постоянный магнит (рис. 11.1).

    Рис. 11.1 Рис. 11.2

    Если поднести к катушке магнит или наоборот, то в катушке появится электрический ток.То же самое и с двумя близко расположенными катушками: если вы подключите источник переменного тока к одной из катушек, то переменный ток также появится в другой.

    (рисунок 11.2), но этот эффект лучше всего проявляется, если две катушки соединены сердечником (рисунок 11.3).

    Согласно определению Фарадея, обычным явлением для этих экспериментов является следующее: если поток

    вектора индукции, проникая в замкнутую проводящую цепь, изменяется, то в цепи возникает электрический ток.

    Это явление называется явлением электромагнитной индукции, а тока — индуктивным … При этом явление совершенно не зависит от способа изменения потока вектора магнитной индукции.

    Итак, получается, что движущиеся заряды (ток) создают магнитное поле, а движущееся магнитное поле создает (вихревое) электрическое поле и, собственно, индукционный ток.

    Для каждого конкретного случая Фарадей указал направление индукционного тока.В 1833 году Ленц установил общее правило направления тока:

    индукционный ток всегда направлен таким образом, что магнитное поле этого тока предотвращает изменение магнитного потока, вызывающее индукционный ток. Это утверждение называется правилом Ленца.

    Заполнение всего пространства однородным магнитом при прочих равных приводит к увеличению индукции в µ раз. Этот факт подтверждает, что индукционный ток

    обусловлен изменением потока вектора магнитной индукции B, а не потока вектора напряженности H.

    11.2. Величина индукции ЭДС.

    Для создания тока в цепи необходимо наличие электродвижущей силы. Следовательно, явление электромагнитной индукции указывает на то, что при изменении магнитного потока в цепи возникает электродвижущая сила индукции E i. Наша задача

    , используя законы сохранения энергии, найти значение E i и найти

    Рассмотрим движение движущегося участка 1-2 контуров с током в магнитном поле

    В (рис.11.4).

    Во-первых, пусть магнитное поле B отсутствует. Аккумулятор с ЭДС, равной E 0, создает ток I 0

    . За время dt аккумулятор выполняет свою работу

    dA = E I0 dt (11.2.1)

    — эта работа преобразуется в тепло, которое можно найти согласно закону Джоуля-Ленца:

    Q = dA = E 0 I0 dt = I0 2 Rdt,

    здесь I 0 = ER 0, R — полное сопротивление всей цепи.

    Помещаем схему в однородное магнитное поле с индукцией В.Строки B || n и связаны с направлением тока по правилу подвеса. Поток Ф, связанный с контуром, положительный. R

    На каждый элемент контура действует механическая сила d F. На движущуюся сторону рамы действует сила F 0. Под действием этой силы участок 1-2

    будет двигаться со скоростью υ = dx dt . В данном случае магнитный поток

    индукции.

    Тогда в результате электромагнитной индукции ток в цепи изменится и составит

    ).Эта сила за время dt произведет работу dA: dA = Fdx = IdФ.

    Как и в случае, когда все элементы каркаса неподвижны, источник работы E 0.

    При стационарном контуре эта работа сводилась только к выделению тепла. В нашем случае тепло тоже будет выделяться, но в другом количестве, так как изменился ток. Кроме того, выполняются механические работы. Суммарная работа за время dt равна:

    E 0 Idt = I2 R dt + I dФ

    Умножьте левую и правую части этого выражения на

    .

    Получаем

    Имеем право рассматривать получившееся выражение как закон Ома для цепи, в которой помимо источника E 0 действует E i, что равно:

    Индукционная цепь ЭДС (E i)

    равна скорости изменения магнитного потока

    индукция, пронизывающая эту цепь.

    Это выражение для ЭДС индукции контура полностью универсально, не зависит от способа изменения потока магнитной индукции и называется

    законом Фарадея.

    Знак (-) — математическое выражение правила Ленца о направлении индукционного тока: индукционный ток всегда направлен так, чтобы его поле

    противодействовало изменению начального магнитного поля.

    Направление индукционного тока и направление d dt Ф соединены правилом буравчика (рис.11.5).

    Размер индукции ЭДС: [E i] = [Ф] = B c = B .tc

    Если контур состоит из нескольких витков, то необходимо использовать концепцию магнитосцепления

    (полный магнитный поток):

    Ψ = Ф N,

    где N — количество витков. Итак, если

    E i = –

    ∑Ф и

    i = 1

    ∑ Ф = Ψ

    Ei = —

    11.3. Природа индукции ЭДС.

    Ответим на вопрос, в чем причина движения зарядов, причина возникновения индукционного тока? Рассмотрим рисунок 11.6.

    1) Если перемещать проводник в однородном магнитном поле B, то под действием силы Лоренца электроны отклонятся вниз, а положительные заряды вверх — возникает разность потенциалов. Это будет сила на стороне E i, под действием которого течет ток

    . Как известно, для положительных зарядов

    F l = q +; для электронов F l = –e -.

    2) Если проводник неподвижен, а магнитное поле меняется, какая сила возбуждает индукционный ток в этом случае? Возьмем обычный трансформатор (рисунок 11.7).

    Как только мы замыкаем цепь первичной обмотки, во вторичной обмотке сразу возникает ток. Но ведь сила Лоренца тут ни при чем, потому что она действует на движущиеся заряды, а они вначале были в покое (они были в тепловом движении — хаотическом, но здесь нужно направленное движение).

    Ответ был дан Дж. Максвеллом в 1860 году: любое переменное магнитное поле возбуждает электрическое поле (E «) в окружающем пространстве. Это причина индукционного тока в проводнике. То есть E» возникает только в наличие переменного магнитного поля (трансформатор не работает с постоянным током).

    Суть явления электромагнитной индукции вовсе не в появлении индукционного тока (ток появляется при наличии зарядов и замкнутой цепи), а в появлении вихревого электрического поля (не только в проводнике, но также в окружающем пространстве, в вакууме).

    Это поле имеет совершенно иную структуру, чем поле, созданное начислениями. Поскольку он не создается зарядами, силовые линии не могут начинаться и заканчиваться на зарядах, как это было в нашей электростатике. Это поле вихревое, его силовые линии замкнуты.

    Поскольку это поле перемещает заряды, значит, оно имеет силу. Введем вектор напряженности вихревого электрического поля E «

    . Сила, с которой это поле действует на заряд

    F» = q E «.

    Но когда заряд движется в магнитном поле, на него действует сила Лоренца

    F «= q.

    Добавить комментарий

    Ваш адрес email не будет опубликован. Обязательные поля помечены *

    2024 © Все права защищены.